Exam 1

Réussis tes devoirs et examens dès maintenant avec Quizwiz!

Which description of the phases of the second stage of labor is accurate?

Descent phase: Significant increase in contractions, Ferguson reflux activated, average duration varied

When assessing a woman in the first stage of labor, the nurse recognizes that the most conclusive sign that uterine contractions are effective would be:

Dilation of the cervix.

The maternity nurse understands that as the uterus contracts during labor, maternal-fetal exchange of oxygen and waste products:

Diminishes as the spiral arteries are compressed.

A primigravida at 39 weeks of gestation is observed for 2 hours in the intrapartum unit. The fetal heart rate has been normal. Contractions are 5 to 9 minutes apart, 20 to 30 seconds in duration, and of mild intensity. Cervical dilation is 1 to 2 cm and uneffaced (unchanged from admission). Membranes are intact. The nurse should expect the woman to be:

Discharged home to await the onset of true labor

In relation to primary and secondary powers, the maternity nurse comprehends that:

Primary powers are responsible for effacement and dilation of the cervix.

Fetal well-being during labor is assessed by: 1 the response of the fetal heart rate (FHR) to uterine contractions (UCs). 2 maternal pain control. 3 accelerations in the FHR. 4 an FHR greater than 110 beats/min

1 Fetal well-being during labor is measured by the response of the FHR to UCs . In general, reassuring FHR patterns are characterized by an FHR baseline in the range of 110 to 160 beats/min with no periodic changes, a moderate baseline variability, and accelerations with fetal movement. Maternal pain control is not the measure used to determine fetal well-being in labor. Although FHR accelerations are a reassuring pattern, they are only one component of the criteria by which fetal well-being is assessed. Although an FHR greater than 110 beats/min may be reassuring, it is only one component of the criteria by which fetal well-being is assessed. More information is needed to determine fetal well-being. Test-Taking Tip: Being emotionally prepared for an examination is key to your success. Proper use of this text over an extended period of time ensures your understanding of the mechanics of the examination and increases your confidence about your nursing knowledge. Your lifelong dream of becoming a nurse is now within your reach! You are excited, yet anxious. This feeling is normal. A little anxiety can be good because it increases awareness of reality; but excessive anxiety has the opposite effect, acting as a barrier and keeping you from reaching your goal. Your attitude about yourself and your goals will help keep you focused, adding to your strength and inner conviction to achieve success.

A woman diagnosed with marginal placenta previa gave birth vaginally 15 minutes ago. At the present time, she is at the greatest risk for: 1 hemorrhage. 2 infection. 3 urinary retention. 4 thrombophlebitis.

1 Hemorrhage is the most immediate risk because the lower uterine segment has limited ability to contract to reduce blood loss. Infection is a risk because of the location of the placental attachment site; however, it is not a priority concern at this time. Placenta previa poses no greater risk for urinary retention than with a normally implanted placenta. There is no greater risk for thrombophlebitis than with a normally implanted placenta.

The quantitative human chorionic gonadotropin (β-hCG) levels are high in a patient who is on methotrexate therapy for dissolving abdominal pregnancy. Which instruction does the nurse give to this patient? 1 "Avoid sexual activity." 2 "Avoid next pregnancy." 3 "Avoid feeling sad and low." 4 "Take folic acid without fail."

1 High β-hCG levels indicate that the abdominal pregnancy is not yet dissolved. Therefore the nurse advises the patient to avoid sexual activity until the β-hCG levels drop and the pregnancy is dissolved completely. If the patient engages in vaginal intercourse, the pelvic pressure may rupture the mass and cause pain. Abdominal pregnancy increases the chances of infertility or recurrent ectopic pregnancy in patients. However, the nurse need not instruct the patient to avoid further pregnancy, because it may increase the feelings of sadness and guilt in the patient. The nurse encourages the patient to share feelings of guilt or sadness related to pregnancy loss. Folic acid is contraindicated with methotrexate therapy, because it may exacerbate ectopic rupture.

Which condition does the nurse expect to find in the medical record of a pregnant patient with diabetes mellitus? 1 Hypoglycemia 2 Cyanosis of lips 3 Increasing fatigue 4 Generalized edema

1 In the first trimester of pregnancy, there is an increase in insulin production and a decrease in hepatic glucose production because of the rising levels of estrogen and progesterone. This lowers the glucose levels and makes the pregnant patient prone to hypoglycemia. Cyanosis of lips, increasing fatigue, and generalized edema are signs of cardiac decompensation in a pregnant patient with preexisting cardiac disease. It occurs because the heart is unable to maintain sufficient cardiac output because of pregnancy.

The nurse is caring for a patient who is administered local perineal infiltration anesthesia. In what situation does the nurse expect the use of local perineal infiltration anesthesia? When a(n): 1 Episiotomy is required. 2 Forceps birth is expected. 3 Cesarean birth is expected. 4 Vacuum extractor is to be used.

1 Local perineal infiltration anesthesia may be used when an episiotomy is to be performed. It may also be used when lacerations must be sutured after birth in a patient who does not have regional anesthesia. Pudendal nerve block is administered late in the second stage of labor if an episiotomy is to be performed or if forceps or a vacuum extractor is to be used to facilitate birth. Low spinal anesthesia (block) may be used for cesarean birth.

A patient who is pregnant for the first time is anxious about the pain related to labor. Which physiologic factor does the nurse relate that may increase the intensity of pain during childbirth? 1 History of dysmenorrhea 2 Low level of prostaglandin 3 Cramps during menstruation 4 High level of β-endorphin

1 Patients with a history of dysmenorrhea may experience increased pain during childbirth. These patients are known to have high levels of prostaglandin. Low levels of prostaglandin do not increase the intensity of pain during labor. The level of beta (β) endorphins increases during pregnancy and birth. β endorphins are endogenous opioids that reduce pain. Back pain associated with menstruation also increases the likelihood of contraction-related low back pain.

Which condition is seen in a pregnant patient if uterine artery Doppler measurements in the second trimester of pregnancy are abnormal? 1 Preeclampsia 2 HELLP syndrome 3 Molar pregnancy 4 Gestational hypertension

1 Preeclampsia is a condition in which patients develop hypertension and proteinuria after 20 weeks' gestation. It can be diagnosed if uterine artery Doppler measurements in the second trimester of pregnancy are abnormal. HELLP syndrome is characterized by hemolysis (H), elevated liver enzymes (EL), and low platelet count (LP) in a patient with preeclampsia. Molar pregnancy refers to the growth of the placental trophoblast due to abnormal fertilization. Gestational hypertension is a condition in which hypertension develops in a patient after 20 weeks' gestation.

Which factor is known to increase the risk of gestational diabetes mellitus? 1 Previous birth of large infant 2 Maternal age younger than 25 3 Underweight before pregnancy 4 Previous diagnosis of type 2 diabetes mellitus

1 Previous birth of a large infant suggests gestational diabetes mellitus. A woman younger than 25 is not at risk for gestational diabetes mellitus. Obesity (greater than 90 kg or 198 lb) creates a higher risk for gestational diabetes. The person with type 2 diabetes mellitus already has diabetes and will continue to have it after pregnancy. Insulin may be required during pregnancy because oral hypoglycemia drugs are contraindicated during pregnancy. Test-Taking Tip: Be alert for details. Details provided in the stem of the item, such as behavioral changes or clinical changes (or both) within a certain time period, can provide a clue to the most appropriate response or (in some cases) responses.

Nurses should be aware of the difference experience can make in labor pain, such as: 1 sensory pain for nulliparous women often is greater than for multiparous women during early labor. 2 affective pain for nulliparous women usually is less than for multiparous women throughout the first stage of labor. 3 women with a history of substance abuse experience more pain during labor. 4 multiparous women have more fatigue from labor and therefore experience more pain.

1 Sensory pain is greater for nulliparous women because their reproductive tract structures are less supple. Affective pain is greater for nulliparous women during the first stage but decreases for both nulliparous and multiparous during the second stage. Women with a history of substance abuse experience the same amount of pain as those without such a history. Nulliparous women have longer labors and therefore experience more fatigue. Test-Taking Tip: Be alert for grammatical inconsistencies. If the response is intended to complete the stem (an incomplete sentence) but makes no grammatical sense to you, it might be a distractor rather than the correct response. Question writers typically try to eliminate these inconsistencies.

The primary health care provider has administered terbutaline (Brethine) to a pregnant patient to postpone preterm labor. What changes would the nurse observe in the fetal heart monitor after this drug was administered? 1 Increase in fetal heart rate 2 Decrease in fetal heart rate 3 Accelerations in heart rate 4 Decelerations in heart rate

1 Terbutaline (Brethine) is usually prescribed to postpone labor, because the drug reduces the frequency of uterine contractions. Terbutaline (Brethine) can also increase the fetal heart rate (FHR). Terbutaline (Brethine) does not decrease the heart rate, nor does it cause any accelerations or decelerations in the FHR. Heart block or viral infections can decrease the FHR and may result in bradycardia. There may be accelerations in the FHR during a vaginal examination. A parasympathetic response may cause decelerations in heart rate. Terbutaline is a sympathomimetic drug and thus does not cause decelerations in FHR.

When the nurse observes this fetal heart pattern, the most important nursing action is to: 1 document the finding. 2 position mother on left side. 3 apply 10 L of oxygen via face mask. 4 notify the health care provider

1 The fetal heart strip shows an early deceleration indicating expected head compression during contractions. Documenting this finding is appropriate. Positioning the woman on the left aside, applying oxygen via a face mask, and notifying the health care provider are correct actions for a late deceleration.

The nurse is teaching pain relief techniques to a group of expectant patients. What does the nurse teach the patients about the gate-control theory of pain? 1 Distractions block the nerve pathways. 2 Neuromuscular activity can increase pain. 3 All sensations travel together to the brain. 4 Motor activity during labor intensifies pain

1 The gate-control theory of pain explains the way pain relief techniques work to relieve the pain of labor. Distractions close down a hypothetic gate in the spinal cord, thus preventing pain signals from reaching the brain. According to this theory only a limited number of sensations can travel through the sensory nerve pathways to the brain at one time. When the laboring patient engages in motor activity and neuromuscular activity, activity within the spinal cord itself further modifies the transmission of pain. Test-Taking Tip: Multiple-choice questions can be challenging, because students think that they will recognize the right answer when they see it or that the right answer will somehow stand out from the other choices. This is a dangerous misconception. The more carefully the question is constructed, the more each of the choices will seem like the correct response.

Which instructions does the nurse give to a patient who is prescribed methotrexate therapy for dissolving the tubal pregnancy? 1 "Discontinue folic acid supplements." 2 "Get adequate exposure to sunlight." 3 "Take stronger analgesics for severe pain." 4 "Vaginal intercourse is safe during the therapy."

1 The nurse advises the patient to discontinue folic acid supplements as they interact with methotrexate and may exacerbate ectopic rupture in the patient. Exposure to sunlight is avoided as the therapy makes the patient photosensitive. Analgesics stronger than acetaminophen are avoided, because they may mask symptoms of tubal rupture. Vaginal intercourse is avoided until the pregnancy is dissolved completely.

Which clinical reports does the nurse evaluate to identify ectopic pregnancy in a patient? Select all that apply. 1 Quantitative human chorionic gonadotropin (β-hCG) levels 2 Transvaginal ultrasound 3 Progesterone level 4 Thyroid test reports 5 Kleihauer-Betke (KB) test

1, 2, 3 An ectopic pregnancy is indicated when β-hCG levels are >1500 milli-international units/mL but no intrauterine pregnancy is seen on the transvaginal ultrasound. A transvaginal ultrasound is repeated to verify if the pregnancy is inside the uterus. A progesterone level <5 ng/mL indicates ectopic pregnancy. Thyroid test reports need to be evaluated in case the patient has hyperemesis gravidarum, as hyperthyroidism is associated with this disorder. The KB test is used to determine transplacental hemorrhage.

The nurse is teaching a class on childbirth. What does the nurse teach about signs of local anesthetic toxicity? Select all that apply. 1 Tinnitus 2 Metallic taste 3 Slurred speech 4 Long stage II labor 5 Increased use of oxytocin

1, 2, 3 The central nervous system can be affected if a local anesthetic agent is injected accidentally into a blood vessel leading to local anesthetic toxicity . Signs include metallic taste, tinnitus, and slurred speech. Longer stage II labor and increased use of oxytocin are side effects of epidural and spinal anesthesia.

A pregnant patient in the first trimester reports spotting of blood with the cervical os closed and mild uterine cramping. What does the nurse need to assess? Select all that apply. 1 Progesterone levels 2 Transvaginal ultrasounds 3 Human chorionic gonadotropin (hCG) measurement 4 Blood pressure 5 Kleihauer-Betke (KB) test reports

1, 2, 3 The spotting of blood with the cervical os closed and mild uterine cramping in the first trimester indicates a threatened miscarriage. Therefore the nurse needs to assess progesterone levels, transvaginal ultrasounds, and measurement of hCG to determine whether the fetus is alive and within the uterus. Blood pressure measurements do not help determine the fetal status. KB assay is prescribed to identify fetal-to-maternal bleeding, usually after a trauma.

The nurse acts as an advocate for the patient during an informed consent. What care must the nurse take while obtaining an informed consent? Select all that apply. 1 Check for the patient's signature. 2 Ensure that the consent is in English. 3 Obtain a family member's signature. 4 Check for the date on the consent form. 5 Check the anesthetic care provider's signature.

1, 4, 5 The nurse must ensure that the consent form has the correct date. The nurse must ensure that the patient has not been compelled to consent for the procedure. The form must carry the signature of the anesthetic care provider, certifying that the patient has received and expresses understanding of the explanation. The consent form must be written or explained in the patient's primary language. The nurse need not obtain a family member's signature on the document. The patient's signature is important.

Stages

1. Engagement 2. Descent 3. Flexion 4. Internal rotation 5. Extension 6. External rotation 7. Expulsion

what are some antepartum interventions with a mom who has a cardiac disorder

1. minimize stress to the heart** 2. treat infections promptly 3. nutrition counseling 4. avoid straining to poo 5. anticoagulant therapy 6. heart surgery

In providing counseling to pregnant women, the nurse can include information on the components of weight gain during pregnancy and the amount of weight that will be lost after the birth. Match the tissues contributing to maternal weight gain at 40 weeks of gestation with the approximate amount of weight gain in kilograms. a.0.5 to 1.8 b.1.8 to 2.3 c.3.2 to 3.9 d.0.9 e.0.9 to 1.1 1. Fetus 2. Placenta 3. Breast tissue 4. Amniotic fluid 5. Blood volume

1. ANS: C DIF: Cognitive Level: Analyze REF: p. 349 TOP: Nursing Process: Assessment MSC: Client Needs: Physiologic Integrity NOT: Other components of tissue that contribute to weight gain include: an increase in uterine tissue (0.9), increased tissue fluid (1.4 to 2.3), and increased fat stores (1.8 to 2.7). Because lactation can help gradually reduce maternal energy stores, it also provides an opportunity to promote breastfeeding. 2. ANS: E DIF: Cognitive Level: Analyze REF: p. 349 TOP: Nursing Process: Assessment MSC: Client Needs: Physiologic Integrity NOT: Other components of tissue that contribute to weight gain include: an increase in uterine tissue (0.9), increased tissue fluid (1.4 to 2.3), and increased fat stores (1.8 to 2.7). Because lactation can help gradually reduce maternal energy stores, it also provides an opportunity to promote breastfeeding. 3. ANS: A DIF: Cognitive Level: Analyze REF: p. 349 TOP: Nursing Process: Assessment MSC: Client Needs: Physiologic Integrity NOT: Other components of tissue that contribute to weight gain include: an increase in uterine tissue (0.9), increased tissue fluid (1.4 to 2.3), and increased fat stores (1.8 to 2.7). Because lactation can help gradually reduce maternal energy stores, it also provides an opportunity to promote breastfeeding. 4. ANS: D DIF: Cognitive Level: Analyze REF: p. 349 TOP: Nursing Process: Assessment MSC: Client Needs: Physiologic Integrity NOT: Other components of tissue that contribute to weight gain include: an increase in uterine tissue (0.9), increased tissue fluid (1.4 to 2.3), and increased fat stores (1.8 to 2.7). Because lactation can help gradually reduce maternal energy stores, it also provides an opportunity to promote breastfeeding. 5. ANS: B DIF: Cognitive Level: Analyze REF: p. 349 TOP: Nursing Process: Assessment MSC: Client Needs: Physiologic Integrity NOT: Other components of tissue that contribute to weight gain include: an increase in uterine tissue (0.9), increased tissue fluid (1.4 to 2.3), and increased fat stores (1.8 to 2.7). Because lactation can help gradually reduce maternal energy stores, it also provides an opportunity to promote breastfeeding.

when does a fetus begin to make their own insulin

10 wks

A new mother asks the nurse when the "soft spot" on her son's head will go away. The nurse's answer is based on the knowledge that the anterior fontanel closes after birth by _____ months.

18

What action does the nurse take to relieve choking in a pregnant patient who is in the third trimester? 1 Administering anesthesia 2 Administering chest thrusts 3 Placing a towel under the hips 4 Positioning the patient onto one side

2 Choking is often relieved in patients by administering abdominal thrusts. However if the patient is in the third trimester of pregnancy, chest thrusts are administered to prevent injury to the uterus. Administering anesthesia or positioning the patient onto one side will not help dislodge the object and relieve choking. The nurse needs to place a towel under the hips to displace the uterus while administering cardiopulmonary resuscitation (CPR).

How does the nurse advise the patient who has given birth to an infant with microcephaly in the past and is now planning for the next child? 1 "There is a higher chance of having a preterm birth." 2 "You should be screened for phenylketonuria (PKU)." 3 "There may be a miscarriage in your second pregnancy." 4 "You must go for genetic counseling before conception."

2 If a patient has given birth to an infant with microcephaly in the past, there is a possibility that the patient has phenylketonuria (PKU). PKU results from a deficiency in the enzyme phenylalanine hydrolase. Preterm birth is a possibility in pregnant women with untreated hypothyroidism. PKU affects brain development and function in the child; it does not cause miscarriage. Genetic counseling is more important for patients who have hereditary disorders, which can be passed on to the child. Test-Taking Tip: Because few things in life are absolute without exceptions, avoid selecting answers that include words such as always, never, all, every, and none. Answers containing these key words are rarely correct.

A woman with severe preeclampsia is being treated with an intravenous infusion of magnesium sulfate. This treatment is considered successful if: 1 blood pressure is reduced to prepregnant baseline. 2 seizures do not occur. 3 deep tendon reflexes become hypotonic. 4 diuresis reduces fluid retention

2 Magnesium sulfate is a central nervous system (CNS) depressant given primarily to prevent seizures . A temporary decrease in blood pressure can occur; however, this is not the purpose of administering this medication. Hypotonia is a sign of an excessive serum level of magnesium. It is critical that calcium gluconate be on hand to counteract the depressant effects of magnesium toxicity. Diuresis is not an expected outcome of magnesium sulfate administration.

Which clinical finding in a pregnant patient will indicate proper fetal brain development? 1 Hemoglobin A1c levels greater than 6 2 Normal maternal thyroxine (T4) levels 3 3% amniotic fluid phosphatidylglycerol 4 Fasting glucose levels less than 95 mg/dL

2 Proper fetal brain development depends on normal maternal T4 levels early in pregnancy. Mild maternal hypothyroidism during the first trimester can cause neuropsychological damage in the infant. Hemoglobin A1c levels greater than 6 indicate long-term elevated glucose levels in the patient. A 3% amniotic fluid phosphatidylglycerol indicates proper lung maturation in the fetus. Fasting glucose levels less than 95 mg/dL indicate proper glycemic control in the pregnant patient.

The nurse is assisting a patient who is prepared to use the paced breathing method. What does the nurse remind the patient to do at the beginning of the breathing pattern? 1 Exhale a deep breath. 2 Take a deep relaxing breath. 3 Take 32 breaths per minute. 4 Take three breaths per minute.

2 The patient must remember that all breathing patterns begin with a deep, relaxing "cleansing breath" to "greet the contraction." The patient must then exhale a deep breath to "blow the contraction away." These deep breaths ensure adequate oxygen for the mother and the baby and signal that a contraction is beginning or has ended. The patient must take three to four breaths per minute when performing slow-paced breathing. As contractions increase in frequency and intensity, the patient takes shallow, fast breaths, about 32 to 40 per minute.

From 4% to 8% of pregnant women have asthma, making it one of the most common preexisting conditions of pregnancy. Severity of symptoms usually peaks: 1 in the first trimester. 2 between 17 to 24 weeks of gestation. 3 during the last 4 weeks of pregnancy. 4 immediately postpartum

2 The period between 17 and 24 weeks of pregnancy is associated with the greatest severity of symptoms. Women often have few symptoms of asthma during the first trimester. During the last 4 weeks of pregnancy symptoms often subside. Often issues have resolved by the time the woman gives birth.

While monitoring the fetal heart rate (FHR), the nurse instructs the patient to change positions and lie in the knee-to-chest position. What is the reason for the nurse to give this instruction to the patient? 1 Late decelerations in the FHR 2 Variable decelerations in the FHR 3 Early decelerations in the FHR 4 Prolonged decelerations in the FHR

2 Variable decelerations in the FHR are usually caused by umbilical cord compression. The knee-to-chest position is useful for relieving cord compression, and thus the nurse should ask the patient to move into this position. Prolonged decelerations in the FHR are not affected by the mother's position. If the nurse finds late decelerations in the FHR, the nurse should ask the mother to lie in the lateral position. Early decelerations in the FHR are a normal finding, and no nursing intervention is required.

The nurse is assisting a pregnant patient in labor. What instructions should the nurse give to the patient to promote comfort? Select all that apply. 1 "You should cough frequently." 2 "Breathe with your mouth open." 3 "Lie down in the lateral position." 4 "Lie in the supine position in bed." 5 "Lie in the semi-Fowler position."

2, 3, 5 The nurse helps the pregnant patient during labor. This includes teaching the patient relaxation techniques. The nurse teaches the patient to keep the mouth open during exhalation to allow air to easily leave the lungs. Placing the patient in a semi-Fowler or lateral position is helpful during labor. Therefore the nurse should instruct the patient to maintain a lateral or semi-Fowler position with a lateral tilt. Asking the patient to cough frequently would increase intraabdominal pressure of the patient and would make the patient uncomfortable. Having the patient lie down in a supine position during labor may cause orthostatic hypotension. Therefore the nurse should instruct the patient to lie down in a position other than supine.

what should PKU levels be

2-6

The BMI for a woman who is 51 kg before pregnancy and 1.57 m tall is _________.

20.7 A commonly used method of evaluating the appropriateness of weight for height is the BMI, which is calculated by the following formula. BMI = Weight in kg divided by the height in meters squared BMI = 51 kg ¸ (1.57 m)2 51 ¸ 2.47 = 20.69

The nurse is preparing to discharge a 30-year-old woman who has experienced a miscarriage at 10 weeks of gestation. Which statement by the woman indicates a correct understanding of the discharge instructions? 1 "I will not experience mood swings since I was only at 10 weeks of gestation." 2 "I will avoid sexual intercourse for 6 weeks and pregnancy for 6 months." 3 "I should eat foods that are high in iron and protein to help my body heal." 4 "I should expect the bleeding to be heavy and bright red for at least 1 week."

3 A woman who has experienced a miscarriage should be advised to eat foods that are high in iron and protein to help replenish her body after the loss. After a miscarriage, a woman may experience mood swings and depression from the reduction of hormones and the grieving process. Sexual intercourse should be avoided for 2 weeks or until the bleeding has stopped and should avoid pregnancy for 2 months. The woman should not experience bright red, heavy, profuse bleeding; this should be reported to the health care provider.

What major side effect does the nurse expect if a patient in labor is administered diazepam (Valium)? 1 Severe nausea and vomiting in the mother 2 Neonatal central nervous system depression 3 Disrupted temperature control in the newborn 4 Magnified pain if administered without analgesic

3 Diazepam (Valium) disrupts thermoregulation in the newborn. Thus the newborn is less able to maintain body temperature. Benzodiazepines, when given with an opioid analgesic, seem to enhance pain relief and reduce nausea and vomiting. Pain is magnified if a barbiturate is given without an analgesic to a patient who is experiencing pain. This is because the normal coping mechanism in the patient may be blunted. Barbiturates should be avoided if birth is anticipated within 12 to 24 hours because it has the potential to cause neonatal central nervous system depression.

The nurse assesses the fetal heart rate (FHR) of a pregnant patient and finds minimal FHR variability. The nurse reassesses the patient 30 minutes later and finds moderate variability. What should the nurse infer? 1 No acceleration 2 Late deceleration 3 Baseline heart rate is 150 beats/min 4 Baseline heart rate is 180 beats/min

3 If the nurse notes minimal FHR variability, the nurse should reassess the heart rate to determine a pattern. If in 30 minutes the nurse notices moderate variability, the fetus may be in a sleep state. The nurse would further confirm after half an hour and report it as moderate variability, where the heart rate baseline is confirmed as normal (110-160 beats/min). Heart rate variability is a characteristic of the baseline FHR and does not include accelerations or decelerations of the FHR. A fetal baseline heart rate of 180 beats/min is considered severe variability.

While assessing a pregnant patient who is in labor, the nurse observes W-shaped waves on the fetal heart rate (FHR) monitor. What would the nurse infer from this observation? 1 Placental abruption 2 Dilated cervical layers 3 Umbilical cord compression 4 Elevated uterine contractions

3 W-shaped waves in the FHR monitor are indicative of variable decelerations in the FHR. Variable decelerations are seen when the umbilical cord is compressed at the time of labor. Placental abruption and dilated cervical layers do not cause variable decelerations but may cause late decelerations. Similarly, increased rate of uterine contractions may also cause late decelerations in FHR.

A patient asks the nurse about the use of transcutaneous electrical nerve stimulation (TENS). What does the nurse teach about TENS? 1 It involves the use of one pair of electrodes. 2 It is kept at low intensity during contractions. 3 It releases continuous low-intensity impulses. 4 It is useful for pain in the second stage of labor.

3 When TENS is applied for pain relief, the electrodes provide continuous low-intensity electrical impulses or stimuli from a battery-operated device. TENS is most useful for lower back pain during the early first stage of labor. TENS involves the placing of two pairs of flat electrodes on either side of the woman's thoracic and sacral spine. During a contraction, the patient increases the stimulation from low to high intensity by turning the control knobs on the device.

During a prenatal assessment a patient asks the nurse about the disadvantages of spinal anesthesia. What does the nurse teach the patient about the potential effect of spinal anesthesia? 1 It reduces maternal consciousness. 2 It increases maternal muscular tension. 3 It increases probability of operative birth. 4 It increases the possibility of fetal hypoxia.

3 When a spinal anesthetic is given, the need for episiotomy, forceps-assisted birth, or vacuum-assisted birth tends to increase because voluntary expulsive efforts are reduced or eliminated. Maternal consciousness is maintained. Fetal hypoxia is absent as maternal blood pressure is maintained within a normal range. There is no muscular tension; excellent muscular relaxation is achieved.

The nurse is assessing a pregnant patient during labor and reports the normal duration of the contraction period as 2 minutes, 15 seconds in a span of 10 minutes. What would be the number of contractions observed in this span of 10 minutes? Record your answer using a whole number._______

3 n a pregnant patient the normal range of uterine contractions (UCs) during labor are noted to be 2 to 5 in every 10 minutes. Each one contraction lasts from 45 to 80 seconds. Therefore, when the nurse reports the contraction period as 2 minutes, 15 seconds (135 seconds) in 10 minutes of time, the nurse should have observed 135 ÷ 45 = 3 contractions.

The nurse is assessing a pregnant patient through a tocotransducer placed externally and a spiral electrode placed internally. What information would the nurse obtain by this arrangement? Select all that apply. 1 Lactate levels in the fetal blood 2 Strength of uterine contractions 3 Duration of uterine contractions 4 Frequency of uterine contractions 5 Accelerations of fetal heart rate

3, 4, 5 A tocotransducer is an external device that is used for assessment of uterine activity (UA). This instrument would report duration and frequency of the uterine contractions (UCs). The spiral electrode can monitor accelerations of the fetal heart rate. These systems do not report the intensity of UCs. Strength of UCs can be assessed using an intrauterine pressure catheter (IUPC). Neither a tocotransducer nor a spiral electrode is used to determine the lactate level; it is obtained by the fetal scalp sampling method.

The nurse is teaching a woman with gestational diabetes the technique to inject insulin. What should the nurse include in the teaching session? Select all that apply. 1 Aspirate before injecting. 2 Clean injection site with alcohol. 3 Insert the needle at a 45- to 90-degree angle. 4 Inject insulin slowly. 5 After injection, cover site with sterile gauze.

3, 4, 5 Insulin should be injected with the short needle inserted at a 45- to 90-degree angle, depending on fatty tissue. Insulin is injected slowly to allow tissue expansion and minimize pressure, which can cause insulin leakage. After injection, the site should be covered with sterile gauze. Gentle pressure should be applied to prevent bleeding. Aspirating when injecting into subcutaneous tissue is not necessary. The injection site should be clean, but using alcohol is not necessary.

Which medication is ideal for the treatment of systemic lupus erythematosus (SLE) in a pregnant patient? 1 Aspirin (Ecotrin) 2 Azathioprine (Imuran) 3 Prednisone (Deltasone) 4 Hydroxychloroquine (Plaquenil)

4 Hydroxychloroquine (Plaquenil) reduces SLE disease activity in a pregnant patient without any adverse effects on the fetus. Aspirin (Ecotrin) is not recommended during pregnancy, because it has an increased risk for premature closure of the fetal ductus arteriosus. Azathioprine (Imuran) is discontinued before conception, because it is fetotoxic. Prednisone (Deltasone) is prescribed to treat SLE during pregnancy, but it increases the risk for bone demineralization, gestational diabetes, preeclampsia, premature rupture of membranes (PROM), and intrauterine growth restriction (IUGR).

After reviewing the umbilical cord acid-base report, the nurse confirms that the fetus has respiratory acidosis. Which reading is consistent with the nurse's conclusion? 1 A base deficit value ≥12 mmol/L 2 Blood glucose levels = 120 mg/dL 3 Arterial pH >7.20 4 Partial pressure carbon dioxide >55 mm Hg

4 If pH 2 >55 mm Hg (elevated), and base deficit value respiratory acidosis. In this case, the partial pressure carbon dioxide >55 mm Hg is indicative of respiratory acidosis. A pH >7.20 and base deficit value ≥12 mmol/L are all considered normal. Blood glucose level is not a part of this acid-base report.

Which fetal risk is associated with an ectopic pregnancy? 1 Miscarriage 2 Fetal anemia 3 Preterm birth 4 Fetal deformity

4 In an ectopic pregnancy, the risk for fetal deformity is high because of the pressure deformities caused by oligohydramnios. There may be facial or cranial asymmetry, various joint deformities, limb deficiency, and central nervous system (CNS) anomalies. Miscarriage is not likely to happen in an ectopic pregnancy. Instead, the patient is at risk for pregnancy-related death resulting from ectopic rupture. Fetal anemia is a risk associated with placenta previa. Preterm birth is not possible because the pregnancy is dissolved when it is diagnosed or a surgery is performed to remove the fetus.

After observing the reports of the umbilical cord acid-base determination test, the nurse informs the patient that the newborn's condition is normal. Which value indicates the normal condition of the newborn? 1 Umbilical artery: pH, 7.1; Pco2, 50 mm Hg; Po2, 20 mm Hg 2 Umbilical artery: pH, 7.3; Pco2, 40 mm Hg; Po2, 10 mm Hg 3 Umbilical artery: pH, 7.4; Pco2, 52 mm Hg; Po2, 27 mm Hg 4 Umbilical artery: pH, 7.3; Pco2, 45 mm Hg; Po2, 25 mm Hg

4 In the umbilical cord acid-base stimulation method, arterial values indicate the condition of the newborn. Arterial blood pH of 7.2 to 7.3, carbon dioxide pressure (Pco2) value of 45 to 55 mm Hg, and oxygen pressure (Po2) value of 15 to 25 mm Hg approximately indicates the normal fetal condition. Therefore pH of 7.3, Pco2 of 45 mm Hg, and Po2 of 25 mm Hg represent the normal fetal condition. Arterial blood pH of 7.1, Pco2 of 50 mm Hg, and Po2 of 20 mm Hg indicate that the fetus may have respiratory acidosis. Arterial blood pH of 7.4 is indicative of fetal alkalosis.

The charge nurse instructed a group of student nurses about the monitoring of uterine activity (UA) during labor. Which statement by the student nurse is accurate regarding the calculation of Montevideo units? "They can be calculated: 1 Using a spiral electrode monitoring device." 2 Using a tocotransducer monitoring system." 3 Using an ultrasound transducer machine." 4 With an intrauterine pressure catheter (IUPC)."

4 Montevideo units can only be calculated using the internal monitoring of UA. An intrauterine pressure catheter (IUPC) monitors UA internally. Therefore Montevideo units can only be calculated using the IUPC. Spiral electrode monitoring is used for assessing the fetal heart rate (FHR), not UA internally. The tocotransducer monitoring system is used to monitor the UA externally. An ultrasound transducer is also used to monitor the FHR externally.

When caring for a patient in the first phase of labor, the nurse observes that the patient is experiencing visceral pain. In which area does visceral pain occur? 1 Abdominal wall and thighs 2 Gluteal area and iliac crests 3 Lumbosacral area of the back 4 Lower portion of the abdomen

4 Visceral pain in the first stage of labor occurs in the lower portion of the abdomen. Visceral pain is a result of distention of the lower uterine segment and stretching of cervical tissue as it effaces and dilates. Pressure and traction on uterine tubes, ovaries, ligaments, nerves, and uterine ischemia also cause visceral pain. Pain that originates in the uterus radiates to the gluteal area, iliac crests, abdominal wall, thighs, lumbosacral area of the back, and lower back. This pain is called referred pain.

While assessing a pregnant patient using a fetoscope, the nurse also palpates the abdomen of the patient. What is the purpose of palpating the abdomen of the patient? 1 Detection of fetal heart rate deceleration 2 Evaluation of the severity of the pain caused by active labor 3 Assessment of pain from pressure applied by the fetoscope 4 Assessment of changes in fetal heart rate during and after contraction

4 While assessing the fetal heart rate (FHR) with a fetoscope, the nurse palpates the abdomen of the fetus to evaluate uterine contractions (UCs). This is done to detect any changes in the FHR during and after UCs. FHR decelerations are not identified by palpating the abdomen. It is assessed using the electronic fetal monitoring system. Pain perception is a subjective assessment. Moreover, the pressure from the fetoscope is very minimal and does not cause pain.

At 1 minute after birth, the nurse assesses the newborn to assign an Apgar score. The apical heart rate is 110 bpm, and the infant is crying vigorously with the limbs flexed. The infant's trunk is pink, but the hands and feet are blue. What is the correct Apgar score for this infant?

9 [1 point is deducted from the total score of 10 for the infant's blue hands and feet.]

Before the physician performs an external version, the nurse should expect an order for a: a. Tocolytic drug. c. Local anesthetic. b. Contraction stress test (CST). d. Foley catheter

A A tocolytic drug will relax the uterus before and during version, thus making manipulation easier. CST is used to determine the fetal response to stress. A local anesthetic is not used with external version. The bladder should be emptied; however, catheterization is not necessary.

Which collection of risk factors most likely would result in damaging lacerations including episiotomies ?

A first-time mother with reddish hair whose rapid labor was overseen by an obstetrician

Which nutrient's recommended dietary allowance (RDA) is higher during lactation than during pregnancy? a.Energy (kcal) b.Iron c.Vitamin A d.Folic acid

ANS: A Nutrient needs for energy—protein, calcium, iodine, zinc, B vitamins, and vitamin C—remain higher during lactation than during pregnancy. The need for iron is not higher during lactation than during pregnancy. A lactating woman does not have a greater requirement for vitamin A than a nonpregnant woman. Folic acid requirements are the highest during the first trimester of pregnancy.

As relates to fetal positioning during labor, nurses should be aware that:

Birth is imminent when the presenting part is at +4 to +5 cm below the spine

Which occurrence is associated with cervical dilation and effacement?

Bloody show

When assessing a woman in labor, the nurse is aware that the relationship of the fetal body parts to one another is called fetal: a. Lie. c. Attitude. b. Presentation. d. Position.

C Attitude is the relation of the fetal body parts to one another. Lie is the relation of the long axis (spine) of the fetus to the long axis (spine) of the mother. Presentation refers to the part of the fetus that enters the pelvic inlet first and leads through the birth canal during labor at term. Position is the relation of the presenting part to the four quadrants of the mother's pelvis.

The nurse teaches a pregnant woman about the characteristics of true labor contractions. The nurse evaluates the woman's understanding of the instructions when she states, "True labor contractions will: a. Subside when I walk around." b. Cause discomfort over the top of my uterus." c. Continue and get stronger even if I relax and take a shower." d. Remain irregular but become stronger."

C True labor contractions occur regularly, becoming stronger, lasting longer, and occurring closer together. They may become intense during walking and continue despite comfort measures. Typically true labor contractions are felt in the lower back, radiating to the lower portion of the abdomen. During false labor, contractions tend to be irregular and felt in the abdomen above the navel. Typically the contractions often stop with walking or a change of position.

When managing the care of a woman in the second stage of labor, the nurse uses various measures to enhance the progress of fetal descent. These measures include:

Encouraging the woman to try various upright positions, including squatting and standing.

Question: Place the cardinal movements of labor in the order in which they occur. Engagement, then descent Flexion Internal rotation Extension External rotation Expulsion

Engagement, then descent Flexion Internal rotation Extension External rotation Expulsion

Which action is correct when palpation is used to assess the characteristics and pattern of uterine contractions?

Evaluate the intensity by pressing the fingertips into the uterine fundus.

A pregnant woman is in her third trimester. She asks the nurse to explain how she can tell true labor from false labor. The nurse would explain that "true" labor contractions:

Increase with activity such as ambulation.

A woman who is 39 weeks pregnant expresses fear about her impending labor and how she will manage. The nurse's best response is:

It's normal to be anxious about labor. Let's discuss what makes you afraid."

Signs that precede labor include

Lightening. Bloody show. Rupture of membranes.

Which statement is the best rationale for assessing maternal vital signs between contractions?

Maternal circulating blood volume increases temporarily during contractions.

When implementing care, the nurse would anticipate that a woman from which country would have the father of the baby in attendance?

Mexico

The slight overlapping of cranial bones or shaping of the fetal head during labor is called:

Molding.

What is an expected characteristic of amniotic fluid?

Pale, straw color with small white particles

Which basic type of pelvis includes the correct description and percentage of occurrence in women?

Platypelloid: flattened, wide, shallow; 3%

The factors that affect the process of labor and birth, known commonly as the five Ps, include all except:

Pressure.

The nurse expects to administer an oxytocic, Pitocin, Methergine to a woman after expulsion of her placenta to:

Stimulate uterine contraction.

The nurse recognizes that a woman is in true labor when she states:

The contractions in my uterus are getting stronger and closer together.

With regard to factors that affect how the fetus moves through the birth canal, nurses should be aware that:

The normal attitude of the fetus is called general flexion.

With regard to a woman's intake and output during labor, nurses should be aware that:

The tradition of restricting the laboring woman to clear liquids and ice chips is being challenged because regional anesthesia is used more often than general anesthesia.

what is PKU (phenylkentonuria)

a condition where your body cant metabolize the amini acid phenylanine which is found in foods with protein and it will build up and cause problems

what is the most common disorder in pregnancy

anemia

Which test is performed to determine if membranes are ruptured? a) Urine analysis b) Fern test c) Leopold maneuvers d) AROM

b) Fern test

12. To prevent gastrointestinal upset, clients should be instructed to take iron supplements: a. On a full stomach. b. At bedtime. c. After eating a meal. d. With milk.

b. At bedtime.

what foods contain iron but block absorption

bran, tea, coffee, milk

what medications should be avoided in a pregnant mom with epilepsy

carbamazepine and valproate

For the labor nurse, care of the expectant mother begins with any or all of these situations except: a) The onset of progressive, regular contractions b) The bloody, or pink, show c) The spontaneous rupture of membranes d) formulation of the woman's plan of care for labor

d) formulation of the woman's plan of care for labor

In a variation of rooming-in, called couplet care, the mother and infant share a room and the mother shares the care of the infant with: a) The father of the infant b) Her mother (the infant's grandmother) c) Her eldest daughter (the infant's sister) d) the nurse

d) the nurse

What are the risk factors of gestational diabetes

family hx of diabetes obesity previous stillbirth or macrosomic fetus HTN glucosuria >25 yo

what are some s/s of sickle cell crisis

fever pain in the joints, abdomen, extremities

when referring to glucose and insulin, which one or both can cross the placenta?

glucose can, insulin cannot

with pregestational diabetes during pregnancy moms insulin requirements will fluctuate. When will they increase and decrease

increase *3-7 wks *2nd and 3rd trimester decrease *7-15 wks

When planning care for a laboring woman whose membranes have ruptured, the nurse recognizes that:

the woman's risk for intrauterine infection has increased. [When the membranes rupture, microorganisms from the vagina can ascend into the amniotic sac and cause chorioamnionitis and placentitis.]

what should be given with iron to increase absorption

vitamin C

when anemia is combined with other complications what can result

chf

A woman is experiencing back labor and complains of constant, intense pain in her lower back. An effective relief measure is to use: A) counterpressure against the sacrum. B) pant-blow (breaths and puffs) breathing techniques. C) effleurage. D) biofeedback.

*A) counterpressure against the sacrum.* Rationale: Counterpressure is steady pressure applied by a support person to the sacral area with the fist or heel of the hand. This technique helps the woman cope with the sensations of internal pressure and pain in the lower back. Pant-blow breathing techniques are usually helpful during contractions per the gate-control theory. Effleurage is light stroking, usually of the abdomen, in rhythm with breathing during contractions. It is used as a distraction from contraction pain; however, it is unlikely to be effective for back labor. Biofeedback-assisted relaxation techniques are not always successful in reducing labor pain. Using this technique effectively requires strong caregiver support.

A woman in latent labor who is positive for opiates on the urine drug screen is complaining of severe pain. Maternal vital signs are stable, and the fetal heart monitor displays a reassuring pattern. The nurse's MOST appropriate analgesic for pain control is: A) fentanyl (Sublimaze). B) promethazine (Phenergan). C) butorphanol tartrate (Stadol). D) nalbuphine (Nubain).

*A) fentanyl (Sublimaze).* Rationale: Fentanyl is a commonly used opioid agonist analgesic for women in labor. It is fast and short acting. This patient may require higher than normal doses to achieve pain relief due to her opiate use. Phenergan is not an analgesic. Phenergan is an ataractic (tranquilizer) that may be used to augment the desirable effects of the opioid analgesics but has few of those drugs' undesirable effects. *Stadol and Nubain are opioid agonist-antagonist analgesics. Their use may precipitate withdrawals in a patient with a history of opiate use.*

Nurses should be aware of the difference experience can make in labor pain, such as: A) sensory pain for nulliparous women often is greater than for multiparous women during early labor. B) affective pain for nulliparous women usually is less than for multiparous women throughout the first stage of labor. C) women with a history of substance abuse experience more pain during labor. D) multiparous women have more fatigue from labor and therefore experience more pain.

*A) sensory pain for nulliparous women often is greater than for multiparous women during early labor.* Rationale: Sensory pain is greater for nulliparous women because their reproductive tract structures are less supple. Affective pain is greater for nulliparous women during the first stage but decreases for both nulliparous and multiparous during the second stage. Women with a history of substance abuse experience the same amount of pain as those without such a history. Nulliparous women have longer labors and therefore experience more fatigue.

After delivering a healthy baby boy with epidural anesthesia, a woman on the postpartum unit complains of a severe headache. The nurse should anticipate which actions in the patient's plan of care? (Select all that apply.) A) Keeping the head of bed elevated at all times B) Administration of oral analgesics C) Avoid caffeine D) Assisting with a blood patch procedure E) Frequent monitoring of vital signs

*B) Administration of oral analgesics* *D) Assisting with a blood patch procedure* *E) Frequent monitoring of vital signs* Rationale: The nurse should suspect the patient is suffering from a postdural puncture headache (PDPH). Characteristically, assuming an upright position triggers or intensifies the headache, whereas assuming a supine position achieves relief (Hawkins and Bucklin, 2012). Conservative management for a PDPH includes administration of oral analgesics and methylxanthines (e.g., caffeine or theophylline). Methylxanthines cause constriction of cerebral blood vessels and may provide symptomatic relief. An autologous epidural blood patch is the most rapid, reliable, and beneficial relief measure for PDPH. Close monitoring of vital signs is essential.

With regard to systemic analgesics administered during labor, nurses should be aware that: A) systemic analgesics cross the maternal blood-brain barrier as easily as they do the fetal blood-brain barrier. B) effects on the fetus and newborn can include decreased alertness and delayed sucking. C) IM administration is preferred over IV administration. D) IV patient-controlled analgesia (PCA) results in increased use of an analgesic.

*B) effects on the fetus and newborn can include decreased alertness and delayed sucking.* Rationale: Systemic analgesics cross the fetal blood-brain barrier more readily than the maternal blood-brain barrier. Effects depend on the specific drug given, the dosage, and the timing. IV administration is preferred over IM administration because the drug acts faster and more predictably. PCAs result in decreased use of an analgesic.

A laboring woman becomes anxious during the transition phase of the first stage of labor and develops a rapid and deep respiratory pattern. She complains of feeling dizzy and light-headed. The nurse's immediate response would be to: A) encourage the woman to breathe more slowly. B) help the woman breathe into a paper bag. C) turn the woman on her side. D) administer a sedative.

*B) help the woman breathe into a paper bag.* Rationale: Just telling her to breathe more slowly does not ensure a change in respirations. The woman is exhibiting signs of hyperventilation. This leads to a decreased carbon dioxide level and respiratory alkalosis. Rebreathing her air would increase the carbon dioxide level. Turning her on her side will not solve this problem. Administration of a sedative could lead to neonatal depression since this woman, being in the transition phase, is near the birth process. The side-lying position would be appropriate for supine hypotension.

After change of shift report, the nurse assumes care of a multiparous patient in labor. The woman is complaining of pain that radiates to her abdominal wall, lower back, buttocks, and down her thighs. Before implementing a plan of care, the nurse should understand that this type of pain is: A) visceral. B) referred. C) somatic. D) afterpain.

*B) referred.* Rationale: *Visceral pain* is that which predominates the first stage of labor. This pain originates from cervical changes, distention of the lower uterine segment, and uterine ischemia. Visceral pain is located over the lower portion of the abdomen. As labor progresses the woman often experiences *referred pain*. This occurs when pain that originates in the uterus radiates to the abdominal wall, the lumbosacral area of the back, the gluteal area, and thighs. The woman usually has pain only during a contraction and is free from pain between contractions. *Somatic pain* is described as intense, sharp, burning, and well localized. This results from stretching of the perineal tissues and the pelvic floor. This occurs during the second stage of labor. Pain experienced during the third stage of labor or afterward during the early postpartum period is uterine. This pain is very similar to that experienced in the first stage of labor.

A woman is in the second stage of labor and has a spinal block in place for pain management. The nurse obtains the woman's blood pressure and notes that it is 20% lower than the baseline level. Which action should the nurse take? A) Encourage her to empty her bladder. B) Decrease her intravenous (IV) rate to a keep vein-open rate. C) Turn the woman to the left lateral position or place a pillow under her hip. D) No action is necessary since a decrease in the woman's blood pressure is expected.

*C) Turn the woman to the left lateral position or place a pillow under her hip.* Rationale: Encouraging the woman to empty her bladder will not help the hypotensive state and may cause her to faint if she ambulates to the bathroom. The IV rate should be kept at the current rate or increased to maintain the appropriate perfusion. Turning the woman to her left side is the best action to take in this situation since this will increase placental perfusion to the infant while waiting for the doctor's or nurse midwife's instruction. Hypotension indicated by a 20% drop from preblock level is an emergency situation and action must be taken.

With regard to what might be called the tactile approaches to comfort management, nurses should be aware that: A) either hot or cold applications may provide relief, but they should never be used together in the same treatment. B) acupuncture can be performed by a skilled nurse with just a little training. C) hand and foot massage may be especially relaxing in advanced labor when a woman's tolerance for touch is limited. D) therapeutic touch (TT) uses handheld electronic stimulators that produce sympathetic vibrations.

*C) hand and foot massage may be especially relaxing in advanced labor when a woman's tolerance for touch is limited.* Rationale: Heat and cold may be applied in an alternating fashion for greater effect. Unlike acupressure, acupuncture, which involves the insertion of thin needles, should be done only by a certified therapist. The woman and her partner should experiment with massage before labor to see what might work best. Therapeutic touch is a laying-on of hands technique that claims to redirect energy fields in the body.

When monitoring a woman in labor who has just received spinal analgesia, the nurse should report which assessment findings to the health care provider? (Select all that apply.) A) Maternal blood pressure of 108/79 B) Maternal heart rate of 98 C) Respiratory rate of 14 breaths/min D) Fetal heart rate of 100 beats/min E) Minimal variability on a fetal heart monitor

*D) Fetal heart rate of 100 beats/min* *E) Minimal variability on a fetal heart monitor* Rationale: After induction of the anesthetic, maternal blood pressure, pulse, and respirations and fetal heart rate and pattern must be checked and documented every 5 to 10 minutes. If signs of serious maternal hypotension (e.g., the systolic blood pressure drops to 100 mm Hg or less or the blood pressure falls 20% or more below the baseline) or fetal distress (e.g., bradycardia, minimal or absent variability, late decelerations) develop, emergency care must be given.

A pregnant patient reports abdominal pain in the right lower quadrant, along with nausea and vomiting. The patient's urinalysis report shows an absence of any urinary tract infection in the patient. A chest x-ray also rules out lower-lobe pneumonia. Which condition does the nurse suspect in the patient? 1 Appendicitis 2 Cholelithiasis 3 Placenta previa 4 Uterine rupture

1 Abdominal pain in the right lower quadrant, accompanied by nausea and vomiting, indicates appendicitis in a pregnant patient. Cholelithiasis is characterized by right upper quadrant pain. Placenta previa is a condition wherein the placenta is implanted in the lower uterine segment covering the cervix, which causes bleeding when the cervix dilates. Uterine rupture is seen in a pregnant patient as a result of trauma, which may cause fetal death.

On review of a fetal monitor tracing, the nurse notes that for several contractions the fetal heart rate decelerates as a contraction begins and returns to baseline just before it ends. The nurse should: 1 describe the finding in the nurse's notes. 2 reposition the woman onto her side. 3 call the physician for instructions. 4 administer oxygen at 8 to 10 L/min with a tight face mask.

1 An early deceleration pattern from head compression is described. No action other than documentation of the finding is required because this is an expected reaction to compression of the fetal head as it passes through the cervix. Repositioning the woman onto her side would be implemented when non-reassuring or ominous changes are noted. Calling the physician would be implemented when non-reassuring or ominous changes are noted. Administering oxygen would be implemented when non-reassuring or ominous changes are noted.

A woman at 37 weeks of gestation is admitted with a placental abruption after a motor vehicle accident. Which assessment data are most indicative of her condition worsening? 1 Pulse (P) 112, respiration (R) 32, blood pressure (BP) 108/60; fetal heart rate (FHR) 166--178 2 P 98, R 22, BP 110/74; FHR 150--162 3 P 88, R 20, BP 114/70; FHR 140--158 4 P 80, R 18, BP 120/78; FHR 138--150

1 Bleeding is the most dangerous problem, which impacts the mother's well-being as well as that of her fetus. The decreasing blood volume would cause increases in pulse and respirations and a decrease in blood pressure. The fetus often responds to decreased oxygenation as a result of bleeding, causing a decrease in perfusion. This causes the fetus' heart rate to increase above the normal range of 120--160 beats per minute. The other options have measurements that are in the "normal" range and would not reflect a deterioration of the patient's physical status. Test-Taking Tip: Do not worry if you select the same numbered answer repeatedly, because there usually is no pattern to the answers.

A woman is experiencing back labor and complains of constant, intense pain in her lower back. An effective relief measure is to use: 1 counterpressure against the sacrum. 2 pant-blow (breaths and puffs) breathing techniques. 3 effleurage. 4 biofeedback

1 Counterpressure is steady pressure applied by a support person to the sacral area with the fist or heel of the hand. This technique helps the woman cope with the sensations of internal pressure and pain in the lower back. Pant-blow breathing techniques are usually helpful during contractions per the gate-control theory. Effleurage is light stroking, usually of the abdomen, in rhythm with breathing during contractions. It is used as a distraction from contraction pain; however, it is unlikely to be effective for back labor. Biofeedback-assisted relaxation techniques are not always successful in reducing labor pain. Using this technique effectively requires strong caregiver support.

A woman in latent labor who is positive for opiates on the urine drug screen is complaining of severe pain. Maternal vital signs are stable, and the fetal heart monitor displays a reassuring pattern. The nurse's most appropriate analgesic for pain control is: 1 fentanyl (Sublimaze). 2 promethazine (Phenergan). 3 butorphanol tartrate (Stadol). 4 nalbuphine (Nubain)

1 Fentanyl is a commonly used opioid agonist analgesic for women in labor. It is fast and short acting. This patient may require higher than normal doses to achieve pain relief due to her opiate use. Phenergan is not an analgesic. Phenergan is an ataractic (tranquilizer) that may be used to augment the desirable effects of the opioid analgesics but has few of those drugs' undesirable effects. Stadol is an opioid agonist-antagonist analgesic. Its use may precipitate withdrawals in a patient with a history of opiate use. Nubain is an opioid agonist-antagonist analgesic. Its use may precipitate withdrawals in a patient with a history of opiate use.

The prenatal medical record for a pregnant patient with diabetes states that amniotic fluid phosphatidylglycerol is greater than 3%. What does this indicate? 1 The fetal lung maturation is normal. 2 The mother may develop hydramnios. 3 There is a chance of fetal macrosomia. 4 The mother is at risk for hypoglycemia

1 Fetal lung maturation is predicted by the presence by amniotic fluid phosphatidylglycerol. If it is greater than 3%, the fetal lung maturation is normal. The risk for hypoglycemia can be confirmed by evaluating blood glucose levels. The risk for fetal macrosomia is increased if there is poor glycemic control in pregnancy. If hyperglycemia occurs, the patient is at risk for hydramnios. Test-Taking Tip: Survey the test before you start answering the questions. Plan how to complete the examination in the time allowed. Read the directions carefully and answer the questions you know for sure first.

What kind of anesthesia does the nurse expect the primary health care provider to prescribe to a patient who is to have an emergency cesarean birth due to fetal distress? 1 General anesthesia 2 Pudendal nerve block 3 Nitrous oxide with oxygen 4 Local infiltration anesthesia

1 General anesthesia may be necessary if indications necessitate rapid birth (vaginal or emergent cesarean), when there is a pressing need for time and/or primary health care providers to perform a block. Pudendal nerve block is administered late in the second stage of labor. It may be required if an episiotomy is to be performed or if forceps or a vacuum extractor is to be used to facilitate birth. Nitrous oxide mixed with oxygen can be inhaled in 50% or less concentration to provide analgesia during the first and second stages of labor. Local infiltration anesthesia may be used when an episiotomy is to be performed or when lacerations must be sutured after birth in a woman who does not have regional anesthesia. STUDY TIP: Identify your problem areas that need attention. Do not waste time on restudying information you know.

A woman with severe preeclampsia has been receiving magnesium sulfate by intravenous infusion for 8 hours. The nurse assesses the woman and documents the following findings: temperature 37.1° C, pulse rate 96 beats/min, respiratory rate 24 breaths/min, blood pressure 155/112 mm Hg, 3+ deep tendon reflexes, and no ankle clonus. The nurse calls the physician, anticipating an order for: 1 hydralazine. 2 magnesium sulfate bolus. 3 diazepam.

1 Hydralazine is an antihypertensive commonly used to treat hypertension in severe preeclampsia. An additional bolus of magnesium sulfate may be ordered for increasing signs of central nervous system irritability related to severe preeclampsia (e.g., clonus) or if eclampsia develops. Diazepam sometimes is used to stop or shorten eclamptic seizures. Calcium gluconate is used as the antidote for magnesium sulfate toxicity. The patient is not currently displaying any signs or symptoms of magnesium toxicity.

Which action does the nurse take to determine whether the carbohydrate intake is inadequate in a pregnant patient with diabetes? 1 Monitor for urine ketones. 2 Evaluate the nonstress test results. 3 Determine the degree of glycosuria. 4 Schedule a baseline fetal sonogram.

1 If a patient with diabetes does not take in enough carbohydrates, the body resorts to breaking down fats for energy. The by-product of fat metabolism is ketones. Therefore the nurse monitors the urine for ketones. The amount of ketones in the urine helps detect inadequate carbohydrate intake. Nonstress tests will help assess the well-being of the fetus. Glycosuria does not accurately reflect the blood glucose levels because of a lowered renal threshold for glucose during pregnancy. The nurse obtains a baseline sonogram to assess gestational age in the first trimester. Test-Taking Tip: Relax during the last hour before an examination. Your brain needs some recovery time to function effectively.

A patient with gestational hypertension is prescribed labetalol hydrochloride (Normodyne) therapy, which is continued after giving birth. What does the nurse instruct the patient about breastfeeding? 1 "You may breastfeed the infant if you desire." 2 "Breastfeeding may cause convulsions in the infant." 3 "Breastfeed only once a day and use infant formulas." 4 "There may be high levels of the drug in the breast milk."

1 Labetalol hydrochloride (Normodyne) has a low concentration in breast milk, so the patient can breastfeed the infant. Breastfeeding is safe and will not cause convulsions or any side effects in the infant. Infant formulas are used only if the mother is unable to breastfeed the infant or if the mother does not desire to breastfeed.

After being rehydrated in the emergency department, a 24 year-old primipara in her 18th week of pregnancy is at home and is to rest at home for the next two days and take in small but frequent fluids and food as possible. Discharge teaching at the hospital by the nurse has been effective if the patient makes which statement? 1 "I'm going to eat five to six small servings per day, which contain such foods and fluids as tea, crackers, or a few bites of baked potato." 2 "A strip of bacon and a fried egg will really taste good as long as I eat them slowly." 3 "As long as I eat small amounts and allow enough time for digestion, I can eat almost anything, like barbequed chicken or spaghetti." 4 "I'm going to stay only on clear fluids for the next 24 hours and then add dairy products like eggs and milk."

1 Once the vomiting has stopped, feedings are started in small amounts at frequent intervals. In the beginning, limited amounts of oral fluids and bland foods such as crackers, toast, or baked chicken are offered. Clear fluids alone do not contain enough calories and contain no protein. Most women are able to take nourishment by mouth after several days of treatment. They should be encouraged to eat small, frequent meals and foods that sound appealing (e.g., nongreasy, dry, sweet, and salty foods). Test-Taking Tip: Many times the correct answer is the longest alternative given, but do not count on it. NCLEX item writers (those who write the questions) are also aware of this and attempt to avoid offering you such "helpful hints."

Which condition in a pregnant patient with severe preeclampsia is an indication for administering magnesium sulfate? 1 Seizure activity 2 Renal dysfunction 3 Pulmonary edema 4 Low blood pressure (BP)

1 Severe preeclampsia may cause seizure activity or eclampsia in the patient, which is treated with magnesium sulfate. Magnesium sulfate is not administered for renal dysfunction and can cause magnesium toxicity in the patient. Pulmonary enema can be prevented by restricting the patient's fluid intake to 125 mL/hr. Increasing magnesium toxicity can cause low BP in the patient.

The nurse is caring for a pregnant patient who is receiving antibiotic therapy to treat a urinary tract infection (UTI). Which dietary changes does the nurse suggest for the pregnant patient who is receiving antibiotic therapy for UTI? 1 "Include yogurt, cheese, and milk in your diet." 2 "Avoid folic acid supplements until the end of therapy." 3 "Include vitamins C and E supplementation in your diet." 4 "Reduce your dietary fat intake by 40 to 50 g per day.

1 The antibiotic therapy kills normal flora in the genitourinary tract, as well as pathologic organisms. Therefore the nurse instructs the patient to include yogurt, cheese, and milk in daily diet because they contain active acidophilus cultures. Folic acid should not be avoided, because it may affect the fetal development. Vitamins C and E supplementation is usually included in the diet to treat preeclampsia in a patient. Dietary fat is reduced in patients with cholecystitis or cholelithiasis, because it may cause epigastric pain.

When using intermittent auscultation (IA) to assess uterine activity, nurses should be aware that: 1 the examiner's hand should be placed over the fundus before, during, and after contractions. 2 the frequency and duration of contractions are measured in seconds for consistency. 3 contraction intensity is given a judgment number of 1 to 7 by the nurse and patient together. 4 the resting tone between contractions is described as either placid or turbulent.

1 The assessment includes palpation; duration, frequency, intensity, and resting tone. The duration of contractions is measured in seconds; the frequency is measured in minutes. The intensity of contractions usually is described as mild, moderate, or strong. The resting tone usually is characterized as soft or relaxed.

In documenting labor experiences, nurses should know that a uterine contraction is described according to all these characteristics except:

Appearance shape and height

Which conditions during pregnancy can result in preeclampsia in the patient? Select all that apply. 1 Genetic abnormalities 2 Dietary deficiencies 3 Abnormal trophoblast invasion 4 Cardiovascular changes 5 Maternal hypotension

1, 2, 3, 4 Current theories consider that genetic abnormalities and dietary deficiencies can result in preeclampsia. Abnormal trophoblast invasion causes fetal hypoxia and results in maternal hypertension. Cardiovascular changes stimulate the inflammatory system and result in preeclampsia in the pregnant patient. Maternal hypertension, and not hypotension, after 20 weeks' gestation is known as preeclampsia.

Which hypertensive disorders can occur during pregnancy? Select all that apply. 1 Chronic hypertension 2 Preeclampsia-eclampsia 3 Hyperemesis gravidarum 4 Gestational hypertension 5 Gestational trophoblastic disease

1, 2, 4 Chronic hypertension refers to hypertension that developed in the pregnant patient before 20 weeks' gestation. Preeclampsia refers to hypertension and proteinuria that develops after 20 weeks' gestation. Eclampsia is the onset of seizure activity in a pregnant patient with preeclampsia. Gestational hypertension is the onset of hypertension after 20 weeks' gestation. Gestational trophoblastic disease and hyperemesis gravidarum are not hypertensive disorders. Gestational trophoblastic disease refers to a disorder without a viable fetus that is caused by abnormal fertilization. Hyperemesis gravidarum is excessive vomiting during pregnancy that may result in weight loss and electrolyte imbalance.

What interventions does the nurse perform for a laboring patient with hypotension and fetal bradycardia? Select all that apply. 1 Notify the primary health care provider or anesthesiologist. 2 Monitor the fetal heart rate (FHR) every 5 minutes. 3 Monitor maternal blood pressure every 10 minutes. 4 Administer oxygen using a non rebreather facemask. 5 Position the patient in Sims' or modified Sims' position.

1, 2, 4 The nurse must immediately notify the primary health care provider, anesthesiologist, or nurse anesthetist. The nurse must administer oxygen by nonrebreather facemask at 10 to 12 L/min or as per health care facility's protocol. The FHR must be monitored every 5 minutes. The patient must be turned to lateral position or a pillow or wedge must be placed under a hip to displace the uterus. Sims' or modified Sims' position may be used when spinal anesthesia is administered. Maternal blood pressure must be monitored every 5 minutes.

What does the nurse assess to detect the presence of a hypertensive disorder in a pregnant patient? Select all that apply. 1 Proteinuria 2 Epigastric pain 3 Placenta previa 4 Presence of edema 5 Blood pressure (BP)

1, 2, 4, 5 Proteinuria indicates hypertension in a pregnant patient. Proteinuria is concentration ≥300 mg/24 hours in a 24-hour urine collection. The nurse needs to assess the patient for epigastric pain because it indicates severe preeclampsia. Hypertension is likely to cause edema or swollen ankles as a result of greater hydrostatic pressure in the lower parts of the body. Therefore the nurse needs to assess the patient for the presence of edema. Accurate measurement of BP will help detect the presence of any hypertensive disorder. A systolic BP greater than 140 mm Hg or a diastolic BP greater than 90 mm Hg will indicate hypertension. Placenta previa is a condition wherein the placenta is implanted in the lower uterine segment covering the cervix, which causes bleeding when the cervix dilates.

What does the nurse include in the plan of care of a patient with a cardiac disorder during the postpartum period? Select all that apply. 1 Monitor oxygen saturation levels. 2 Place the newborn at the bedside. 3 Put the patient on a full liquid diet. 4 Teach the patient how to breastfeed. 5 Have the patient talk to the newborn.

1, 2, 4, 5 The nurse monitors oxygen saturation levels in the patient to assess for adequate oxygenation. The nurse places the infant at the bedside so that the patient can touch the infant without expending energy. This also helps establish an emotional bond. The nurse assists the patient in breastfeeding by positioning the infant correctly for feeding. The nurse encourages the patient to talk to the newborn to involve the mother in the infant's care and help the patient feel vitally important. A fluid diet is not prescribed, because it does not ensure adequate nutrition for the infant.

A 24-year-old primipara, who is 18 weeks pregnant, has been having increasing vomiting since she was 8 weeks pregnant. Upon arrival at the emergency department, her skin turgor is diminished, temperature is 99.2F (o), pulse is 102, respiration is 18, blood pressure is 102/68, and she has deep furrows on her tongue. What would the nurse expect to do to care for her? Select all that apply. 1 Start an intravenous infusion. 2 Check her urine for ketones 3 Cross match blood for a transfusion. 4 Obtain a complete history. 5 Obtain blood for a complete blood count

1, 2, 4, 5 Whenever a pregnant woman has nausea and vomiting, the first priority is a thorough assessment to determine the severity of the problem. In most cases the woman should be told to come immediately to the health care provider's office or the emergency department because the severity of the illness often is difficult to determine by telephone conversation. The assessment should include frequency, severity, and duration of episodes of nausea and vomiting. If the woman reports vomiting, the assessment also should include the approximate amount and color of the vomitus. The woman is asked to report any precipitating factors relating to the onset of her symptoms. Any pharmacologic or nonpharmacologic treatment measures used should be recorded. Prepregnancy weight and documented weight gain or loss during pregnancy are important to note. The woman's weight and vital signs are measured, and a complete physical examination is performed, with attention to signs of fluid and electrolyte imbalance and nutritional status. The most important initial laboratory test to be obtained is a determination of ketonuria. Other laboratory tests that may be prescribed are a urinalysis, a complete blood cell count, electrolytes, liver enzymes, and bilirubin levels. At this time, there is no supportive evidence that a blood transfusion is required. Based on provided objective data that the patient has deep furrows on her tongue, this may suggest a vitamin B deficiency which should be investigated further.

The nurse is developing the plan of care for a pregnant patient with an underlying history of cardiovascular disease. Which conditions would this patient be at risk for? Select all that apply. 1 Stillbirth 2 Miscarriage 3 Hypoglycemia 4 Atrial septal defect 5 Intrauterine growth restriction

1, 2, 5 Stillbirth and miscarriage may occur because of cardiovascular problems in pregnancy. Intrauterine growth restriction results in a pregnant patient with cardiovascular disease because of low oxygen pressure. Hypoglycemia is seen in a pregnant patient with diabetes because a decrease in glucose levels. An atrial septal defect is a congenital birth defect that is not related to underlying maternal cardiac disease.

A pregnant woman is being examined by the nurse in the outpatient obstetric clinic. The nurse suspects systemic lupus erythematosus (SLE) after the examination reveals which symptoms? Select all that apply. 1 Muscle aches 2 Hyperactivity 3 Weight changes 4 Fever 5 Hypotensio

1, 3, 4 Common symptoms, including myalgias, fatigue, weight change, and fevers, occur in nearly all women with SLE at some time during the course of the disease. Fatigue, rather than hyperactivity, is a common sign of SLE. Hypotension is not a characteristic sign of SLE. Although a diagnosis of SLE is suspected based on clinical signs and symptoms, it is confirmed by laboratory testing that demonstrates the presence of circulating autoantibodies. As with other autoimmune diseases, SLE is characterized by a series of exacerbations (flares) and remissions (Chin and Branch, 2012).

The nurse is caring for a woman who is at 24 weeks of gestation with suspected severe preeclampsia. Which signs and symptoms should the nurse expect to observe? Select all that apply. 1 Decreased urinary output and irritability 2 Transient headache and +1 proteinuria 3 Ankle clonus and epigastric pain 4 Platelet count of less than 100,000/mm3 and visual problems 5 Seizure activity and hypotension

1, 3, 4 Decreased urinary output and irritability are signs of severe eclampsia. Ankle clonus and epigastric pain are signs of severe eclampsia. Platelet count of less than 100,000/mm3 and visual problems are signs of severe preeclampsia. A transient headache and +1 proteinuria are signs of preeclampsia and should be monitored.Seizure activity and hyperreflexia are signs of eclampsia. Test-Taking Tip: Do not worry if you select the same numbered answer repeatedly, because there usually is no pattern to the answers.

The nurse teaches the patient nonpharmacologic pain management methods during a prenatal class. Which methods require practice for best results? Select all that apply. 1 Biofeedback 2 Massage and touch 3 Patterned breathing 4 Controlled relaxation 5 Slow-paced breathing

1, 3, 4 Patterned breathing, controlled relaxation, and biofeedback techniques must be practiced to obtain best results. Patterned breathing and controlled relaxations help to manage pain during labor. Biofeedback is effective when the patient is able to focus and control body responses during labor. The nurse assisting the laboring patient can use methods such as massage and touch and slow-paced breathing successfully without the patient having any prior knowledge about it.

Which interventions does the nurse implement to ease the labor process of a pregnant patient with heart disease? Select all that apply. 1 Place the patient in a side-lying position. 2 Use stirrups to facilitate an easy labor. 3 Monitor the patient's oxygen saturation. 4 Maintain a peaceful, calm environment. 5 Provide the patient with a relaxing back rub.

1, 3, 4, 5 The nurse places the patient in a side-lying position to facilitate uterine perfusion. The nurse monitors the patient's oxygen saturation to assess for adequate oxygenation. The nurse maintains a calm environment to minimize the patient's anxiety. The nurse also provides a back massage to comfort the patient. Stirrups are not used because they may prevent compression of the popliteal veins or increase in blood volume in the chest.

A patient inquires about the use of hypnosis for pain management during a prenatal assessment. What does the nurse teach the patient about this modality? Select all that apply. 1 It gives a better sense of control. 2 It must be performed by a support person. 3 It is a form of deep relaxation or meditation. 4 It is more effective than the use of a placebo. 5 It can cause dizziness, nausea, and headache

1, 3, 5 Hypnosis is a form of deep relaxation, similar to daydreaming or meditation. It enhances relaxation and diminishes fear, anxiety, and perception of pain. It allows the patient to have a greater sense of control over painful contractions. Failure to dehypnotize properly may result in mild dizziness, nausea, and headache. Self-hypnosis must be learnt during childbirth preparation classes. It is not performed by a support person. Although hypnosis is beneficial, studies have not found it to be more effective than the use of a placebo or other interventions for pain management during labor.

What interventions does the nurse perform to provide emotional support to a patient in labor? Select all that apply. 1 Compliment patient efforts during labor. 2 Avoid offering food during labor. 3 Use a calm, confident approach. 4 Discourage activities that distract. 5 Involve the patient in care decisions.

1, 3, 5 The nurse must offer emotional support by complimenting the patient and offering positive reinforcement for efforts during labor. The patient must be involved in decision making regarding own care. The nurse must use a calm and confident approach when assisting the patient during labor. The nurse may offer food and nourishment, if allowed by the primary health care provider. The nurse must encourage participation in distracting activities and nonpharmacologic measures for comfort.

After monitoring the fetal heart activity, the nurse concludes that there is impaired fetal oxygenation. What had the nurse observed in the fetal monitor to come to this conclusion? Select all that apply. 1 Increase in the fetal heart rate (FHR) to over 160 beats/min 2 Early decelerations 3 Moderate variability 4 Late decelerations 5 Occasional variable decelerations

1, 4 Tachycardia (an increase in the FHR) is the early sign of fetal hypoxemia. Prolonged decelerations in FHR lasting for more than 2 minutes indicates the fetus is hypoxemic. Early decelerations, moderate variability, and occasional variable decelerations in the FHR are common observations during labor. These are normal findings and require no intervention.

The nurse is teaching a couple about the use of imagery and visualization in managing pain during labor. What is the patient expected to do during this technique? Select all that apply. 1 Imagine breathing in light and energy. 2 Maintain clenched fists to drive out pain. 3 Engage in dance or rhythmic movements. 4 Imagine walking through a restful garden. 5 Envisage breathing out worries and tension.

1, 4, 5 Imagery and visualization are useful techniques in preparation for birth and are often used in combination with relaxation. Imagery involves techniques, such as breathing in light and energy, imagining a walk through a restful garden, or envisaging breathing out worries and tension. Relaxation or reduction of body tension is a technique that involves rhythmic motion that stimulates the mechanoreceptors of the brain. The nurse must recognize the signs of tension, such as clenching of fists when in pain by the laboring patient.

The nurse teaches acupressure methods for pain relief during labor to a couple in the prenatal clinic. What does the nurse teach about acupressure? Select all that apply. 1 Blood circulation is enhanced. 2 Flow of qi (energy) is restored. 3 Lubricants are used over the area. 4 Pressure is applied with the fingers. 5 Pressure is applied with contractions.

1, 4, 5 Pressure is usually applied with the heel of the hand, fist, or pads of the thumbs and fingers. Pressure is applied with contractions initially and then continuously as labor progresses to the transition phase at the end of the first stage of labor. Acupressure is said to promote the circulation of the blood, the harmony of yin and yang, and the secretion of neurotransmitters. Thus acupressure maintains normal body functions and enhances well-being. Acupressure is applied over the skin without using lubricants. In acupuncture, the flow of qi (energy) is restored.

when are you more likely to see maternal hypoglycemia

1st trimester

Which is the ideal treatment for severe unmanageable hyperthyroidism in a patient who is pregnant? 1 Radioactive iodine 2 Subtotal thyroidectomy 3 Methimazole (Tapazole) 4 Propylthiouracil (Propacil

2 A subtotal thyroidectomy is prescribed for a pregnant patient with severe hyperthyroidism if the drug therapy proves toxic. Oral methimazole and propylthiouracils are prescribed for hyperthyroidism but may be ineffective in severe cases. Radioactive iodine is not used to treat hyperthyroidism in pregnant patients, because it may destroy the fetus's thyroid gland.

What care must the nurse take when implementing aromatherapy for a patient in labor? 1 Apply oil to the skin and massage. 2 Ask the patient to choose the scents. 3 Apply a few drops of oil to the hair. 4 Allow inhalation of warm oil vapors

2 Certain scents can evoke pleasant memories and feelings of love and security. So, it is helpful if the patient is allowed to choose the scents. The oils must never be applied in full strength directly on to the skin. Most oils should be diluted in a vegetable oil base before use. Inhaling vapors from the oil can lead to unpleasant side effects like nausea or headache. Drops of essential oils can be put on a pillow or on a woman's brow or palms or used as an ingredient in creating massage oil. It is not applied to the hair. STUDY TIP: Do not change your pattern of study. It obviously has contributed to your being here, so it worked. If you have studied alone, continue to study alone. If you have studied in a group, form a study group.

The nurse observes that a pregnant patient with gestational hypertension who is on magnesium sulfate therapy is prescribed nifedipine (Adalat). What action does the nurse take? 1 Evaluates the patient's renal function test 2 Obtains a prescription for a change of drug 3 Reduces the nifedipine (Adalat) dose by 50% 4 Administers both medications simultaneously

2 Concurrent use of nifedipine (Adalat) and magnesium sulfate can result in skeletal muscle blockade in the patient. Therefore the nurse needs to report immediately to the primary health care provider (PHP) and obtain a prescription for a change of drug. The nurse assesses the patient's renal function to determine the risk for toxicity after administering any drug. However, it is not a priority in this case. Reducing the nifedipine (Adalat) dose is not likely to prevent the drug interaction in the patient. The nurse does not administer both drugs simultaneously because it may be harmful for the patient.

Diabetes in pregnancy puts the fetus at risk in several ways. Nurses should be aware that: 1 with good control of maternal glucose levels, sudden and unexplained stillbirth is no longer a major concern. 2 the most important cause of perinatal loss in diabetic pregnancy is congenital malformations. 3 infants of mothers with diabetes have the same risks for respiratory distress syndrome because of the careful monitoring. 4 at birth, the neonate of a diabetic mother is no longer at any greater risk.

2 Congenital malformations account for 30% to 50% of perinatal deaths. Even with good control, sudden and unexplained stillbirth remains a major concern. Infants of diabetic mothers are at increased risk for respiratory distress syndrome. The transition to extrauterine life often is marked by hypoglycemia and other metabolic abnormalities.

When a pregnant woman with diabetes experiences hypoglycemia while hospitalized, what should the nurse have the woman do? 1 Eat a candy bar. 2 Eat six saltine crackers or drink 8 oz of milk. 3 Drink 4 oz of orange juice followed by 8 oz of milk. 4 Drink 8 oz of orange juice with 2 teaspoons of sugar added

2 Crackers provide carbohydrates in the form of polysaccharides. A candy bar provides only monosaccharides. Milk is a disaccharide and orange juice is a monosaccharide. This will provide an increase in blood sugar but will not sustain to level. Orange juice and sugar will increase the blood sugar, but not provide a slow-burning carbohydrate to sustain the blood sugar. Test-Taking Tip: Being prepared reduces your stress or tension level and helps you maintain a positive attitude.

A pregnant woman with cardiac disease is informed about signs of cardiac decompensation. She should be told that the earliest sign of decompensation is most often: 1 orthopnea. 2 decreasing energy levels. 3 moist frequent cough and frothy sputum. 4 crackles (rales) at the bases of the lungs on auscultation.

2 Decreasing energy level (fatigue) is an early finding of heart failure. Care must be taken to recognize it as a warning rather than a typical change of the third trimester. Cardiac decompensation is most likely to occur early in the third trimester, during childbirth, and during the first 48 hours following birth. Orthopnea, a moist, frequent cough, and crackles and rales appear later when a failing heart reduces renal perfusion and fluid accumulates in the pulmonary interstitial space, leading to pulmonary edema.

The nurse is assisting a patient in labor. What breathing pattern must the nurse remind the patient to use when the contractions increase in frequency and intensity in the first phase of labor? 1 Slow-paced breathing 2 Modified-paced breathing 3 3:1 pattern-paced breathing 4 4:1 pattern-paced breathing

2 During the first phase of labor, as contractions increase in frequency and intensity, the patient must change breathing patterns to a modified-paced breathing technique. This breathing pattern is shallower and faster than the patient's normal rate of breathing, but should not exceed twice the resting respiratory rate. Slow-paced breathing is performed at approximately half the normal breathing rate and is initiated when the patient can no longer walk or talk through contractions. Patterned-paced breathing is suggested in the second phase of labor. It consists of panting breaths combined with soft blowing breaths at regular intervals. The patterns may vary, the 3:1 pattern is pant, pant, pant, blow and the 4:1 pattern is pant, pant, pant, pant, and blow.

Which is an important nursing intervention when a patient has an incomplete miscarriage with heavy bleeding? 1 Initiate expectant management at once. 2 Prepare the patient for dilation and curettage. 3 Administer the prescribed oxytocin (Pitocin). 4 Obtain a prescription for ergonovine (Methergine).

2 In the case of an incomplete miscarriage, sometimes there is heavy bleeding and excessive cramping and some part of fetal tissue remains in the uterus. Therefore the nurse needs to prepare the patient for dilation and curettage for the removal of the fetal tissue. Expectant management is initiated if the pregnancy continues after a threatened miscarriage. Oxytocin (Pitocin) is administered to prevent hemorrhage after evacuation of the uterus. Ergonovine (Methergine) is administered to contract the uterus.

What does the nurse teach the patient about the benefits of breathing techniques in the second stage of labor? 1 Does not interfere with fetal descent 2 Causes increase in abdominal pressure 3 Reduces discomfort during contractions 4 Increases the size of the abdominal cavity

2 In the second stage of labor breathing technique is used to increase abdominal pressure and expel the fetus. In the first stage of labor, breathing helps to promote the relaxation of the abdominal muscles, thereby increasing the size of the abdominal cavity. This lessens the discomfort during contraction caused by the friction between the abdominal wall and the uterus. It also relaxes the muscles of the genital area and does not interfere with fetal descent. Test-Taking Tip: Do not panic while taking an exam! Panic will only increase your anxiety. Stop for a moment, close your eyes, take a few deep breaths, and resume review of the question.

Which medication is administered to a pregnant patient to treat hyperthyroidism? 1 Isotretinoin (Accutane) 2 Methimazole (Tapazole) 3 Levothyroxine (Synthroid) 4 Sodium iodide 131I (Hicon

2 Methimazole (Tapazole) is administered to a pregnant patient to control symptoms of hyperthyroidism, improve weight gain, and reduce tachycardia. Isotretinoin (Accutane) is prescribed for cystic acne. It is not prescribed during pregnancy, because it is highly teratogenic. Levothyroxine (Synthroid) is used to treat hypothyroidism in pregnant patients who do not have a functioning thyroid tissue. Sodium iodide 131I (Hicon) is radioactive iodine. It is not used to treat hyperthyroidism in pregnant patients, because it may destroy the fetal thyroid. Test-Taking Tip: Eat breakfast or lunch before an examination. Avoid greasy, heavy foods and overeating. This will help keep you calm and give you energy.

The most prevalent clinical manifestation of abruptio placentae (as opposed to placenta previa) is: 1 bleeding. 2 intense abdominal pain. 3 uterine activity. 4 cramping

2 Pain is absent with placenta previa but may be agonizing with abruptio placentae. Bleeding may be present in varying degrees for both placental conditions. Uterine activity may be present with both placental conditions. Cramping is a form of uterine activity that may be present in both placental conditions.

Which intervention does the nurse implement for a patient immediately after a severe abdominal trauma? 1 Prep the patient for cesarean birth. 2 Send the patient for pelvic computed tomography (CT) scanning. 3 Provide fluids to the patient as part of the protocol for ultrasound examination. 4 Prepare to administer Rho(D) immunoglobulin.

2 Pelvic CT scanning helps visualize extraperitoneal and retroperitoneal structures and the genitourinary tract. The nurse needs to prepare the patient for cesarean birth if there is no evidence of a maternal pulse. Ultrasound examination is not as effective as electronic fetal monitoring for determining placental abruption in the patient after the trauma. Therefore the nurse prepares the patient for a CT scan after a severe abdominal trauma. The nurse needs to administer Rho(D) immunoglobulin in an Rh-negative pregnant trauma patient. This helps protect the patient from isoimmunization.

A pregnant patient has a systolic blood pressure that exceeds 160 mm Hg. Which action should the nurse take for this patient? 1 Administer magnesium sulfate intravenously. 2 Obtain a prescription for antihypertensive medications. 3 Restrict intravenous and oral fluids to 125 mL/hr. 4 Monitor fetal heart rate (FHR) and uterine contractions (UCs).

2 Systolic blood pressure exceeding 160 mm Hg indicates severe hypertension in the patient. The nurse should alert the health care provider and obtain a prescription for antihypertensive medications, such as nifedipine (Adalat) and labetalol hydrochloride (Normodyne). Magnesium sulfate would be administered if the patient was experiencing eclamptic seizures. Oral and intravenous fluids are restricted when the patient is at risk for pulmonary edema. Monitoring FHR and UCs is a priority when the patient experiences a trauma so that any complications can be addressed immediately.

The diagnostic test reports of a pregnant patient reveal a baseline fetal heart rate of 175 beats/min. What does this finding indicate to the nurse? 1 Presence of fetal ischemia 2 Fetal tachycardia 3 Fetal bradycardia 4 Hypotension in the fetus

2 The normal baseline fetal heart rate ranges from 110 to 160 beats/min. If the fetal heart rate is more than 160 beats/min, then tachycardia in the fetus is indicated. Ischemia is a condition in which there is a reduced blood supply to the fetal tissues. Baseline heart rate below 110 beats/min indicates bradycardia in fetus. Hypotension indicates a blood pressure level below 120/80 mm Hg, which is a life-threatening condition for the fetus.

A pregnant patient is at risk for cardiac arrest as a result of profound hypovolemia after a trauma. Which action does the nurse take? The nurse: 1 Assesses airway, breathing, and pulse rate. 2 Administers warmed crystalloid solutions. 3 Administers calcium gluconate intravenously. 4 Obtains a prescription for magnesium sulfate.

2 The nurse administers warmed crystalloid solutions for massive fluid resuscitation in the patient who has profound hypovolemia after a trauma. The nurse needs to assess the airway, breathing, and pulse in a patient after a convulsion so that prompt actions can be taken to stabilize the patient. The nurse administers calcium gluconate as an antidote to a patient who has magnesium toxicity. The nurse may administer magnesium sulfate for the treatment of eclamptic seizures in a patient with preeclampsia.

A 24-year-old primipara, 10 weeks pregnant, who has been experiencing vomiting every morning for the past few weeks, asks the nurse at her check-up how long this "morning sickness" will continue. Which statement by the nurse is most accurate? 1 "It will end by the 15th week of pregnancy." 2 "It usually subsides by the 20th week of pregnancy." 3 "It's a very common but not serious problem." 4 "In some women, it can last throughout the pregnancy and become serious."

2 This discomfort of pregnancy usually subsides by the 20th week of pregnancy. An absolute definite end of vomiting during pregnancy can never be stated. Test-Taking Tip: Identifying content and what is being asked about that content is critical to your choosing the correct response. Be alert for words in the stem of the item that are the same or similar in nature to those in one or two of the options. Example: If the item relates to and identifies stroke rehabilitation as its focus and only one of the options contains the word stroke in relation to rehabilitation, you are safe in identifying this choice as the correct response.

After observing the electronic fetal monitor, a primary health care provider asks the nurse to conduct an electrocardiogram (ECG) of the fetus. What should the nurse assess before obtaining an ECG of the fetus? Select all that apply. 1 Fetal lactate levels 2 Placental membranes 3 Cervical dilation 4 Umbilical cord compression 5 Frequency of uterine contractions

2, 3 When performing the ECG of the fetus, the nurse should insert the electrode into the cervix to reach the fetus. Therefore the nurse should check if the cervix is dilated up to 3 cm and if the membranes are ruptured. This allows the nurse to reach the fetus's position. Lactate levels do not affect the ECG testing and thus need not be checked before the test. Umbilical cord compression or decreased frequency of UCs is not the required conditions for performing an ECG on the fetus.

A pregnant woman with type 1 diabetes is on rapid-acting, short-acting, and intermediate-acting insulin injections. Which are rapid and short-acting insulins? Select all that apply. 1 NPH (Novolin N) 2 Regular (Humalin) 3 Lispro (Humalog) 4 Aspart (NovoLog) 5 Glargine (Lantus)

2, 3, 4 Humalog and NovoLog are rapid-acting insulins and Humalin is a short-acting insulin. Novolin N is an intermediate-acting insulin and Lantus is a long-acting insulin.

A pregnant patient with severe preeclampsia who is being transported to a tertiary care center needs to be administered magnesium sulfate injection for seizure activity. What actions does the nurse take when administering the drug? Select all that apply. 1 A 10-g dose is administered in the buttock. 2 A local anesthetic is added to the solution. 3 The Z-track technique is used to inject the drug. 4 The injection site is massaged after the injection. 5 The subcutaneous route is used to inject the drug.

2, 3, 4 The nurse adds a local anesthetic to the solution to reduce pain that is caused by the injection. The Z-track technique is used to inject the drug so that the drug is injected in the intramuscular (IM) tissue safely. The nurse gently massages the site after administering the injection to reduce pain. The nurse administers two separate injections of 5 g in each buttock. Magnesium sulfate injections are administered in the IM layer and not the subcutaneous layer.

What are the possible causes of miscarriage during early pregnancy? Select all that apply. 1 Premature dilation of cervix 2 Chromosomal abnormalities 3 Endocrine imbalance 4 Hypothyroidism 5 Antiphospholipid antibodies

2, 3, 4, 5 Chromosomal abnormalities account for 50% of all early pregnancy losses. Endocrine imbalance is caused by luteal phase defects, hypothyroidism, and diabetes mellitus in pregnant patients and results in miscarriage. Antiphospholipid antibodies also increase the chances of miscarriage in pregnant patients. Premature dilation of the cervix may cause a second-trimester loss and is usually seen in patients between 12 and 20 weeks' gestation.

A patient with gestational diabetes tells the nurse, "I'm extremely worried that my child will be diabetic, too." Which actions does the nurse take to alleviate the patient's anxiety? Select all that apply. 1 Evaluate the test results to assess fetal growth. 2 Listen to the feelings and concerns of the patient. 3 Provide factual information of risks to the patient. 4 Use therapeutic communication with the patient. 5 Ask the patient to share any fears with the nurse.

2, 3, 4, 5 The nurse listens to the patient's feelings and concerns to assess for any misconception or misinformation that can be causing anxiety. The nurse provides factual information about any risks to the patient to correct any misconceptions. Using therapeutic communication will develop an open relationship that also helps promote trust. The nurse encourages the patient to share concerns with the nursing staff to promote collaboration in the care process. Evaluating test reports for fetal growth will help assess fetal well-being, although it does not help alleviate the patient's fears.

What does the nurse include in the plan of care of a pregnant patient with mild preeclampsia? Select all that apply. 1 Ensure prolonged bed rest. 2 Provide diversionary activities. 3 Encourage the intake of more fluids. 4 Restrict sodium and zinc in the diet. 5 Refer to Internet-based support group

2, 3, 5 Activity is restricted in patients with preeclampsia, so it is necessary to provide diversionary activities to such patients to prevent boredom. The nurse encourages the patient to increase fluid intake to enhance renal perfusion and bowel function. The nurse can suggest Internet-based support groups to reduce boredom and stress in the patient. Patients need to restrict activity, but complete bed rest is not advised because it may cause cardiovascular deconditioning, muscle atrophy, and psychological stress. The patient needs to include adequate zinc and sodium in the diet for proper fetal development.

A pregnant woman presents to the emergency department complaining of persistent nausea and vomiting. She is diagnosed with hyperemesis gravidarum. The nurse should include which information when teaching about diet for hyperemesis? Select all that apply. 1 Eat three larger meals a day. 2 Eat a high-protein snack at bedtime. 3 Ice cream may stay down better than other foods. 4 Avoid ginger tea or sweet drinks. 5 Eat what sounds good to you even if your meals are not well-balanced.

2, 3, 5 The diet for hyperemesis includes: (1) Avoid an empty stomach. Eat frequently, at least every 2 to 3 hours. Separate liquids from solids and alternate every 2 to 3 hours. (2) Eat a high-protein snack at bedtime. (3) Eat dry, bland, low-fat, and high-protein foods. Cold foods may be better tolerated than those served at a warm temperature. (4) In general eat what sounds good to you rather than trying to balance your meals. (5) Follow the salty and sweet approach; even so-called junk foods are okay. (6) Eat protein after sweets. (7) Dairy products may stay down more easily than other foods. (8) If you vomit even when your stomach is empty, try sucking on a Popsicle. (9) Try ginger tea. Peel and finely dice a knuckle-sized piece of ginger and place it in a mug of boiling water. Steep for 5 to 8 minutes and add brown sugar to taste. (10) Try warm ginger ale (with sugar, not artificial sweetener) or water with a slice of lemon. (11) Drink liquids from a cup with a lid.

A pregnant patient with chronic hypertension is at risk for placental abruption. Which symptoms of abruption does the nurse instruct the patient to be alert for? Select all that apply. 1 Weight loss 2 Abdominal pain 3 Vaginal bleeding 4 Shortness of breath 5 Uterine tenderness

2, 3, 5 The nurse instructs the pregnant patient to be alert for abdominal pain, vaginal bleeding, and uterine tenderness as these indicates placental abruption. Weight loss indicates fluid and electrolyte loss and not placental abruption. Shortness of breath indicates inadequate oxygen, which is usually seen in a patient who is having cardiac arrest.

Which device can be used as a noninvasive way to assess the fetal heart rate (FHR) in a patient whose membranes are not ruptured? 1 Tocotransducer 2 Spiral electrode 3 Ultrasound transducer 4 Intrauterine pressure catheter (IUPC)

3 An ultrasound transducer is used to assess the FHR by an external mode of electronic fetal monitoring. It does not require membrane rupture and cervical dilation. A tocotransducer can be used to assess the uterine activity (UA) in a pregnant patient whose cervix is not sufficiently dilated, but it does not assess the FHR. Spiral electrode is used as an internal mode of electronic fetal monitoring to assess the FHR. It can be used only when the membranes are ruptured and the cervix is dilated during the intrapartum period. IUPC is used to assess uterine activity in internal mode. It can be used only when the membranes are ruptured and the cervix is dilated during the intrapartum period.

The nurse instructs a pregnant patient to breathe through the mouth and keep it open while pushing during labor. What is the rationale for this nursing intervention? 1 To avoid nasal congestion in the patient 2 To decrease the efforts required for pushing 3 To facilitate increased oxygen to the fetus 4 To avoid deceleration in the fetal heart rate

3 During labor, the nurse asks the patient to breathe through the mouth to keep the mouth open to increase both maternal and fetal oxygenation. Nasal congestion is not a complication associated with labor. Opening of the mouth does not increase the pushing capability. Early decelerations are observed by pushing which does not require any intervention.

The nurse observes that maternal hypotension has decreased uterine and fetal perfusion in a pregnant patient. What does the nurse need to assess further to understand the maternal status? 1 D-dimer blood test 2 Kleihauer-Betke (KB) test 3 Electronic fetal monitoring 4 Electrocardiogram reading

3 Electronic fetal monitoring reflects fetal cardiac responses to hypoxia and hypoperfusion and helps to assess maternal status after a trauma. The D-dimer blood test is used to rule out the presence of a thrombus. The KB test is used to evaluate transplacental hemorrhage. Electrocardiogram reading is more useful to assess the cardiac functions in nonpregnant cardiac patients.

The nurse is caring for a patient who is using fentanyl citrate (Sublimaze) through patient-controlled analgesia (PCA) while in labor. What effects of fentanyl citrate does the nurse expect? 1 Provides long duration of action 2 Requires only a single dose 3 Provides quick relief to pain 4 Causes sedation and nausea

3 Fentanyl citrate (Sublimaze) is a potent short-acting opioid agonist analgesic. Therefore it provides quick pain relief. It rapidly crosses the placenta, so it is present in the fetal blood within 1 minute after intravenous maternal administration. It is a short-acting drug, so the patient will require more frequent dosing. It is often administered as a patient controlled analgesic. It has fewer neonatal effects as compared to meperidine, and causes less maternal sedation and nausea.

At 37 weeks of gestation, the patient is in a severe automobile crash where her abdomen was hit by the steering wheel and her seat belt. What actions would the emergency room nurse expect to perform upon the patient's arrival at the hospital? 1 Stay with the patient, assure a patent airway is present, and keep the patient as calm as possible. 2 Move the patient's skirt to determine if any vaginal bleeding is present, find out who to call, and monitor the level of consciousness. 3 Assess the patient's vital signs, determine location and severity of pain, and establish continual fetal heart rate monitoring. 4 Obtain arterial blood gases, obtain a hemoglobin and hematocrit, and oxygen saturation rate.

3 Full assessment of the patient and her fetus are essential and include vital signs, continual fetal heart rate monitoring, determining the location and severity of pain, whether any vaginal bleeding is dark red or bright red, and the status of the abdomen, which would be expected to be rigid or "board like." Staying with the patient, assuring a patent airway is present, and keeping the patient as calm as possible would be appropriate at the crash site before the arrival of emergency medical services (EMS). The current status of the patient and fetus are thepriority. The health care provider would prescribe the arterial blood gases and other laboratory work after the patient is assessed and stabilized.

What does the nurse recommend to a pregnant patient with diabetes who works long, irregular hours? 1 "Eat a snack hourly when at work." 2 "Try taking naps when you are free." 3 "Keep fruits or fruit juice available." 4 "Quit working for a while."

3 If the patient has to be away from home for long hours, the nurse advises the patient to carry fruits or fruit juices. They contain simple carbohydrates that help control blood glucose levels. The nurse should not advise the patient to quit working, because it may not be feasible for the patient. Instead, the nurse should encourage the patient to follow a consistent daily schedule. Taking naps when free ensures rest, but it does not help keep glucose levels in check. Eating a snack every hour is not advised, because it may fluctuate blood glucose levels. Instead, three meals and two or three snacks are advised.

The primary health care provider has administered general anesthesia to a patient who is scheduled for an elective cesarean section. What changes should the nurse observe in the fetal heart rate (FHR) after the administration of general anesthesia? 1 Decrease 2 Increase 3 Minimal variability 4 Moderate variability

3 It is necessary to monitor the FHR in the pregnant patient who is given general anesthesia. General anesthesia usually causes minimal variability or no change in the FHR. Tachycardia is caused by fetal hypoxemia, whereas bradycardia is caused from a structural defect in the fetal heart. Moderate variability in the FHR indicates normal fetal activity.

The nurse is caring for a nulliparous patient in labor. How is the experience for a nulliparous patient different from that of a multiparous patient? The patient experiences: 1 Less sensory pain during early labor. 2 Greater sensory pain in the second stage of labor. 3 Greater fatigue due to longer duration of labor. 4 Greater affective pain in the second stage of labor.

3 Parity influences the perception of labor pain. The nulliparous patient often has longer labor and therefore, greater fatigue. Sensory pain for nulliparous women is often greater than that for multiparous women during early labor, because their reproductive tract structures are less supple. Affective pain in the nulliparous patient is greater in the first stage as compared to a multiparous patient. It decreases for both patients during the second stage of labor. During the second stage of labor, the multiparous patient may experience greater sensory pain than the nulliparous patient. This is because tissues of the multiparous patient are more supple and increase the speed of fetal descent, thereby intensifying the pain. Test-Taking Tip: Answer every question because, on the NCLEX exam, you must answer a question before you can move on to the next question.

What does the nurse advise a pregnant patient who is prescribed phenazopyridine (Pyridium) for cystitis? 1 "Avoid sweet foods in diet." 2 "Limit exposure to sunlight." 3 "Do not wear contact lenses." 4 "Restrict oral fluids to 125 mL per hour."

3 Phenazopyridine (Pyridium) colors the tears orange. Therefore the nurse instructs the patient to avoid wearing contact lenses. Sweet foods are avoided in patients with diabetes mellitus, because they can cause fluctuating glucose levels, which may harm the fetus. Exposure to sunlight is avoided when the patient is receiving methotrexate therapy, because it causes photosensitivity. Oral fluids are restricted in patients who are at risk for pulmonary edema.

What instruction does the nurse provide to a pregnant patient with mild preeclampsia? 1 "You need to be hospitalized for fetal evaluation." 2 "Nonstress testing can be done once every month." 3 "Fetal movement counts need to be evaluated daily." 4 "Take complete bed rest during the entire pregnancy."

3 Preeclampsia can affect the fetus and may cause fetal growth restrictions, decreased amniotic fluid volume, abnormal fetal oxygenation, low birth weight, and preterm birth. Therefore the fetal movements need to be evaluated daily. Patients with mild preeclampsia can be managed at home effectively and need not be hospitalized. Nonstress testing is performed once or twice per week to determine fetal well-being. Patients need to restrict activity, but complete bed rest is not advised because it may cause cardiovascular deconditioning, muscle atrophy, and psychological stress.

Which intervention will help prevent the risk of pulmonary edema in a pregnant patient with severe preeclampsia? 1 Assess fetal heart rate (FHR) abnormalities regularly. 2 Place the patient on bed rest in a darkened environment. 3 Restrict total intravenous (I.V.) and oral fluids to 125 mL/hr. 4 Ensure that magnesium sulfate is administered as prescribed.

3 Pulmonary edema may be seen in patients with severe preeclampsia. Therefore the nurse needs to restrict total intravenous (I.V.) and oral fluids to 125 mL/hr. FHR monitoring helps assess any fetal complications. The patient is placed on bed rest in a darkened environment to prevent stress. Magnesium sulfate is administered to prevent eclamptic seizures.

During the prenatal assessment of a patient, the nurse teaches the patient about nonpharmacologic pain management. What does the nurse tell the patient about this method? 1 It is technical and expensive. 2 It requires intensive training. 3 It provides the patient with a sense of control. 4 It is used only in stage I of labor.

3 The patient makes choices about the nonpharmacologic pain management methods that are best suited. This provides the patient with a sense of control over childbirth. These measures are relatively simple and inexpensive. They do not require intensive training. However, the patient may obtain best results from the practice. It can be used throughout labor.

Signs of a threatened abortion (miscarriage) are noted in a woman at 8 weeks of gestation. What is an appropriate management approach for this type of abortion? 1 Prepare the woman for a dilation and curettage (D&C). 2 Place the woman on bed rest for at least 1 week and reevaluate. 3 Prepare the woman for an ultrasound and bloodwork. 4 Comfort the woman by telling her that if she loses this baby, she may attempt to get pregnant again in 1 month.

3 Repetitive transvaginal ultrasounds and measurement of human chorionic gonadotropin (hCG) and progesterone levels may be performed to determine if the fetus is alive and within the uterus. If the pregnancy is lost , the woman should be guided through the grieving process. D&C is not considered until signs of the progress to an inevitable abortion are noted or the contents are expelled and incomplete. Bed rest is recommended for 48 hours initially. Telling the woman that she can get pregnant again soon is not a therapeutic response because it discounts the importance of this pregnancy.

During pregnancy, alcohol withdrawal may be treated using: 1 disulfiram (Antabuse). 2 corticosteroids. 3 benzodiazepines. 4 aminophylline.

3 Symptoms that occur during alcohol withdrawal can be managed with short-acting barbiturates or benzodiazepines. Disulfiram is contraindicated in pregnancy because it is teratogenic. Corticosteroids are not used to treat alcohol withdrawal. Aminophylline is not used to treat alcohol withdrawal. Test-Taking Tip: Identify option components as correct or incorrect. This may help you identify a wrong answer. Example: If you are being asked to identify a diet that is specific to a certain condition, your knowledge about that condition would help you choose the correct response (e.g., cholecystectomy = low-fat, high-protein, low-calorie diet).

Which condition should the nurse be alert for after administering terbutaline (Brethine) to a pregnant patient with diabetes mellitus? 1 Dyspnea 2 Infection 3 Ketoacidosis 4 Hypoglycemia

3 Terbutaline (Brethine) is a beta-mimetic drug administered for tocolysis to stop preterm labor. It may lead to hyperglycemia and cause ketoacidosis in the pregnant patient. Dyspnea on exertion may be seen in a pregnant patient with acquired cardiac disease. Infection in pregnant women happens because of an alteration in the normal resistance of the body to infection. Hypoglycemia occurs if there is an increase in the insulin levels.

A labor and delivery nurse is in the process of admitting a patient who is 39 and at 5 weeks' gestation with a diagnosis of preeclampsia. The nurse has evaluated vital signs, weight, and deep tendon reflexes. Although the presence of edema is no longer included in the definition of preeclampsia, it is an important component of the nurse's evaluation. Edema is assessed for distribution, degree and pitting. Although the amount of edema is difficult to quantify, it is important to record the relative degrees of edema formation. From the graphic below, please select the illustration that best displays +3 edema. 1. A 2. B 3. C 4. D

3 The graphic illustrates a depth of 6 mm when the nurse applies finger pressure to the swollen area. This measurement indicates a +3 measurement for edema. Upon applying finger pressure, a 2 mm depression would be the equivalent of a +1, a 4 mm depression, a +2, and an 8 mm depression a +4.

A blunt abdominal trauma causes fetal hemorrhage in a pregnant patient. The nurse finds that the patient is Rh negative. What action does the nurse take? 1 Initiate magnesium sulfate per protocol. 2 Administer oxytocin (pitocin). 3 Administer prescribed Rho (D) immunoglobulin. 4 Prepare the patient for magnetic resonance imaging (MRI).

3 The nurse administers the prescribed Rho(D) immunoglobulin to the patient to protect the patient from isoimmunization. The nurse needs to obtain a prescription for magnesium sulfate if there are eclamptic seizures in a patient with preeclampsia. Oxytocin (Pitocin) is administered to prevent bleeding after birth or the evacuation of the uterus. Magnetic resonance imaging (MRI) is used to assess injuries in a patient after trauma.

What instruction should the nurse give to a pregnant patient with hyperthyroidism who often gets fatigued and weak as a result of nervousness and hyperactivity? 1 "Ensure that you wear warm clothes." 2 "Perform aerobic exercises every day." 3 "Become involved in reading or a craft." 4 "Avoid going out in the cold or at night."

3 The nurse advises the patient to engage in quiet activities, such as reading or crafting, to prevent fatigue and weakness. Extreme cold temperature is prevented and warm clothing is suggested if the patient has cold intolerance because of hypothyroidism. The patient is hypersensitive to heat and gets easily fatigued; therefore aerobic exercises are not advised.

A patient is diagnosed with severe cardiac disease in the fourth week of pregnancy. What should be the nurse's priority? 1 Initiate fetal surveillance twice a week. 2 Assess the patient for signs of heart failure. 3 Explain the risks of continuing pregnancy. 4 Tell the patient to end the pregnancy right away.

3 The nurse explains the risks of continuing the pregnancy to help the patient make an informed decision. Fetal surveillance begins after 28 weeks if there are any complications, such as diabetes mellitus, in the patient. The nurse informs the patient about the complications that may arise as the pregnancy progresses. If the pregnancy is continued, the nurse assesses for cardiac decompensation to evaluate if the heart is able to maintain a sufficient cardiac output. The nurse does not ask the patient to terminate the pregnancy.

What intervention does the nurse perform to provide a relaxed environment for labor? 1 Stand at the bedside. 2 Encourage rapid birth. 3 Control sensory stimuli. 4 Demonstrate excitement

3 The nurse must assist the patient by providing a quiet and relaxed environment. A relaxed environment for labor is created by controlling sensory stimuli, such as light, noise, and temperature, as per the patient's preferences. The nurse must provide reassurance and comfort by sitting rather than standing at the bedside whenever possible. The nurse must not encourage or hurry the patient for rapid birth. The nurse must maintain a calm and unhurried attitude when caring for the patient. Test-Taking Tip: Be aware that information from previously asked questions may help you respond to other examination questions.

During the second phase of labor the patient initiates pattern-paced breathing. What adverse symptoms must the nurse watch for when the patient initiates this method? 1 Pallor 2 Nausea 3 Dizziness 4 Diaphoresis

3 The nurse must watch for symptoms of hyperventilation and resulting respiratory alkalosis. Symptoms of respiratory alkalosis during pattern-paced breathing include dizziness, light-headedness, tingling of fingers, or circumoral numbness. Pallor, nausea, and diaphoresis are generally observed in the active and transition phases of the first stage of labor. They are physiologic effects of pain.

A nurse caring for a woman in labor understands that increased variability of the fetal heart rate might be caused by: 1 narcotics. 2 barbiturates. 3 methamphetamines. 4 tranquilizers.

3 The use of illicit drugs (such as cocaine or methamphetamines) might cause increased variability . Maternal ingestion of narcotics may be the cause of decreased variability. The use of barbiturates may also result in a significant decrease in variability as these are known to cross the placental barrier. Tranquilizer use is a possible cause of decreased variability in the fetal heart rate.

Fetal monitoring of a pregnant patient revealed that the fetal heart rate has minimal variability. Which prescribed drug is most likely responsible for the condition? 1 Hydroxyzine (Vistaril) 2 Terbutaline (Brethine) 3 Secobarbital (Seconal) 4 Atropine (Sal-Tropine)

3 Variability in the fetal heart rate can be classified as absent, mild, or moderate variability. This results in hypoxia and metabolic acidemia in the fetus. Central nervous system (CNS) depressants, such as secobarbital (Seconal), cause variability in the fetal heart rate. This medication affects the baseline heart rate in the fetus by less than 5 beats/min. Hydroxyzine (Vistaril), terbutaline (Brethine), and atropine (Sal-Tropine) may result in tachycardia in the fetus. These drugs can increase the baseline fetal heart rate as much as 25 beats/min.

The nurse administers an amnioinfusion to a pregnant patient according to the primary health care provider's (PHP's) instructions. What is the reason behind the PHP's instructions? 1 Late decelerations 2 Early decelerations 3 Variable decelerations 4 Prolonged decelerations

3 Variable decelerations in the fetal heart rate (FHR) are observed when the umbilical cord is compressed. An amnioinfusion refers to the infusion of isotonic fluid into the uterine cavity when the amniotic fluid levels are decreased. This intervention is usually done for the prevention of umbilical cord compression. Late decelerations are observed when infections or elevated uterine contractions (UCs) are seen in a patient. This condition will be reversed by maintaining an I.V. solution, but aminoinfusion is not administered. Early deceleration in the FHR is a normal sign that does not require any intervention. Prolonged deceleration of the FHR occurs when there is a marked reduction of the fetal oxygen supply.

What care must the nurse take when assisting a laboring patient with hydrotherapy? 1 Initiate hydrotherapy in the first stage of labor at 3 cm. 2 Ensure water is warm at 32.5° to 34° C (90.5° to 93.2°F). 3 Check the fetal heart rate (FHR) with internal electrodes. 4 Obtain the approval of the primary health care provider.

4 Agency policy must be consulted to determine if the approval of the laboring woman's primary health care provider is required. The nurse must ensure that all criteria are met in terms of the status of the maternal and fetal unit. Hydrotherapy is usually initiated when the patient is in active labor, at approximately 5 cm. This reduces the risk of a prolonged labor. FHR monitoring is done by Doppler, fetoscope, or wireless external monitor when hydrotherapy is in use. Use of internal electrodes for monitoring FHR is contraindicated in jet hydrotherapy. The temperature of the water should be maintained at 36° to 37° C (96.8° to 98.6° F). Test-Taking Tip: The night before the examination you may wish to review some key concepts that you believe need additional time, but then relax and get a good night's sleep. Remember to set your alarm, allowing yourself plenty of time to dress comfortably (preferably in layers, depending on the weather), have a good breakfast, and arrive at the testing site at least 15 to 30 minutes early.

The nurse is teaching a group of nursing students regarding fetal oxygenation. The nurse questions a student, "What happens when oxytocin levels are elevated in the patient?" What would be the most appropriate answer given by the nursing student related to the patient's condition? 1 "Hemoglobin levels will decrease." 2 "Blood glucose levels will increase." 3 "Placenta lowers the blood supply." 4 "Uterine contractions (UCs) will increase."

4 An elevated level of oxytocin increases UCs during labor. A reduced hemoglobin level leads to a decreased oxygen supply to the fetus but is not a complication associated with an elevated oxytocin level. Oxytocin has no effect on the blood glucose levels. A family history of diabetes may increase the risk for gestational diabetes in the patient. Conditions such as hypertension in the patient may lower the blood supply to the placenta but are not associated with oxytocin levels.

What should be the first step taken by the nurse when assessing fetal heart activity using an ultrasound transducer? 1 Auscultate the apical heart rate of the pregnant patient. 2 Apply some conductive gel on the maternal abdomen. 3 Apply some conductive gel on the ultrasound transducer. 4 Locate the maximal intensity area of the fetal heart rate.

4 Before the ultrasonic recording, the nurse should first locate the site on the abdomen where the maximal intensity of the fetal heart rate can be assessed. This should be done to find where the ultrasound transducer head can be placed. The apical heart rate of the patient need not be assessed before this procedure, because this procedure does not interfere with the cardiac activity of the pregnant patient. After finding the site of application, the nurse can apply conductive gel on the transducer and on the abdomen of the patient.

Which medication does the primary health care provider ask the nurse to administer to a patient during labor in a vaginal delivery, who has a history of a myocardial infarction (MI)? 1 Oxytocin 2 Diuretics 3 Anticoagulant 4 Epidural analgesia

4 Epidural analgesia is administered during labor to a patient with MI to prevent pain, which can result in tachycardia and increased cardiac demands. Oxytocin is administered to a patient after birth to prevent hemorrhage. Diuretics are administered to prevent fluid retention in a pregnant patient with a heart disease. Anticoagulant therapy is administered for recurrent venous thrombosis in pregnancy. Test-Taking Tip: Notice how the subjects of the questions are related and, through that relationship, the answers to some of the questions you skipped may be provided within other questions of the test.

What does the nurse administer to a patient if there is excessive bleeding after suction curettage? 1 Nifedipine (Procardia) 2 Methyldopa (Aldomet) 3 Hydralazine (Apresoline) 4 Ergonovine (Methergine)

4 Ergonovine (Methergine) is an ergot product, which is administered to contract the uterus when there is excessive bleeding after suction curettage. Nifedipine (Procardia) is prescribed for gestational hypertension or severe preeclampsia. Methyldopa (Aldomet) is an antihypertensive medication indicated for pregnant patients with hypertension. Hydralazine (Apresoline) is also an antihypertensive medication used for treating hypertension intrapartum.

The nurse is caring for a patient in the last trimester of pregnancy. What assessments will the patient display related to the effects of fear and anxiety during labor? An increase in: 1 Blood flow. 2 The progression of labor. 3 Contractions. 4 Muscle tension

4 Fear and excessive anxiety leads to increased muscle tension. It causes more catecholamine secretion. This increases the stimuli to the brain from the pelvis due to increased muscle tension and decreased blood flow. Thus fear and anxiety magnifies the perception of pain. Anxiety does not increase uterine contractions, but reduces the effectiveness of the contractions leading to increased discomfort. This slows the progress of labor. Test-Taking Tip: Do not select answers that contain exceptions to the general rule, controversial material, or degrading responses.

When does the nurse use the fetal scalp stimulation technique to assess the fetal scalp pH? 1 If the patient's contractions have increased 2 If there is maternal weight loss in the last trimester 3 If fetal bradycardia is present 4 When the fetal heart rate (FHR) is within the baseline

4 Fetal scalp and vibroacoustic stimulation are two stimulating methods that are used to determine the fetal scalp blood pH. They are performed only when the fetal baseline heart rate is within the normal range. These techniques are not suggested if there is fetal bradycardia. These stimulation methods are related to neither the patient's weight nor uterine contractions.

A patient reports excessive vomiting in the first trimester of the pregnancy, which has resulted in nutritional deficiency and weight loss. The urinalysis report of the patient indicates ketonuria. Which disorder does the patient have? 1 Preeclampsia 2 Hyperthyroid disorder 3 Gestational hypertension 4 Hyperemesis gravidarum

4 Hyperemesis gravidarum is characterized by excessive vomiting during pregnancy, which causes nutritional deficiency and weight loss. The presence of ketonuria is another indication of this disorder. Preeclampsia refers to hypertension and proteinuria in patients after 20 weeks' gestation. Hyperthyroid disorder may be one of the causes of hyperemesis gravidarum. Gestational hypertension also develops after 20 weeks' gestation.

The nurse observes variable decelerations in fetal heart rate (FHR) while assessing a pregnant patient with oligohydramnios. What medication should be immediately given to the patient? 1 Oxytocin (Pitocin) 2 Terbutaline (Brethine) 3 Phenylephrine (Endal) 4 Lactated Ringer's solution

4 Oligohydramnios is a condition that may cause umbilical cord compression and results in variable decelerations in the FHR. Usually lactated Ringer's or normal saline solution can be administered into the umbilical cord to increase the amniotic fluid volume and normalize fetal heart activity. Terbutaline (Brethine) is a uterine relaxant. It is mostly used to reduce uterine tachysystole. The nurse can administer phenylephrine (Endal) if other measures are unsuccessful in improving maternal hypotension. Oxytocin (Pitocin) is a uterine stimulant to induce labor. It is not used to reduce the umbilical cord compression.

A pregnant woman at 28 weeks of gestation has been diagnosed with gestational diabetes. The nurse caring for this woman understands that: 1 oral hypoglycemic agents should not be used if the woman is reluctant to give herself insulin. 2 dietary modifications and insulin are both required for adequate treatment. 3 glucose levels are monitored by testing urine 4 times a day and at bedtime. 4 dietary management involves distributing nutrient requirements over three meals and two or three snacks.

4 Small frequent meals over a 24-hour period help decrease the risk for hypoglycemia and ketoacidosis. Oral hypoglycemic agents can be used as an alternative to insulin in women with GDM who require medication in addition to diet for blood glucose control. In some women gestational diabetes can be controlled with dietary modifications alone. Blood, not urine, glucose levels are monitored several times a day. Urine is tested for ketone content; results should be negative.

The nurse is assessing a pregnant patient who has been given terbutaline (Brethine). What is the desired outcome from the administration of the drug? 1 Increased fetal accelerations 2 Reduced placental abruption 3 An Apgar score less than 2 4 A cord blood ph result of 7.2

4 Terbutaline (Brethine) is administered during pregnancy, especially during elective cesarean birth. Terbutaline (Brethine) is known to improve the Apgar score of the fetus to more than 5 and the pH value of the cord to 7.2. Terbutaline (Brethine) has no effect on placental integrity or function. Terbutaline (Brethine) does not cause fetal heart rate (FHR) acceleration. The fetal scalp stimulators are used to improve the accelerations.

The nurse has a prescription to obtain a blood sample from a patient to determine fetal lactate levels. What information should the nurse provide to the patient before the procedure? 1 "There is an increased risk for postpartum hemorrhage." 2 "There may be a need to reconduct the diagnostic test." 3 "There is an increased risk for requiring a cesarean birth." 4 "There will be a small incision on the scalp of the newborn."

4 The fetal blood is collected by making a small incision on the fetal scalp, which is visible in the newborn. This might be disturbing to the patient, but the nurse should help the patient understand the purpose of the test. Postpartum hemorrhage or increased risk for cesarean birth is not associated with this procedure. The test has to be conducted only once, and it does not have to be reconducted.

Which is a priority nursing action when a pregnant patient with severe gestational hypertension is admitted to the health care facility? 1 Prepare the patient for cesarean delivery. 2 Administer intravenous (I.V.) and oral fluids. 3 Provide diversionary activities during bed rest. 4 Administer the prescribed magnesium sulfate.

4 The nurse administers the prescribed magnesium sulfate to the patient to prevent eclamptic seizures. I.V. oral fluids are indicated when there is severe dehydration in the patient. It is important to provide diversionary activities during bed rest, but it is secondary in this case. A patient who has experienced a multisystem trauma is prepared for cesarean delivery if there is no evidence of a maternal pulse, which increases the chance of maternal survival.

What does the nurse inform a breastfeeding patient who is taking propylthiouracil (Propacil) for hyperthyroidism? 1 "The medication is likely to decrease milk production." 2 "Stop breastfeeding the child, and start infant formula." 3 "It can adversely affect the neonate's thyroid function." 4 "Take the medication immediately after breastfeeding."

4 The nurse advises the patient to take the medication immediately after breastfeeding to allow a 3- to 4-hour period for the medication to absorb before nursing again. Milk production decreases if there is poor metabolic control, not because of antithyroid medications. It is not necessary to stop breastfeeding or provide infant formula, because there are no side effects of the medication in the infant. The medication also does not adversely affect the neonate's thyroid function because the infant has a normal thyroid function.

It is paramount for the obstetric nurse to understand the regulatory procedures and criteria for admitting a woman to the hospital labor unit. Which guideline is an important legal requirement of maternity care? a. The patient is not considered to be in true labor (according to the Emergency Medical Treatment and Active Labor Act [EMTALA]) until a qualified health care provider says she is. b. The woman can have only her male partner or predesignated "doula" with her at assessment. c. The patient's weight gain is calculated to determine whether she is at greater risk for cephalopelvic disproportion (CPD) and cesarean birth. d. The nurse may exchange information about the patient with family members.

A According to EMTALA, a woman is entitled to active labor care and is presumed to be in "true" labor until a qualified health care provider certifies otherwise. A woman can have anyone she wishes present for her support. The risk for CPD is especially great for petite women or those who have gained 16 kg or more. All patients should have their weight and BMI calculated on admission. This is part of standard nursing care on a maternity unit and not a regulatory concern. According to the Health Insurance Portability and Accountability Act (HIPAA), the patient must give consent for others to receive any information related to her condition.

The nurse providing care for the laboring woman should understand that amnioinfusion is used to treat: a. Variable decelerations. c. Fetal bradycardia. b. Late decelerations. d. Fetal tachycardia.

A Amnioinfusion is used during labor either to dilute meconium-stained amniotic fluid or to supplement the amount of amniotic fluid to reduce the severity of variable decelerations caused by cord compression. Amnioinfusion has no bearing on late decelerations, fetal bradycardia, or fetal tachycardia alterations in fetal heart rate (FHR) tracings.

A woman in preterm labor at 30 weeks of gestation receives two 12-mg doses of betamethasone intramuscularly. The purpose of this pharmacologic treatment is to: a. Stimulate fetal surfactant production. b. Reduce maternal and fetal tachycardia associated with ritodrine administration. c. Suppress uterine contractions. d. Maintain adequate maternal respiratory effort and ventilation during magnesium sulfate therapy.

A Antenatal glucocorticoids given as intramuscular injections to the mother accelerate fetal lung maturity. Inderal would be given to reduce the effects of ritodrine administration. Betamethasone has no effect on uterine contractions. Calcium gluconate would be given to reverse the respiratory depressive effects of magnesium sulfate therapy.

Which occurrence is associated with cervical dilation and effacement? a. Bloody show b. Lightening c. False labor d. Bladder distention

A As the cervix begins to soften, dilate, and efface, expulsion of the mucous plug that sealed the cervix during pregnancy occurs. This causes rupture of small cervical capillaries. Cervical dilation and effacement do not occur with false labor. Lightening is the descent of the fetus toward the pelvic inlet before labor. Bladder distention occurs when the bladder is not emptied frequently. It may slow down the descent of the fetus during labor.

With regard to the process of augmentation of labor, the nurse should be aware that it: a. Is part of the active management of labor that is instituted when the labor process is unsatisfactory. b. Relies on more invasive methods when oxytocin and amniotomy have failed. c. Is a modern management term to cover up the negative connotations of forceps-assisted birth. d. Uses vacuum cups.

A Augmentation is part of the active management of labor that stimulates uterine contractions after labor has started but is not progressing satisfactorily. Augmentation uses amniotomy and oxytocin infusion, as well as some gentler, noninvasive methods. Forceps-assisted births and vacuum-assisted births are appropriately used at the end of labor and are not part of augmentation.

Which patient status is an acceptable indication for serial oxytocin induction of labor? a. Past 42 weeks' gestation c. Polyhydramnios b. Multiple fetuses d. History of long labors

A Continuing a pregnancy past the normal gestational period is likely to be detrimental to fetal health. Multiple fetuses overdistend the uterus and make induction of labor high risk. Polyhydramnios overdistends the uterus, again making induction of labor high risk. History of rapid labors is a reason for induction of labor because of the possibility that the baby would otherwise be born in uncontrolled circumstances.

A normal uterine activity pattern in labor is characterized by: a. Contractions every 2 to 5 minutes. b. Contractions lasting about 2 minutes. c. Contractions about 1 minute apart. d. A contraction intensity of about 1000 mm Hg with relaxation at 50 mm Hg.

A Contractions normally occur every 2 to 5 minutes and last less than 90 seconds (intensity 800 mm Hg) with about 30 seconds in between (20 mm Hg or less).

The nurse would expect which maternal cardiovascular finding during labor? a. Increased cardiac output b. Decreased pulse rate c. Decreased white blood cell (WBC) count d. Decreased blood pressure

A During each contraction, 400 mL of blood is emptied from the uterus into the maternal vascular system. This increases cardiac output by about 51% above baseline pregnancy values at term. The heart rate increases slightly during labor. The WBC count can increase during labor. During the first stage of labor, uterine contractions cause systolic readings to increase by about 10 mm Hg. During the second stage, contractions may cause systolic pressures to increase by 30 mm Hg and diastolic readings to increase by 25 mm Hg.

Which deceleration of the fetal heart rate would not require the nurse to change the maternal position? a. Early decelerations b. Late decelerations c. Variable decelerations d. It is always a good idea to change the woman's position.

A Early decelerations (and accelerations) generally do not need any nursing intervention. Late decelerations suggest that the nurse should change the maternal position (lateral); variable decelerations also require a maternal position change (side to side). Although changing positions throughout labor is recommended, it is not required in response to early decelerations.

The nurse caring for the laboring woman should understand that early decelerations are caused by: a. Altered fetal cerebral blood flow. c. Uteroplacental insufficiency. b. Umbilical cord compression. d. Spontaneous rupture of membranes.

A Early decelerations are the fetus's response to fetal head compression. Variable decelerations are associated with umbilical cord compression. Late decelerations are associated with uteroplacental insufficiency. Spontaneous rupture of membranes has no bearing on the fetal heart rate unless the umbilical cord prolapses, which would result in variable or prolonged bradycardia.

The nurse providing care for the laboring woman comprehends that accelerations with fetal movement: a. Are reassuring. b. Are caused by umbilical cord compression. c. Warrant close observation. d. Are caused by uteroplacental insufficiency.

A Episodic accelerations in the fetal heart rate (FHR) occur during fetal movement and are indications of fetal well-being. Umbilical cord compression results in variable decelerations in the FHR. Accelerations in the FHR are an indication of fetal well-being and do not warrant close observation. Uteroplacental insufficiency would result in late decelerations in the FHR.

Fetal well-being during labor is assessed by: a. The response of the fetal heart rate (FHR) to uterine contractions (UCs). b. Maternal pain control. c. Accelerations in the FHR. d. An FHR above 110 beats/min.

A Fetal well-being during labor can be measured by the response of the FHR to UCs. In general, reassuring FHR patterns are characterized by an FHR baseline in the range of 110 to 160 beats/min with no periodic changes, a moderate baseline variability, and accelerations with fetal movement. Maternal pain control is not the measure used to determine fetal well-being in labor. Although FHR accelerations are a reassuring pattern, they are only one component of the criteria by which fetal well-being is assessed. Although an FHR above 110 beats/min may be reassuring, it is only one component of the criteria by which fetal well-being is assessed. More information would be needed to determine fetal well-being.

Which description of the four stages of labor is correct for both definition and duration? a. First stage: onset of regular uterine contractions to full dilation; less than 1 hour to 20 hours b. Second stage: full effacement to 4 to 5 cm; visible presenting part; 1 to 2 hours c. Third state: active pushing to birth; 20 minutes (multiparous women), 50 minutes (first-timer) d. Fourth stage: delivery of the placenta to recovery; 30 minutes to 1 hour

A Full dilation may occur in less than 1 hour, but in first-time pregnancies it can take up to 20 hours. The second stage extends from full dilation to birth and takes an average of 20 to 50 minutes, although 2 hours is still considered normal. The third stage extends from birth to expulsion of the placenta and usually takes a few minutes. The fourth stage begins after expulsion of the placenta and lasts until homeostasis is reestablished (about 2 hours).

What is an essential part of nursing care for the laboring woman? a. Helping the woman manage the pain b. Eliminating the pain associated with labor c. Sharing personal experiences regarding labor and delivery to decrease her anxiety d. Feeling comfortable with the predictable nature of intrapartum care

A Helping a woman manage the pain is an essential part of nursing care because pain is an expected part of normal labor and cannot be fully relieved. Decreasing anxiety is important; however, managing pain is a top priority. The labor nurse should consistently deliver care based on the standard of care related to the maternity patient. Because of the unpredictable nature of labor, the nurse should always be alert for unanticipated events.

Which presentation is described accurately in terms of both presenting part and frequency of occurrence? a. Cephalic: occiput; at least 95% b. Shoulder: scapula; 10% to 15% c. Breech: sacrum; 10% to 15% d. Cephalic: cranial; 80% to 85%

A In cephalic presentations (head first), the presenting part is the occiput; this occurs in 96% of births. In a breech birth, the sacrum emerges first; this occurs in about 3% of births. In shoulder presentations, the scapula emerges first; this occurs in only 1% of births.

Which maternal condition is considered a contraindication for the application of internal monitoring devices? a. Unruptured membranes b. External monitors in current use c. Cervix dilated to 4 cm d. Fetus with a known heart defect

A In order to apply internal monitoring devices, the membranes must be ruptured. Cervical dilation of 4 cm permits the insertion of fetal scalp electrodes and intrauterine catheter. The external monitor can be discontinued after the internal ones are applied. A compromised fetus should be monitored with the most accurate monitoring devices.

While evaluating an external monitor tracing of a woman in active labor, the nurse notes that the fetal heart rate (FHR) for five sequential contractions begins to decelerate late in the contraction, with the nadir of the decelerations occurring after the peak of the contraction. The nurse's first priority is to: a. Change the woman's position. c. Assist with amnioinfusion. b. Notify the care provider. d. Insert a scalp electrode.

A Late decelerations may be caused by maternal supine hypotension syndrome. They usually are corrected when the woman turns on her side to displace the weight of the gravid uterus from the vena cava. If the fetus does not respond to primary nursing interventions for late decelerations, the nurse would continue with subsequent intrauterine resuscitation measures, including notifying the care provider. An amnioinfusion may be used to relieve pressure on an umbilical cord that has not prolapsed. The FHR pattern associated with this situation most likely reveals variable deceleration. A fetal scalp electrode would provide accurate data for evaluating the well-being of the fetus; however, this is not a nursing intervention that would alleviate late decelerations, nor is it the nurse's first priority.

Leopold maneuvers would be an inappropriate method of assessment to determine: a. Gender of the fetus. b. Number of fetuses. c. Fetal lie and attitude. d. Degree of the presenting part's descent into the pelvis.

A Leopold maneuvers help identify the number of fetuses, the fetal lie and attitude, and the degree of descent of the presenting part into the pelvis. The gender of the fetus is not a goal of the examination at this time.

The nurse caring for a laboring woman is aware that maternal cardiac output can be increased by: a. Change in position. c. Regional anesthesia. b. Oxytocin administration. d. Intravenous analgesic.

A Maternal supine hypotension syndrome is caused by the weight and pressure of the gravid uterus on the ascending vena cava when the woman is in a supine position. This reduces venous return to the woman's heart, as well as cardiac output, and subsequently reduces her blood pressure. The nurse can encourage the woman to change positions and avoid the supine position. Oxytocin administration, regional anesthesia, and intravenous analgesic may reduce maternal cardiac output.

Perinatal nurses are legally responsible for: a. Correctly interpreting fetal heart rate (FHR) patterns, initiating appropriate nursing interventions, and documenting the outcomes. b. Greeting the client on arrival, assessing her, and starting an intravenous line. c. Applying the external fetal monitor and notifying the care provider. d. Making sure that the woman is comfortable.

A Nurses who care for women during childbirth are legally responsible for correctly interpreting FHR patterns, initiating appropriate nursing interventions based on those patterns, and documenting the outcomes of those interventions. Greeting the client, assessing her, and starting an IV; applying the external fetal monitor and notifying the care provider; and making sure the woman is comfortable may be activities that a nurse performs, but they are not activities for which the nurse is legally responsible.

When assessing a multiparous woman who has just given birth to an 8-pound boy, the nurse notes that the woman's fundus is firm and has become globular in shape. A gush of dark red blood comes from her vagina. The nurse concludes that: a. The placenta has separated. b. A cervical tear occurred during the birth. c. The woman is beginning to hemorrhage. d. Clots have formed in the upper uterine segment.

A Placental separation is indicated by a firmly contracting uterus, a change in the uterus from a discoid to a globular ovoid shape, a sudden gush of dark red blood from the introitus, an apparent lengthening of the umbilical cord, and a finding of vaginal fullness. Cervical tears that do not extend to the vagina result in minimal blood loss. Signs of hemorrhage are a boggy uterus, bright red vaginal bleeding, alterations in vital signs, pallor, lightheadedness, restlessness, decreased urinary output, and alteration in the level of consciousness. If clots have formed in the upper uterine segment, the nurse would expect to find the uterus boggy and displaced to the side.

A means of controlling the birth of the fetal head with a vertex presentation is: a. The Ritgen maneuver. b. Fundal pressure. c. The lithotomy position. d. The De Lee apparatus.

A The Ritgen maneuver extends the head during the actual birth and protects the perineum. Gentle, steady pressure against the fundus of the uterus facilitates vaginal birth. The lithotomy position has been commonly used in Western cultures, partly because it is convenient for the health care provider. The De Lee apparatus is used to suction fluid from the infant's mouth.

When using intermittent auscultation (IA) to assess uterine activity, the nurse should be cognizant that: a. The examiner's hand should be placed over the fundus before, during, and after contractions. b. The frequency and duration of contractions is measured in seconds for consistency. c. Contraction intensity is given a judgment number of 1 to 7 by the nurse and client together. d. The resting tone between contractions is described as either placid or turbulent.

A The assessment is done by palpation; duration, frequency, intensity, and resting tone must be assessed. The duration of contractions is measured in seconds; the frequency is measured in minutes. The intensity of contractions usually is described as mild, moderate, or strong. The resting tone usually is characterized as soft or relaxed.

The most critical nursing action in caring for the newborn immediately after birth is: a. Keeping the newborn's airway clear. b. Fostering parent-newborn attachment. c. Drying the newborn and wrapping the infant in a blanket. d. Administering eye drops and vitamin K.

A The care given immediately after the birth focuses on assessing and stabilizing the newborn. Although fostering parent-infant attachment is an important task for the nurse, it is not the most critical nursing action in caring for the newborn immediately after birth. The nursing activities would be (in order of importance) to maintain a patent airway, support respiratory effort, and prevent cold stress by drying the newborn and covering the infant with a warmed blanket or placing the newborn under a radiant warmer. After the newborn has been stabilized, the nurse assesses the newborn's physical condition, weighs and measures the newborn, administers prophylactic eye ointment and a vitamin K injection, affixes an identification bracelet, wraps the newborn in warm blankets, and then gives the infant to the partner or mother when he or she is ready.

To care for a laboring woman adequately, the nurse understands that the __________ stage of labor varies the most in length? a. First b. Third c. Second d. Fourth

A The first stage of labor is considered to last from the onset of regular uterine contractions to full dilation of the cervix. The first stage is much longer than the second and third stages combined. In a first-time pregnancy the first stage of labor can take up to 20 hours. The second stage of labor lasts from the time the cervix is fully dilated to the birth of the fetus. The average length is 20 minutes for a multiparous woman and 50 minutes for a nulliparous woman. The third stage of labor lasts from the birth of the fetus until the placenta is delivered. This stage may be as short as 3 minutes or as long as 1 hour. The fourth stage of labor, recovery, lasts about 2 hours after delivery of the placenta.

In evaluating the effectiveness of oxytocin induction, the nurse would expect: a. Contractions lasting 40 to 90 seconds, 2 to 3 minutes apart. b. The intensity of contractions to be at least 110 to 130 mm Hg. c. Labor to progress at least 2 cm/hr dilation. d. At least 30 mU/min of oxytocin will be needed to achieve cervical dilation.

A The goal of induction of labor would be to produce contractions that occur every 2 to 3 minutes and last 60 to 90 seconds. The intensity of the contractions should be 40 to 90 mm Hg by intrauterine pressure catheter. Cervical dilation of 1 cm/hr in the active phase of labor would be the goal in an oxytocin induction. The dose is increased by 1 to 2 mU/min at intervals of 30 to 60 minutes until the desired contraction pattern is achieved. Doses are increased up to a maximum of 20 to 40 mU/min.

Immediately after the forceps-assisted birth of an infant, the nurse should: a. Assess the infant for signs of trauma. b. Give the infant prophylactic antibiotics. c. Apply a cold pack to the infant's scalp. d. Measure the circumference of the infant's head.

A The infant should be assessed for bruising or abrasions at the site of application, facial palsy, and subdural hematoma. Prophylactic antibiotics are not necessary with a forceps delivery. A cold pack would put the infant at risk for cold stress and is contraindicated. Measuring the circumference of the head is part of the initial nursing assessment.

The exact cause of preterm labor is unknown and believed to be multifactorial. Infection is thought to be a major factor in many preterm labors. Select the type of infection that has not been linked to preterm births. a. Viral c. Cervical b. Periodontal d. Urinary tract

A The infections that increase the risk of preterm labor and birth are all bacterial. They include cervical, urinary tract, periodontal, and other bacterial infections. Therefore, it is important for the client to participate in early, continual, and comprehensive prenatal care. Evidence has shown a link between periodontal infections and preterm labor. Researchers recommend regular dental care before and during pregnancy, oral assessment as a routine part of prenatal care, and scrupulous oral hygiene to prevent infection. Cervical infections of a bacterial nature have been linked to preterm labor and birth. The presence of urinary tract infections increases the risk of preterm labor and birth.

In relation to primary and secondary powers, the maternity nurse comprehends that: a. Primary powers are responsible for effacement and dilation of the cervix. b. Effacement generally is well ahead of dilation in women giving birth for the first time; they are closer together in subsequent pregnancies. c. Scarring of the cervix caused by a previous infection or surgery may make the delivery a bit more painful, but it should not slow or inhibit dilation. d. Pushing in the second stage of labor is more effective if the woman can breathe deeply and control some of her involuntary needs to push, as the nurse directs.

A The primary powers are responsible for dilation and effacement; secondary powers are concerned with expulsion of the fetus. Effacement generally is well ahead of dilation in first-timers; they are closer together in subsequent pregnancies. Scarring of the cervix may slow dilation. Pushing is more effective and less fatiguing when the woman begins to push only after she has the urge to do so.

The nurse knows that proper placement of the tocotransducer for electronic fetal monitoring is located: a. Over the uterine fundus. c. Inside the uterus. b. On the fetal scalp. d. Over the mother's lower abdomen.

A The tocotransducer monitors uterine activity and should be placed over the fundus, where the most intensive uterine contractions occur. The tocotransducer is for external use.

When assessing a woman in the first stage of labor, the nurse recognizes that the most conclusive sign that uterine contractions are effective would be: a. Dilation of the cervix. b. Descent of the fetus. c. Rupture of the amniotic membranes. d. Increase in bloody show.

A The vaginal examination reveals whether the woman is in true labor. Cervical change, especially dilation, in the presence of adequate labor indicates that the woman is in true labor. Descent of the fetus, or engagement, may occur before labor. Rupture of membranes may occur with or without the presence of labor. Bloody show may indicate slow, progressive cervical change (e.g., effacement) in both true and false labor

A pregnant woman's amniotic membranes rupture. Prolapsed umbilical cord is suspected. What intervention would be the top priority? a. Placing the woman in the knee-chest position b. Covering the cord in sterile gauze soaked in saline c. Preparing the woman for a cesarean birth d. Starting oxygen by face mask

A The woman is assisted into a position (e.g., modified Sims position, Trendelenburg position, or the knee-chest position) in which gravity keeps the pressure of the presenting part off the cord. Although covering the cord in sterile gauze soaked saline, preparing the woman for a cesarean, and starting oxygen by face mark are appropriate nursing interventions in the event of a prolapsed cord, the intervention of top priority would be positioning the mother to relieve cord compression.

In assisting with the two factors that have an effect on fetal status (i.e., pushing and positioning), nurses should: a. Encourage the woman's cooperation in avoiding the supine position. b. Advise the woman to avoid the semi-Fowler position. c. Encourage the woman to hold her breath and tighten her abdominal muscles to produce a vaginal response. d. Instruct the woman to open her mouth and close her glottis, letting air escape after the push.

A The woman should maintain a side-lying position. The semi-Fowler position is the recommended side-lying position with a lateral tilt to the uterus. The Valsalva maneuver, which encourages the woman to hold her breath and tighten her abdominal muscles, should be avoided. Both the mouth and glottis should be open, letting air escape during the push.

A laboring woman is lying in the supine position. The most appropriate nursing action at this time is to: a. Ask her to turn to one side. b. Elevate her feet and legs. c. Take her blood pressure. d. Determine whether fetal tachycardia is present.

A The woman's supine position may cause the heavy uterus to compress her inferior vena cava, thus reducing blood return to her heart and reducing placental blood flow. Elevating her legs will not relieve the pressure from the inferior vena cava. If the woman is allowed to stay in the supine position and blood flow to the placental is reduced significantly, fetal tachycardia may occur. The most appropriate nursing action is to prevent this from occurring by turning the woman to her side. Blood pressure readings may be obtained when the patient is in the appropriate and safest position.

A pregnant woman is in her third trimester. She asks the nurse to explain how she can tell true labor from false labor. The nurse would explain that "true" labor contractions: a. Increase with activity such as ambulation. b. Decrease with activity. c. Are always accompanied by the rupture of the bag of waters. d. Alternate between a regular and an irregular pattern.

A True labor contractions become more intense with walking. False labor contractions often stop with walking or position changes. Rupture of membranes may occur before or during labor. True labor contractions are regular.

When managing the care of a woman in the second stage of labor, the nurse uses various measures to enhance the progress of fetal descent. These measures include: a. Encouraging the woman to try various upright positions, including squatting and standing. b. Telling the woman to start pushing as soon as her cervix is fully dilated. c. Continuing an epidural anesthetic so pain is reduced and the woman can relax. d. Coaching the woman to use sustained, 10- to 15-second, closed-glottis bearing-down efforts with each contraction.

A Upright positions and squatting both may enhance the progress of fetal descent. Many factors dictate when a woman will begin pushing. Complete cervical dilation is necessary, but it is only one factor. If the fetal head is still in a higher pelvic station, the physician or midwife may allow the woman to "labor down" (allowing more time for fetal descent, thereby reducing the amount of pushing needed) if the woman is able. The epidural may mask the sensations and muscle control needed for the woman to push effectively. Closed glottic breathing may trigger the Valsalva maneuver, which increases intrathoracic and cardiovascular pressures, reducing cardiac output and inhibiting perfusion of the uterus and placenta. In addition, holding the breath for longer than 5 to 7 seconds diminishes the perfusion of oxygen across the placenta and results in fetal hypoxia.

When planning care for a laboring woman whose membranes have ruptured, the nurse recognizes that the woman's risk for _________________________ has increased. a. Intrauterine infection b. Hemorrhage c. Precipitous labor d. Supine hypotension

A When the membranes rupture, microorganisms from the vagina can ascend into the amniotic sac and cause chorioamnionitis and placentitis. Rupture of membranes (ROM) is not associated with fetal or maternal bleeding. Although ROM may increase the intensity of contractions and facilitate active labor, it does not result in precipitous labor. ROM has no correlation with supine hypotension.

With regard to a woman's intake and output during labor, nurses should be aware that: a. The tradition of restricting the laboring woman to clear liquids and ice chips is being challenged because regional anesthesia is used more often than general anesthesia. b. Intravenous (IV) fluids usually are necessary to ensure that the laboring woman stays hydrated. c. Routine use of an enema empties the rectum and is very helpful for producing a clean, clear delivery. d. When a nulliparous woman experiences the urge to defecate, it often means birth will follow quickly.

A Women are awake with regional anesthesia and are able to protect their own airway, which reduces the worry over aspiration. Routine IV fluids during labor are unlikely to be beneficial and may be harmful. Routine use of an enema is at best ineffective and may be harmful. A multiparous woman may feel the urge to defecate and it may mean birth will follow quickly, but not for a first-timer.

39. Most women with uncomplicated pregnancies can use the nurse as their primary source for nutritional information. The nurse or midwife should refer a client to a registered dietitian for in-depth nutritional counseling in the following situations (Select all that apply). a. Preexisting or gestational illness such as diabetes b. Ethnic or cultural food patterns c. Obesity d. Vegetarian diet e. Allergy to tree nuts

A, B, C, D

Women who have participated in childbirth education classes often bring a "birth bag" or "Lamaze bag" with them to the hospital. These items often assist in reducing stress and providing comfort measures. The nurse caring for women in labor should be aware of common items that a client may bring, including (Select all that apply): a. Rolling pin. b. Tennis balls. c. Pillow. d. Stuffed animal or photo. e. Candles.

A, B, C, D The rolling pin and tennis balls are used to provide counterpressure, especially if the woman is experiencing back labor. Although the facility has plenty of pillows, when the client brings her own, it is a reminder of home and provides added comfort. A stuffed animal or framed photo can be used to provide a focal point during contractions. Although many women find the presence of candles conducive to creating calm and relaxing surroundings, these are not suitable for a hospital birthing room environment. Oxygen may be in use, resulting in a fire hazard. Flameless candles are often sold in hospital gift shops. It is also important for the nurse to orient the patient and her family to the call bell and light switches to familiarize herself with the environment.

Which factors influence cervical dilation (Select all that apply) ? a. Strong uterine contractions b. The force of the presenting fetal part against the cervix c. The size of the female pelvis d. The pressure applied by the amniotic sac e. Scarring of the cervix

A, B, D, E Dilation of the cervix occurs by the drawing upward of the musculofibrous components of the cervix, which is caused by strong uterine contractions. Pressure exerted by the amniotic fluid while the membranes are intact or by the force applied by the presenting part also can promote cervical dilation. Scarring of the cervix as a result of a previous infection or surgery may slow cervical dilation. Pelvic size does not affect cervical dilation.

Signs that precede labor include (Select all that apply): a. Lightening. b. Exhaustion. c. Bloody show. d. Rupture of membranes. e. Decreased fetal movement.

A, C, D Signs that precede labor may include lightening, urinary frequency, backache, weight loss, surge of energy, bloody show, and rupture of membranes. Many women experience a burst of energy before labor. A decrease in fetal movement is an ominous sign that does not always correlate with labor.

Induction of labor is considered an acceptable obstetric procedure if it is in the best interest to deliver the fetus. The charge nurse on the labor and delivery unit is often asked to schedule patients for this procedure and therefore must be cognizant of the specific conditions appropriate for labor induction. These include (Select all that apply): a. Rupture of membranes at or near term. b. Convenience of the woman or her physician. c. Chorioamnionitis (inflammation of the amniotic sac). d. Post-term pregnancy. e. Fetal death.

A, C, D, E These are all acceptable indications for induction. Other conditions include intrauterine growth retardation (IUGR), maternal-fetal blood incompatibility, hypertension, and placental abruption. Elective inductions for the convenience of the woman or her provider are not recommended; however, they have become commonplace. Factors such as rapid labors and living a long distance from a health care facility may be valid reasons in such a circumstance. Elective delivery should not occur before 39 weeks' completed gestation.

Which opiate causes euphoria, relaxation, drowsiness, and detachment from reality and has possible effects on the pregnancy, including preeclampsia, intrauterine growth restriction, and premature rupture of membranes? A. Heroin B. Alcohol C. Phencyclidine palmitate (PCP) D. Cocaine

A. Heroin The opiates include opium, heroin, meperidine, morphine, codeine, and methadone. The signs and symptoms of heroin use are euphoria, relaxation, relief from pain, detachment from reality, impaired judgment, drowsiness, constricted pupils, nausea, constipation, slurred speech, and respiratory depression. Possible effects on pregnancy include preeclampsia, intrauterine growth restriction, miscarriage, premature rupture of membranes, infections, breech presentation, and preterm labor. Alcohol is not an opiate. PCP is not an opiate. Cocaine is not an opiate.

The nurse is caring for a woman with mitral stenosis who is in the active stage. Which action should the nurse take to promote cardiac function? A. Maintain the woman in a side-lying position with the head and shoulders elevated to facilitate hemodynamics B. Prepare the woman for delivery by cesarean section since this is the recommended delivery method to sustain hemodynamics C. Encourage the woman to avoid the use of narcotics or epidural regional analgesia since this alters cardiac function D. Promote the use of the Valsalva maneuver during pushing in the second stage to improve diastolic ventricular filling

A. Maintain the woman in a side-lying position with the head and shoulders elevated to facilitate hemodynamics The side-lying position with the head and shoulders elevated helps to facilitate hemodynamics during labor. A vaginal delivery is the preferred method of delivery for a woman with cardiac disease as it sustains hemodynamics better than a cesarean section. The use of supportive care, medication, and narcotics or epidural regional analgesia is not contraindicated with a woman with heart disease. The use of the Valsalva maneuver during pushing in the second stage should be avoided because it reduces diastolic ventricular filling and obstructs left ventricular outflow.

A nurse is caring for a woman with mitral stenosis who is in the active stage. Which action should the nurse take to promote cardiac function? A. Maintain the woman in a side-lying position with the head and shoulders elevated to facilitate hemodynamics. B. Prepare the woman for delivery by cesarean section because this is the recommended delivery method to sustain hemodynamics. C. Encourage the woman to avoid the use of narcotics or epidural regional analgesia because this alters cardiac function. D. Promote the use of the Valsalva maneuver during pushing in the second stage to improve diastolic ventricular filling.

A. Maintain the woman in a side-lying position with the head and shoulders elevated to facilitate hemodynamics. The side-lying position with the head and shoulders elevated helps facilitate hemodynamics during labor. A vaginal delivery is the preferred method for a woman with cardiac disease because it sustains hemodynamics better than a cesarean section. The use of supportive care, medication, and narcotics or epidural regional analgesia is not contraindicated with a woman with heart disease. Epidural anesthesia for labor is preferred. (Easterling and Stout, 2012). Using the Valsalva maneuver during pushing in the second stage should be avoided because it reduces diastolic ventricular filling and obstructs left ventricular outflow.

A pregnant woman is being examined by the nurse in the outpatient obstetric clinic. The nurse suspects systemic lupus erythematosus (SLE) after revealing which symptoms? (Select all that apply.) A. Muscle aches B. Hyperactivity C. Weight changes D. Fever E. Hypotension

A. Muscle aches C. Weight changes D. Fever Fatigue, rather than hyperactivity is a common sign of SLE. Hypotension is not a characteristic sign of SLE. Common symptoms, including myalgias, fatigue, weight change, and fevers, occur in nearly all women with SLE at some time during the course of the disease. Although a diagnosis of SLE is suspected based on clinical signs and symptoms, it is confirmed by laboratory testing that demonstrates the presence of circulating autoantibodies. As is the case with other autoimmune diseases, SLE is characterized by a series of exacerbations (flares) and remissions (Chin and Branch, 2012).

Thalassemia is a relatively common anemia in which: A. an insufficient amount of hemoglobin is produced to fill the red blood cells (RBCs). B. RBCs have a normal life span but are sickled in shape. C. folate deficiency occurs. D. there are inadequate levels of vitamin B12.

A. an insufficient amount of hemoglobin is produced to fill the red blood cells (RBCs). Thalassemia is a hereditary disorder that involves the abnormal synthesis of the á or â chains of hemoglobin. An insufficient amount of hemoglobin is produced to fill the RBCs. This is the underlying description for sickle cell anemia. Folate deficiency is the most common cause of megaloblastic anemias during pregnancy. B12 deficiency must also be considered if the pregnant woman presents with anemia.

Hypothyroidism occurs in 2 to 3 pregnancies per 1000. Pregnant women with untreated hypothyroidism are at risk for: (Select all that apply.) A. miscarriage. B. macrosomia. C. gestational hypertension. D. placental abruption. E. stillbirth.

A. miscarriage. C. gestational hypertension. D. placental abruption. E. stillbirth. Hypothyroidism is often associated with both infertility and an increased risk of miscarriage. Infants born to mothers with hypothyroidism are more likely to be of low birth weight or preterm. These outcomes can be improved with early diagnosis and treatment. Pregnant women with hypothyroidism are more likely to experience both preeclampsia and gestational hypertension. Placental abruption and stillbirth are risks associated with hypothyroidism. Placental abruption and stillbirth are risks associated with hypothyroidism.

34. A woman is 6 weeks pregnant. She has had a previous spontaneous abortion at 14 weeks of gestation and a pregnancy that ended at 38 weeks with the birth of a stillborn girl. What is her gravidity and parity using the GTPAL system? ___________________

ANS: 3-1-0-1-0 The correct calculation of this woman's gravidity and parity is 3-1-0-1-0. Using the GPTAL system, this client's gravidity and parity information is calculated as follows: G: Total number of times the woman has been pregnant (she is pregnant for the third time) T: Number of pregnancies carried to term (she has had only one pregnancy that resulted in a fetus at term) P: Number of pregnancies that resulted in a preterm birth (none) A: Abortions or miscarriages before the period of viability (she has had one) L: Number of children born who are currently living (she has no living children)

The nurse has formulated a diagnosis of Imbalanced nutrition: Less than body requirements for the client. Which goal is most appropriate for this client to obtain? a.Gain a total of 30 pounds. b.Consistently take daily supplements. c.Decrease her intake of snack foods. d.Increase her intake of complex carbohydrates.

ANS: A A weight gain of 30 pounds is one indication that the client has gained a sufficient amount for the nutritional needs of pregnancy. A daily supplement is not the best goal for this client and does not meet the basic need of proper nutrition during pregnancy. Decreasing snack foods may be needed and should be assessed; however, assessing weight gain is the best method of monitoring nutritional intake for this pregnant client. Although increasing the intake of complex carbohydrates is important for this client, monitoring the weight gain should be the end goal.

Which information regarding protein in the diet of a pregnant woman is most helpful to the client? a.Many protein-rich foods are also good sources of calcium, iron, and B vitamins. b.Many women need to increase their protein intake during pregnancy. c.As with carbohydrates and fat, no specific recommendations exist for the amount of protein in the diet. d.High-protein supplements can be used without risk by women on macrobiotic diets.

ANS: A Good sources for protein, such as meat, milk, eggs, and cheese, have a lot of calcium and iron. Most women already eat a high-protein diet and do not need to increase their intake. Protein is sufficiently important that specific servings of meat and dairy are recommended. High-protein supplements are not recommended because they have been associated with an increased incidence of preterm births.

A pregnant woman reports that she is still playing tennis at 32 weeks of gestation. Which recommendation would the nurse make for this particular client after a tennis match? a.Drink several glasses of fluid. b.Eat extra protein sources such as peanut butter. c.Enjoy salty foods to replace lost sodium. d.Consume easily digested sources of carbohydrate.

ANS: A If no medical or obstetric problems contraindicate physical activity, then pregnant women should get 30 minutes of moderate physical exercise daily. Liberal amounts of fluid should be consumed before, during, and after exercise because dehydration can trigger premature labor. The woman's caloric intake should be sufficient to meet the increased needs of pregnancy and the demands of exercise.

Which guidance might the nurse provide for a client with severe morning sickness? a.Trying lemonade and potato chips b.Drinking plenty of fluids early in the day c.Immediately brushing her teeth after eating d.Never snacking before bedtime

ANS: A Interestingly, some women can tolerate tart or salty foods when they are nauseated. Lemonade and potato chips are an ideal combination. The woman should avoid drinking too much when nausea is most likely, but she should increase her fluid levels later in the day when she feels better. The woman should avoid brushing her teeth immediately after eating. A small snack of cereal and milk or yogurt before bedtime may help the stomach in the morning.

After the nurse completes nutritional counseling for a pregnant woman, she asks the client to repeat the instructions to assess the client's understanding. Which statement indicates that the client understands the role of protein in her pregnancy? a."Protein will help my baby grow." b."Eating protein will prevent me from becoming anemic." c."Eating protein will make my baby have strong teeth after he is born." d."Eating protein will prevent me from being diabetic."

ANS: A Protein is the nutritional element basic to growth. An adequate protein intake is essential to meeting the increasing demands of pregnancy. These demands arise from the rapid growth of the fetus; the enlargement of the uterus, mammary glands, and placenta; the increase in the maternal blood volume; and the formation of the amniotic fluid. Iron intake prevents anemia. Calcium intake is needed for fetal bone and tooth development. Glycemic control is needed in those with diabetes; protein is one nutritional factor to consider for glycemic control but not the primary role of protein intake.

Which nutritional recommendation regarding fluids is accurate? a.A woman's daily intake should be six to eight glasses of water, milk, and/or juice. b.Coffee should be limited to no more than 2 cups, but tea and cocoa can be consumed without worry. c.Of the artificial sweeteners, only aspartame has not been associated with any maternity health concerns. d.Water with fluoride is especially encouraged because it reduces the child's risk of tooth decay.

ANS: A Six to eight glasses is still the standard for fluids; however, they should be the right fluids. All beverages containing caffeine, including tea, cocoa, and some soft drinks, should be avoided or should be consumed only in limited amounts. Artificial sweeteners, including aspartame, have no ill effects on the normal mother or fetus. However, mothers with phenylketonuria (PKU) should avoid aspartame. Although no evidence indicates that prenatal fluoride consumption reduces childhood tooth decay, fluoride still helps the mother.

13. Which behavior indicates that a woman is "seeking safe passage" for herself and her infant? a. She keeps all prenatal appointments. c. She drives her car slowly. b. She "eats for two." d. She wears only low-heeled shoes.

ANS: A The goal of prenatal care is to foster a safe birth for the infant and mother. Although eating properly, driving carefully, and using proper body mechanics all are healthy measures that a mother can take, obtaining prenatal care is the optimal method for providing safety for both herself and her baby

21. As relates to the father's acceptance of the pregnancy and preparation for childbirth, the maternity nurse should know that: a. The father goes through three phases of acceptance of his own. b. The father's attachment to the fetus cannot be as strong as that of the mother because it does not start until after birth. c. In the last 2 months of pregnancy, most expectant fathers suddenly get very protective of their established lifestyle and resist making changes to the home. d. Typically men remain ambivalent about fatherhood right up to the birth of their child.

ANS: A A father typically goes through three phases of development to reach acceptance of fatherhood: the announcement phase, the moratorium phase, and the focusing phase. The father-child attachment can be as strong as the mother-child relationship and can also begin during pregnancy. In the last 2 months of pregnancy, many expectant fathers work hard to improve the environment of the home for the child. Typically the expectant father's ambivalence ends by the first trimester, and he progresses to adjusting to the reality of the situation and then to focusing on his role.

17. Which statement about pregnancy is accurate? a. A normal pregnancy lasts about 10 lunar months. b. A trimester is one third of a year. c. The prenatal period extends from fertilization to conception. d. The estimated date of confinement (EDC) is how long the mother will have to be bedridden after birth.

ANS: A A lunar month lasts 28 days, or 4 weeks. Pregnancy spans 9 calendar months but 10 lunar months. A trimester is one third of a normal pregnancy, or about 13 to 14 weeks. The prenatal period covers the full course of pregnancy (prenatal means before birth). The EDC is now called the EDB, or estimated date of birth. It has nothing to do with the duration of bed rest.

14. A woman who has completed one pregnancy with a fetus (or fetuses) reaching the stage of fetal viability is called a: a. Primipara. c. Multipara. b. Primigravida. d. Nulligravida.

ANS: A A primipara is a woman who has completed one pregnancy with a viable fetus. To remember terms, keep in mind: gravida is a pregnant woman; para comes from parity, meaning a viable fetus; primi means first; multi means many; and null means none. A primigravida is a woman pregnant for the first time. A multipara is a woman who has completed two or more pregnancies with a viable fetus. A nulligravida is a woman who has never been pregnant.

Nursing care measures are commonly offered to women in labor. Which nursing measure reflects application of the gate-control theory? a. Massaging the woman's back b. Changing the woman's position c. Giving the prescribed medication d. Encouraging the woman to rest between contractions

ANS: A According to the gate-control theory, pain sensations travel along sensory nerve pathways to the brain, but only a limited number of sensations, or messages, can travel through these nerve pathways at one time. Distraction techniques such as massage or stroking, music, focal points, and imagery reduce or completely block the capacity of nerve pathways to transmit pain. These distractions are thought to work by closing down a hypothetic gate in the spinal cord and thus preventing pain signals from reaching the brain. The perception of pain is thereby diminished. Changing the woman's position, giving prescribed medication, and encouraging rest do not reduce or block the capacity of nerve pathways to transmit pain using the gate-control theory.

2. A woman at 10 weeks of gestation who is seen in the prenatal clinic with presumptive signs and symptoms of pregnancy likely will have: a. Amenorrhea. c. Chadwick's sign. b. Positive pregnancy test. d. Hegar's sign.

ANS: A Amenorrhea is a presumptive sign of pregnancy. Presumptive signs of pregnancy are felt by the woman. A positive pregnancy test, the presence of Chadwick's sign, and the presence of Hegar's sign all are probable signs of pregnancy.

The nurse providing newborn stabilization must be aware that the primary side effect of maternal narcotic analgesia in the newborn is: a. Respiratory depression. b. Bradycardia. c. Acrocyanosis. d. Tachypnea.

ANS: A An infant delivered within 1 to 4 hours of maternal analgesic administration is at risk for respiratory depression from the sedative effects of the narcotic. Bradycardia is not the anticipated side effect of maternal analgesics. Acrocyanosis is an expected finding in a newborn and is not related to maternal analgesics. The infant who is having a side effect to maternal analgesics normally would have a decrease in respirations, not an increase.

5. During a client's physical examination the nurse notes that the lower uterine segment is soft on palpation. The nurse would document this finding as: a. Hegar's sign c. Chadwick's sign b. McDonald's sign d. Goodell's sign

ANS: A At approximately 6 weeks of gestation, softening and compressibility of the lower uterine segment occur; this is called Hegar's sign. McDonald's sign indicates a fast food restaurant. Chadwick's sign is the blue-violet coloring of the cervix caused by increased vascularity; this occurs around the fourth week of gestation. Softening of the cervical tip is called Goodell's sign, which may be observed around the sixth week of pregnancy.

29. With regard to medications, herbs, shots, and other substances normally encountered by pregnant women, the maternity nurse should be aware that: a. Both prescription and over-the-counter (OTC) drugs that otherwise are harmless can be made hazardous by metabolic deficiencies of the fetus. b. The greatest danger of drug-caused developmental deficits in the fetus is seen in the final trimester. c. Killed-virus vaccines (e.g., tetanus) should not be given during pregnancy, but live-virus vaccines (e.g., measles) are permissible. d. No convincing evidence exists that secondhand smoke is potentially dangerous to the fetus.

ANS: A Both prescription and OTC drugs that otherwise are harmless can be made hazardous by metabolic deficiencies of the fetus. This is especially true for new medications and combinations of drugs. The greatest danger of drug-caused developmental defects exists in the interval from fertilization through the first trimester, when a woman may not realize that she is pregnant. Live-virus vaccines should be part of postpartum care; killed-virus vaccines may be administered during pregnancy. Secondhand smoke is associated with fetal growth restriction and increases in infant mortality.

With regard to nerve block analgesia and anesthesia, nurses should be aware that: a. Most local agents are related chemically to cocaine and end in the suffix -caine. b. Local perineal infiltration anesthesia is effective when epinephrine is added, but it can be injected only once. c. A pudendal nerve block is designed to relieve the pain from uterine contractions. d. A pudendal nerve block, if done correctly, does not significantly lessen the bearing-down reflex.

ANS: A Common agents include lidocaine and chloroprocaine. Injections can be repeated to prolong the anesthesia. A pudendal nerve block relieves pain in the vagina, vulva, and perineum but not the pain from uterine contractions, and it lessens or shuts down the bearing-down reflex.

A woman is experiencing back labor and complains of intense pain in her lower back. An effective relief measure would be to use: a. Counterpressure against the sacrum. b. Pant-blow (breaths and puffs) breathing techniques. c. Effleurage. d. Conscious relaxation or guided imagery.

ANS: A Counterpressure is steady pressure applied by a support person to the sacral area with the fist or heel of the hand. This technique helps the woman cope with the sensations of internal pressure and pain in the lower back. The pain management techniques of pant-blow, effleurage, and conscious relaxation or guided imagery are usually helpful for contractions per the gate-control theory.

10. A woman is in her seventh month of pregnancy. She has been complaining of nasal congestion and occasional epistaxis. The nurse suspects that: a. This is a normal respiratory change in pregnancy caused by elevated levels of estrogen. b. This is an abnormal cardiovascular change, and the nosebleeds are an ominous sign. c. The woman is a victim of domestic violence and is being hit in the face by her partner. d. The woman has been using cocaine intranasally.

ANS: A Elevated levels of estrogen cause capillaries to become engorged in the respiratory tract. This may result in edema in the nose, larynx, trachea, and bronchi. This congestion may cause nasal stuffiness and epistaxis. Cardiovascular changes in pregnancy may cause edema in lower extremities. Determining that the woman is a victim of domestic violence and was hit in the face cannot be made on the basis of the sparse facts provided. If the woman had been hit in the face, she most likely would have additional physical findings. Determination of the use of cocaine by the woman cannot be made on the basis of the sparse facts provided.

With regard to breathing techniques during labor, maternity nurses should understand that: a. Breathing techniques in the first stage of labor are designed to increase the size of the abdominal cavity to reduce friction. b. By the time labor has begun, it is too late for instruction in breathing and relaxation. c. Controlled breathing techniques are most difficult near the end of the second stage of labor. d. The patterned-paced breathing technique can help prevent hyperventilation.

ANS: A First-stage techniques promote relaxation of abdominal muscles, thereby increasing the size of the abdominal cavity. Instruction in simple breathing and relaxation techniques early in labor is possible and effective. Controlled breathing techniques are most difficult in the transition phase at the end of the first stage of labor when the cervix is dilated 8 to 10 cm. Patterned-paced breathing sometimes can lead to hyperventilation.

6. A woman who is 32 weeks' pregnant is informed by the nurse that a danger sign of pregnancy could be: a. Constipation. b. Alteration in the pattern of fetal movement. c. Heart palpitations. d. Edema in the ankles and feet at the end of the day.

ANS: B An alteration in the pattern or amount of fetal movement may indicate fetal jeopardy. Constipation, heart palpitations, and ankle and foot edema are normal discomforts of pregnancy that occur in the second and third trimesters.

A woman in active labor receives an analgesic opioid agonist. Which medication relieves severe, persistent, or recurrent pain; creates a sense of well-being; overcomes inhibitory factors; and may even relax the cervix but should be used cautiously in women with cardiac disease? a. Meperidine (Demerol) b. Promethazine (Phenergan) c. Butorphanol tartrate (Stadol) d. Nalbuphine (Nubain)

ANS: A Meperidine is the most commonly used opioid agonist analgesic for women in labor throughout the world. It overcomes inhibitory factors in labor and may even relax the cervix. Because tachycardia is a possible adverse reaction, meperidine is used cautiously in women with cardiac disease. Phenergan is an ataractic (tranquilizer) that may be used to augment the desirable effects of the opioid analgesics but has few of the undesirable effects of those drugs. Stadol and Nubain are opioid agonist-antagonist analgesics.

Nurses should be aware of the differences experience can make in labor pain such as: a. Sensory pain for nulliparous women often is greater than for multiparous women during early labor. b. Affective pain for nulliparous women usually is less than for multiparous women throughout the first stage of labor. c. Women with a history of substance abuse experience more pain during labor. d. Multiparous women have more fatigue from labor and therefore experience more pain.

ANS: A Sensory pain is greater for nulliparous women because their reproductive tract structures are less supple. Affective pain is greater for nulliparous women during the first stage but decreases for both nulliparous and multiparous during the second stage. Women with a history of substance abuse experience the same amount of pain as those without such a history. Nulliparous women have longer labors and therefore experience more fatigue.

2. Prenatal testing for human immunodeficiency virus (HIV) is recommended for: a. All women, regardless of risk factors. b. A woman who has had more than one sexual partner. c. A woman who has had a sexually transmitted infection. d. A woman who is monogamous with her partner.

ANS: A Testing for the antibody to HIV is strongly recommended for all pregnant women. A HIV test is recommended for all women, regardless of risk factors. Women who test positive for HIV can be treated, reducing the risk of transmission to the fetus.

In assessing a woman for pain and discomfort management during labor, a nurse most likely would: a. Have the woman use a visual analog scale (VAS) to determine her level of pain. b. Note drowsiness as a sign that the medications were working. c. Interpret a woman's fist clenching as an indication that she is angry at her male partner and the physician. d. Evaluate the woman's skin turgor to see whether she needs a gentle oil massage.

ANS: A The VAS is a means of adding the woman's assessment of her pain to the nurse's observations. Drowsiness is a side effect of medications, not usually (sedatives aside) a sign of effectiveness. The fist clenching likely is a sign of apprehension that may need attention. Skin turgor, along with the moistness of the membranes and the concentration of the urine, is a sign that helps the nurse evaluate hydration.

13. Appendicitis may be difficult to diagnose in pregnancy because the appendix is: a. Displaced upward and laterally, high and to the right. b. Displaced upward and laterally, high and to the left. c. Deep at McBurney point. d. Displaced downward and laterally, low and to the right.

ANS: A The appendix is displaced high and to the right, beyond McBurney point.

25. While teaching the expectant mother about personal hygiene during pregnancy, maternity nurses should be aware that: a. Tub bathing is permitted even in late pregnancy unless membranes have ruptured. b. The perineum should be wiped from back to front. c. Bubble bath and bath oils are permissible because they add an extra soothing and cleansing action to the bath. d. Expectant mothers should use specially treated soap to cleanse the nipples.

ANS: A The main danger from taking baths is falling in the tub. The perineum should be wiped from front to back. Bubble baths and bath oils should be avoided because they may irritate the urethra. Soap, alcohol, ointments, and tinctures should not be used to cleanse the nipples because they remove protective oils. Warm water is sufficient.

5. The multiple marker test is used to assess the fetus for which condition? a. Down syndrome c. Congenital cardiac abnormality b. Diaphragmatic hernia d. Anencephaly

ANS: A The maternal serum level of alpha-fetoprotein is used to screen for Down syndrome, neural tube defects, and other chromosome anomalies. The multiple marker test would not detect diaphragmatic hernia, congenital cardiac abnormality, or anencephaly. Additional testing, such as ultrasonography and amniocentesis, would be required to diagnose these conditions

19. The mucous plug that forms in the endocervical canal is called the: a. Operculum. c. Funic souffle. b. Leukorrhea. d. Ballottement.

ANS: A The operculum protects against bacterial invasion. Leukorrhea is the mucus that forms the endocervical plug (the operculum). The funic souffle is the sound of blood flow

To help clients manage discomfort and pain during labor, nurses should be aware that: a. The predominant pain of the first stage of labor is the visceral pain located in the lower portion of the abdomen. b. Referred pain is the extreme discomfort between contractions. c. The somatic pain of the second stage of labor is more generalized and related to fatigue. d. Pain during the third stage is a somewhat milder version of the second stage.

ANS: A This pain comes from cervical changes, distention of the lower uterine segment, and uterine ischemia. Referred pain occurs when the pain that originates in the uterus radiates to the abdominal wall, lumbosacral area of the back, iliac crests, and gluteal area. Second-stage labor pain is intense, sharp, burning, and localized. Third-stage labor pain is similar to that of the first stage.

26. A first-time mother at 18 weeks of gestation comes for her regularly scheduled prenatal visit. The client tells the nurse that she is afraid that she is going into premature labor because she is beginning to have regular contractions. The nurse explains that this is the Braxton Hicks sign and teaches the client that this type of contraction: a. Is painless. c. Causes cervical dilation. b. Increases with walking. d. Impedes oxygen flow to the fetus.

ANS: A Uterine contractions can be felt through the abdominal wall soon after the fourth month of gestation. Braxton Hicks contractions are regular and painless and continue throughout the pregnancy. Although they are not painful, some women complain that they are annoying. Braxton Hicks contractions usually cease with walking or exercise. They can be mistaken for true labor; however, they do not increase in intensity or frequency or cause cervical dilation. In addition, they facilitate uterine blood flow through the intervillous spaces of the placenta and promote oxygen delivery to the fetus.

32. A woman is in for a routine prenatal checkup. You are assessing her urine for proteinuria. Which findings are considered normal (Select all that apply)? a. Dipstick assessment of trace to +1 c. Dipstick assessment of +2 b. <300 mg/24 hours d. >300 mg/24 hours

ANS: A, B Small amounts of protein in the urine are acceptable during pregnancy. The presence of protein in greater amounts may indicate renal problems. A dipstick assessment of +2 and >300 mg/24 hours are excessive amounts of protein in the urine and should be evaluated further.

34. Signs and symptoms that a woman should report immediately to her health care provider include (Select all that apply): a. Vaginal bleeding. b. Rupture of membranes. c. Heartburn accompanied by severe headache. d. Decreased libido. e. Urinary frequency.

ANS: A, B, C Vaginal bleeding, rupture of membranes, and severe headaches all are signs of potential complications in pregnancy. Clients should be advised to report these signs to the health care provider. Decreased libido and urinary frequency are common discomforts of pregnancy that do not require immediate health care interventions.

Most women with uncomplicated pregnancies can use the nurse as their primary source for nutritional information. However, the nurse or midwife may need to refer a client to a registered dietitian for in-depth nutritional counseling. Which conditions would require such a consultation? (Select all that apply.) a.Preexisting or gestational illness such as diabetes b.Ethnic or cultural food patterns c.Obesity d.Vegetarian diets e.Multifetal pregnancy

ANS: A, B, C, D The nurse should be especially aware that conditions such as diabetes can require in-depth dietary planning and evaluation. To prevent issues with hypoglycemia and hyperglycemia, as well as an increased risk for perinatal morbidity and mortality, the client with a preexisting or gestational illness would benefit from a referral to a dietitian. Consultation with a dietitian may ensure that cultural food beliefs are congruent with modern knowledge of fetal development and that adjustments can be made to ensure that all nutritional needs are met. The obese pregnant client may be under the misapprehension that, because of her excess weight, little or no weight gain is necessary. According to the Institute of Medicine, a client with a BMI in the obese range should gain at least 7 kg to ensure a healthy outcome. This client may require in-depth counseling on the optimal food choices. The vegetarian client needs to have her dietary intake carefully assessed to ensure that the optimal combination of amino acids and protein intake is achieved. Very strict vegetarians (vegans) who consume only plant products may also require vitamin B and mineral supplementation. A multifetal pregnancy can be managed by increasing the number of servings of complex carbohydrates and proteins.

While developing an intrapartum care plan for the patient in early labor, it is important that the nurse recognize that psychosocial factors may influence a woman's experience of pain. These include (Select all that apply): a. Culture. b. Anxiety and fear. c. Previous experiences with pain. d. Intervention of caregivers. e. Support systems.

ANS: A, B, C, E Culture: a woman's sociocultural roots influence how she perceives, interprets, and responds to pain during childbirth. Some cultures encourage loud and vigorous expressions of pain, whereas others value self-control. The nurse should avoid praising some behaviors (stoicism) while belittling others (noisy expression). Anxiety and fear: extreme anxiety and fear magnify sensitivity to pain and impair a woman's ability to tolerate it. Anxiety and fear increase muscle tension in the pelvic area, which counters the expulsive forces of uterine contractions and pushing efforts. Previous experiences with pain: fear and withdrawal are a natural response to pain during labor. Learning about these normal sensations ahead of time helps a woman suppress her natural reactions of fear regarding the impending birth. If a woman previously had a long and difficult labor, she is likely to be anxious. She may also have learned ways to cope and may use these skills to adapt to the present labor experience. Support systems: an anxious partner is less able to provide help and support to a woman during labor. A woman's family and friends can be an important source of support if they convey realistic and positive information about labor and delivery. Although the intervention of caregivers may be necessary for the well-being of the woman and her fetus, some interventions add discomfort to the natural pain of labor (i.e., fetal monitor straps, intravenous lines).

35. A woman has just moved to the United States from Mexico. She is 3 months pregnant and has arrived for her first prenatal visit. During her assessment interview, you discover that she has not had any immunizations. Which immunizations should she receive at this point in her pregnancy (Select all that apply)? a. Tetanus b. Diphtheria c. Chickenpox d. Rubella e. Hepatitis B

ANS: A, B, E Immunization with live or attenuated live viruses is contraindicated during pregnancy because of potential teratogenicity. Vaccines consisting of killed viruses may be used. Immunizations that may be administered during pregnancy include tetanus, diphtheria, recombinant hepatitis B, and rabies vaccines. Live-virus vaccines include those for measles (rubeola and rubella), chickenpox, and mumps.

31. The diagnosis of pregnancy is based on which positive signs of pregnancy (Select all that apply)? a. Identification of fetal heartbeat b. Palpation of fetal outline c. Visualization of the fetus d. Verification of fetal movement e. Positive hCG test

ANS: A, C, D Identification of fetal heartbeat, visualization of the fetus, and verification of fetal movement all are positive, objective signs of pregnancy. Palpation of fetal outline and a positive hCG test are probable signs of pregnancy. A tumor also can be palpated. Medication and tumors may lead to false-positive results on pregnancy tests.

The class of drugs known as opioid analgesics (butorphanol, nalbuphine) is not suitable for administration to women with known opioid dependence. The antagonistic activity could precipitate withdrawal symptoms (abstinence syndrome) in both mothers and newborns. Signs of opioid/narcotic withdrawal in the mother would include (Select all that apply): a. Yawning, runny nose. b. Increase in appetite. c. Chills and hot flashes. d. Constipation. e. Irritability, restlessness.

ANS: A, C, E The woman experiencing maternal opioid withdrawal syndrome will exhibit yawning, runny nose, sneezing, anorexia, chills or hot flashes, vomiting, diarrhea, abdominal pain, irritability, restlessness, muscle spasms, weakness, and drowsiness. It is important for the nurse to assess both mother and baby and to plan care accordingly.

A client states that she plans to breastfeed her newborn infant. What guidance would be useful for this new mother? a.The mother's intake of vitamin C, zinc, and protein can now be lower than during pregnancy. b.Caffeine consumed by the mother accumulates in the infant, who may be unusually active and wakeful. c.Critical iron and folic acid levels must be maintained. d.Lactating women can go back to their prepregnant caloric intake.

ANS: B A lactating woman needs to avoid consuming too much caffeine. Vitamin C, zinc, and protein levels need to be moderately higher during lactation than during pregnancy. The recommendations for iron and folic acid are lower during lactation. Lactating women should consume approximately 500 kcal more than their prepregnancy intake, at least 1800 kcal daily overall.

With regard to spinal and epidural (block) anesthesia, nurses should know that: a. This type of anesthesia is commonly used for cesarean births but is not suitable for vaginal births. b. A high incidence of after-birth headache is seen with spinal blocks. c. Epidural blocks allow the woman to move freely. d. Spinal and epidural blocks are never used together.

ANS: B Headaches may be prevented or mitigated to some degree by a number of methods. Spinal blocks may be used for vaginal births, but the woman must be assisted through labor. Epidural blocks limit the woman's ability to move freely. Combined use of spinal and epidural blocks is becoming increasingly popular.

Assessment of a woman's nutritional status includes a diet history, medication regimen, physical examination, and relevant laboratory tests. Which finding might require consultation to a higher level of care? a.Oral contraceptive use may interfere with the absorption of iron. b.Illnesses that have created nutritional deficits, such as PKU, may require nutritional care before conception. c.The woman's socioeconomic status and educational level are not relevant to her examination; they are the province of the social worker. d.Testing for diabetes is the only nutrition-related laboratory test most pregnant women need.

ANS: B A registered dietitian can help with therapeutic diets. Oral contraceptive use may interfere with the absorption of folic acid. Iron deficiency can appear if placement of an intrauterine device (IUD) results in blood loss. A woman's finances can affect her access to good nutrition; her education (or lack thereof) can influence the nurse's teaching decisions. The nutrition-related laboratory test that pregnant women usually need is a screen for anemia.

Which action is the highest priority for the nurse when educating a pregnant adolescent? a.Emphasize the need to eliminate common teenage snack foods because they are high in fat and sodium. b.Determine the weight gain needed to meet adolescent growth, and add 35 pounds. c.Suggest that she not eat at fast-food restaurants to avoid foods of poor nutritional value. d.Realize that most adolescents are unwilling to make dietary changes during pregnancy.

ANS: B Adolescents should gain in the upper range of the recommended weight gain. They also need to gain weight that would be expected for their own normal growth. Changes in the diet should be kept at a minimum. Snack foods can be included in moderation, and other foods can be added to make up for lost nutrients. Eliminating fast foods would make the adolescent appear different to her peers. The client should be taught to choose foods that add needed nutrients. Adolescents are willing to make changes; however, they still have the need to be similar to their peers.

Many clients are concerned about the increased levels of mercury in fish and may be reluctant to include this source of nutrients in their diet. What is the best advice for the nurse to provide? a.Canned white tuna is a preferred choice. b.Shark, swordfish, and mackerel should be avoided. c.Fish caught in local waterways is the safest. d.Salmon and shrimp contain high levels of mercury.

ANS: B As a precaution, the pregnant client should avoid eating shark, swordfish, and mackerel, as well as the less common tilefish. High levels of mercury can harm the developing nervous system of the fetus. Assisting the client in understanding the differences between numerous sources of mercury is essential for the nurse. A pregnant client may eat as much as 12 ounces a week of canned light tuna; however, canned white, albacore, or tuna steaks contain higher levels of mercury and should be limited to no more than 6 ounces per week. Pregnant women and mothers of young children should check with local advisories about the safety of fish caught by families and friends in nearby bodies of water. If no information is available, then these fish sources should be avoided, limited to less than 6 ounces per week, or the only fish consumed that week. Commercially caught fish that is low in mercury includes salmon, shrimp, pollock, or catfish. The pregnant client may eat up to 12 ounces of commercially caught fish per week. Additional information on levels of mercury in commercially caught fish is available at www.cfsan.fda.gov.

To prevent gastrointestinal (GI) upset, when should a pregnant client be instructed to take the recommended iron supplements? a.On a full stomach b.At bedtime c.After eating a meal d.With milk

ANS: B Iron supplements taken at bedtime may reduce GI upset and should be taken at bedtime if abdominal discomfort occurs when iron supplements are taken between meals. Iron supplements are best absorbed if they are taken when the stomach is empty. Bran, tea, coffee, milk, and eggs may reduce absorption.

Which statement made by a lactating woman leads the nurse to believe that the client might have lactose intolerance? a."I always have heartburn after I drink milk." b."If I drink more than a cup of milk, I usually have abdominal cramps and bloating." c."Drinking milk usually makes me break out in hives." d."Sometimes I notice that I have bad breath after I drink a cup of milk."

ANS: B Lactose intolerance, which is an inability to digest milk sugar because of a lack of the enzyme lactose in the small intestine, is a problem that interferes with milk consumption. Milk consumption may cause abdominal cramping, bloating, and diarrhea in such people, although many lactose-intolerant individuals can tolerate small amounts of milk without symptoms. A woman with lactose intolerance is more likely to experience bloating and cramping, not heartburn. A client who breaks out in hives after consuming milk is more likely to have a milk allergy and should be advised to simply brush her teeth after consuming dairy products.

Maternal nutritional status is an especially significant factor of the many that influence the outcome of pregnancy. Why is this the case? a.Maternal nutritional status is extremely difficult to adjust because of an individual's ingrained eating habits. b.Adequate nutrition is an important preventive measure for a variety of problems. c.Women love obsessing about their weight and diets. d.A woman's preconception weight becomes irrelevant.

ANS: B Nutritional status draws so much attention not only for its effect on a healthy pregnancy and birth but also because significant changes are within relatively easy reach. Pregnancy is a time when many women are motivated to learn about adequate nutrition and make changes to their diet that will benefit their baby. Pregnancy is not the time to begin a weight loss diet. Clients and their caregivers should still be concerned with appropriate weight gain.

16. What type of cultural concern is the most likely deterrent to many women seeking prenatal care? a. Religion c. Ignorance b. Modesty d. Belief that physicians are evil

ANS: B A concern for modesty is a deterrent to many women seeking prenatal care. For some women, exposing body parts, especially to a man, is considered a major violation of their modesty. Many cultural variations are found in prenatal care. Even if the prenatal care described is familiar to a woman, some practices may conflict with the beliefs and practices of a subculture group to which she belongs.

The nerve block used in labor that provides anesthesia to the lower vagina and perineum is called: a. An epidural. b. A pudendal. c. A local. d. A spinal block.

ANS: B A pudendal block anesthetizes the lower vagina and perineum to provide anesthesia for an episiotomy and use of low forceps if needed. An epidural provides anesthesia for the uterus, perineum, and legs. A local provides anesthesia for the perineum at the site of the episiotomy. A spinal block provides anesthesia for the uterus, perineum, and down the legs.

23. With regard to the initial physical examination of a woman beginning prenatal care, maternity nurses should be cognizant of: a. Only women who show physical signs or meet the sociologic profile should be assessed for physical abuse. b. The woman should empty her bladder before the pelvic examination is performed. c. The distribution, amount, and quality of body hair are of no particular importance. d. The size of the uterus is discounted in the initial examination.

ANS: B An empty bladder facilitates the examination; this is also an opportunity to get a urine sample easily for a number of tests. All women should be assessed for a history of physical abuse, particularly because the likelihood of abuse increases during pregnancy. Noting body hair is important because body hair reflects nutritional status, endocrine function, and hygiene. Particular attention is paid to the size of the uterus because it is an indication of the duration of gestation.

8. The musculoskeletal system adapts to the changes that occur during pregnancy. A woman can expect to experience what change? a. Her center of gravity will shift backward. b. She will have increased lordosis. c. She will have increased abdominal muscle tone. d. She will notice decreased mobility of her pelvic joints.

ANS: B An increase in the normal lumbosacral curve (lordosis) develops, and a compensatory curvature in the cervicodorsal region develops to help the woman maintain her balance. The center of gravity shifts forward. She will have decreased muscle tone. She will notice increased mobility of her pelvic joints.

9. A 31-year-old woman believes that she may be pregnant. She took an OTC pregnancy test 1 week ago after missing her period; the test was positive. During her assessment interview, the nurse inquires about the woman's last menstrual period and asks whether she is taking any medications. The woman states that she takes medicine for epilepsy. She has been under considerable stress lately at work and has not been sleeping well. She also has a history of irregular periods. Her physical examination does not indicate that she is pregnant. She has an ultrasound scan, which reveals that she is not pregnant. What is the most likely cause of the false-positive pregnancy test result? a. She took the pregnancy test too early. b. She takes anticonvulsants. c. She has a fibroid tumor. d. She has been under considerable stress and has a hormone imbalance.

ANS: B Anticonvulsants may cause false-positive pregnancy test results. OTC pregnancy tests use enzyme-linked immunosorbent assay technology, which can yield positive results 4 days after implantation. Implantation occurs 6 to 10 days after conception. If the woman were pregnant, she would be into her third week at this point (having missed her period 1 week ago). Fibroid tumors do not produce hormones and have no bearing on hCG pregnancy tests. Although stress may interrupt normal hormone cycles (menstrual cycles), it does not affect human chorionic gonadotropin levels or produce positive pregnancy test results.

With regard to a pregnant woman's anxiety and pain experience, nurses should be aware that: a. Even mild anxiety must be treated. b. Severe anxiety increases tension, which increases pain, which in turn increases fear and anxiety, and so on. c. Anxiety may increase the perception of pain, but it does not affect the mechanism of labor. d. Women who have had a painful labor will have learned from the experience and have less anxiety the second time because of increased familiarity.

ANS: B Anxiety and pain reinforce each other in a negative cycle. Mild anxiety is normal for a woman in labor and likely needs no special treatment other than the standard reassurances. Anxiety increases muscle tension and ultimately can build sufficiently to slow the progress of labor. Unfortunately, an anxious, painful first labor is likely to carry over, through expectations and memories, into an anxious and painful experience in the second pregnancy.

After change-of-shift report the nurse assumes care of a multiparous client in labor. The woman is complaining of pain that radiates to her abdominal wall, lower back, and buttocks and down her thighs. Before implementing a plan of care, the nurse should understand that this type of pain is: a. Visceral. b. Referred. c. Somatic. d. Afterpain.

ANS: B As labor progresses the woman often experiences referred pain. This occurs when pain that originates in the uterus radiates to the abdominal wall, the lumbosacral area of the back, the gluteal area, and thighs. The woman usually has pain only during a contraction and is free from pain between contractions. Visceral pain is that which predominates in the first stage of labor. This pain originates from cervical changes, distention of the lower uterine segment, and uterine ischemia. Visceral pain is located over the lower portion of the abdomen. Somatic pain is described as intense, sharp, burning, and well localized. This results from stretching of the perineal tissues and the pelvic floor. This occurs during the second stage of labor. Pain experienced during the third stage of labor or afterward during the early postpartum period is uterine. This pain is very similar to that experienced in the first stage of labor.

To reassure and educate pregnant clients about changes in their cardiovascular system, maternity nurses should be aware that: a. A pregnant woman experiencing disturbed cardiac rhythm, such as sinus arrhythmia requires close medical and obstetric observation, no matter how healthy she otherwise may appear. b. Changes in heart size and position and increases in blood volume create auditory changes from 20 weeks to term. c. Palpitations are twice as likely to occur in twin gestations. d. All of the above changes likely will occur.

ANS: B Auscultatory changes should be discernible after 20 weeks of gestation. A healthy woman with no underlying heart disease does not need any therapy. The maternal heart rate increases in the third trimester, but palpitations may not occur. Auditory changes are discernible at 20 weeks.

It is important for the nurse to develop a realistic birth plan with the pregnant woman in her care. The nurse can explain that a major advantage of nonpharmacologic pain management is: a. Greater and more complete pain relief is possible. b. No side effects or risks to the fetus are involved. c. The woman remains fully alert at all times. d. A more rapid labor is likely.

ANS: B Because nonpharmacologic pain management does not include analgesics, adjunct drugs, or anesthesia, it is harmless to the mother and the fetus. There is less pain relief with nonpharmacologic pain management during childbirth. The woman's alertness is not altered by medication; however, the increase in pain will decrease alertness. Pain management may or may not alter the length of labor. At times when pain is decreased, the mother relaxes and labor progresses at a quicker pace.

6. Cardiovascular system changes occur during pregnancy. Which finding would be considered normal for a woman in her second trimester? a. Less audible heart sounds (S1, S2) b. Increased pulse rate c. Increased blood pressure d. Decreased red blood cell (RBC) production

ANS: B Between 14 and 20 weeks of gestation, the pulse increases about 10 to 15 beats/min, which persists to term. Splitting of S1 and S2 is more audible. In the first trimester, blood pressure usually remains the same as at the prepregnancy level, but it gradually decreases up to about 20 weeks of gestation. During the second trimester, both the systolic and the diastolic pressures decrease by about 5 to 10 mm Hg. Production of RBCs accelerates during pregnancy.

14. A 3-year-old girl's mother is 6 months pregnant. What concern is this child likely to verbalize? a. How the baby will "get out" c. Whether her mother will die b. What the baby will eat d. What color eyes the baby has

ANS: B By age 3 or 4, children like to be told the story of their own beginning and accept its comparison with the present pregnancy. They like to listen to the fetal heartbeat and feel the baby move. Sometimes they worry about how the baby is being fed and what it wears. School-age children take a more clinical interest in their mother's pregnancy and may want to know, "How did the baby get in there?" and "How will it get out?" Whether her mother will die does not tend to be the focus of a child's questions about the impending birth of a sibling. The baby's eye color does not tend to be the focus of children's questions about the impending birth of a sibling.

With regard to systemic analgesics administered during labor, nurses should be aware that: a. Systemic analgesics cross the maternal blood-brain barrier as easily as they do the fetal blood-brain barrier. b. Effects on the fetus and newborn can include decreased alertness and delayed sucking. c. Intramuscular administration (IM) is preferred over intravenous (IV) administration. d. IV patient-controlled analgesia (PCA) results in increased use of an analgesic.

ANS: B Effects depend on the specific drug given, the dosage, and the timing. Systemic analgesics cross the fetal blood-brain barrier more readily than the maternal blood-brain barrier. IV administration is preferred over IM administration because the drug acts faster and more predictably. PCA results in decreased use of an analgesic.

24. To reassure and educate pregnant clients about the functioning of their kidneys in eliminating waste products, maternity nurses should be aware that: a. Increased urinary output makes pregnant women less susceptible to urinary infection. b. Increased bladder sensitivity and then compression of the bladder by the enlarging uterus results in the urge to urinate even if the bladder is almost empty. c. Renal (kidney) function is more efficient when the woman assumes a supine position. d. Using diuretics during pregnancy can help keep kidney function regular.

ANS: B First bladder sensitivity and then compression of the bladder by the uterus result in the urge to urinate more often. Numerous anatomic changes make a pregnant woman more susceptible to urinary tract infection. Renal function is more efficient when the woman lies in the lateral recumbent position and less efficient when she is supine. Diuretic use during pregnancy can overstress the system and cause problems.

A woman is at 14 weeks of gestation. The nurse would expect to palpate the fundus at which level? a. Not palpable above the symphysis at this time b. Slightly above the symphysis pubis c. At the level of the umbilicus d. Slightly above the umbilicus

ANS: B In normal pregnancies, the uterus grows at a predictable rate. It may be palpated above the symphysis pubis sometime between the twelfth and fourteenth weeks of pregnancy. As the uterus grows, it may be palpated above the symphysis pubis sometime between the twelfth and fourteenth weeks of pregnancy. The uterus rises gradually to the level of the umbilicus at 22 to 24 weeks of gestation.

12. During the first trimester, a woman can expect which of the following changes in her sexual desire? a. An increase, because of enlarging breasts b. A decrease, because of nausea and fatigue c. No change d. An increase, because of increased levels of female hormones

ANS: B Maternal physiologic changes such as breast enlargement, nausea, fatigue, abdominal changes, perineal enlargement, leukorrhea, pelvic vasocongestion, and orgasmic responses may affect sexuality and sexual expression. Libido may be depressed in the first trimester but often increases during the second and third trimesters. During pregnancy, the breasts may become enlarged and tender; this tends to interfere with coitus, decreasing the desire to engage in sexual activity.

18. In understanding and guiding a woman through her acceptance of pregnancy, a maternity nurse should be aware that: a. Nonacceptance of the pregnancy very often equates to rejection of the child. b. Mood swings most likely are the result of worries about finances and a changed lifestyle as well as profound hormonal changes. c. Ambivalent feelings during pregnancy usually are seen only in emotionally immature or very young mothers. d. Conflicts such as not wanting to be pregnant or childrearing and career-related decisions need not be addressed during pregnancy because they will resolve themselves naturally after birth.

ANS: B Mood swings are natural and are likely to affect every woman to some degree. A woman may dislike being pregnant, refuse to accept it, and still love and accept the child. Ambivalent feelings about pregnancy are normal for mature or immature women, younger or older women. Conflicts such as not wanting to be pregnant or childrearing and career-related decisions need to be resolved. The baby ends the pregnancy but not all the issues.

30. A patient in her first trimester complains of nausea and vomiting. She asks, "Why does this happen?" The nurse's best response is: a. "It is due to an increase in gastric motility." b. "It may be due to changes in hormones." c. "It is related to an increase in glucose levels." d. "It is caused by a decrease in gastric secretions."

ANS: B Nausea and vomiting are believed to be caused by increased levels of hormones, decreased gastric motility, and hypoglycemia. Gastric motility decreases during pregnancy. Glucose levels decrease in the first trimester. Although gastric secretions decrease, this is not the main cause of nausea and vomiting.

The role of the nurse with regard to informed consent is to: a. Inform the client about the procedure and have her sign the consent form. b. Act as a client advocate and help clarify the procedure and the options. c. Call the physician to see the client. d. Witness the signing of the consent form.

ANS: B Nurses play a part in the informed consent process by clarifying and describing procedures or by acting as the woman's advocate and asking the primary health care provider for further explanations. The physician is responsible for informing the woman of her options, explaining the procedure, and advising the client about potential risk factors. The physician must be present to explain the procedure to the client. However, the nurse's responsibilities go further than simply asking the physician to see the client. The nurse may witness the signing of the consent form. However, depending on the state's guidelines, the woman's husband or another hospital health care employee may sign as witness.

The laboring woman who imagines her body opening to let the baby out is using a mental technique called: a. Dissociation. b. Effleurage. c. Imagery. d. Distraction.

ANS: C Imagery is a technique of visualizing images that will assist the woman in coping with labor. Dissociation helps the woman learn to relax all muscles except those that are working. Effleurage is self-massage. Distraction can be used in the early latent phase by having the woman engage in another activity.

8. A pregnant woman at 18 weeks of gestation calls the clinic to report that she has been experiencing occasional backaches of mild-to-moderate intensity. The nurse would recommend that she: a. Do Kegel exercises. c. Use a softer mattress. b. Do pelvic rock exercises. d. Stay in bed for 24 hours.

ANS: B Pelvic rock exercises may help stretch and strengthen the abdominal and lower back muscles and relieve low back pain. Kegel exercises increase the tone of the pelvic area, not the back. A softer mattress may not provide the support needed to maintain proper alignment of the spine and may contribute to back pain. Stretching and other exercises to relieve back pain should be performed several times a day.

3. The nurse teaches a pregnant woman about the presumptive, probable, and positive signs of pregnancy. The woman demonstrates understanding of the nurse's instructions if she states that a positive sign of pregnancy is: a. A positive pregnancy test. b. Fetal movement palpated by the nurse-midwife. c. Braxton Hicks contractions. d. Quickening.

ANS: B Positive signs of pregnancy are attributed to the presence of a fetus, such as hearing the fetal heartbeat or palpating fetal movement. A positive pregnancy test and Braxton Hicks contractions are probable signs of pregnancy. Quickening is a presumptive sign of pregnancy.

1. The nurse caring for a newly pregnant woman would advise her that ideally prenatal care should begin: a. Before the first missed menstrual period. b. After the first missed menstrual period. c. After the second missed menstrual period. d. After the third missed menstrual period.

ANS: B Prenatal care ideally should begin soon after the first missed menstrual period. Regular prenatal visits offer opportunities to ensure the health of the expectant mother and her infant.

A first-time mother is concerned about the type of medications she will receive during labor. She is in a fair amount of pain and is nauseous. In addition, she appears to be very anxious. You explain that opioid analgesics often are used with sedatives because: a. "The two together work the best for you and your baby." b. "Sedatives help the opioid work better, and they also will assist you to relax and relieve your nausea." c. "They work better together so you can sleep until you have the baby." d. "This is what the doctor has ordered for you."

ANS: B Sedatives can be used to reduce the nausea and vomiting that often accompany opioid use. In addition, some ataractics reduce anxiety and apprehension and potentiate the opioid analgesic affects. A potentiator may cause the two drugs to work together more effectively, but it does not ensure maternal or fetal complications will not occur. Sedation may be a related effect of some ataractics, but it is not the goal. Furthermore, a woman is unlikely to be able to sleep through transitional labor and birth. "This is what the doctor has ordered for you" may be true, but it is not an acceptable comment for the nurse to make.

Which finding in the urine analysis of a pregnant woman is considered a variation of normal? a. Proteinuria c. Bacteria in the urine. b. Glycosuria d. Ketones in the urine.

ANS: B Small amounts of glucose may indicate "physiologic spilling." The presence of protein could indicate kidney disease or preeclampsia. Urinary tract infections are associated with bacteria in the urine. An increase in ketones indicates that the patient is exercising too strenuously or has an inadequate fluid and food intake

15. Which time-based description of a stage of development in pregnancy is accurate? a. Viability—22 to 37 weeks since the last menstrual period (LMP) (assuming a fetal weight >500 g) b. Term—pregnancy from the beginning of week 38 of gestation to the end of week 42 c. Preterm—pregnancy from 20 to 28 weeks d. Postdate—pregnancy that extends beyond 38 weeks

ANS: B Term is 38 to 42 weeks of gestation. Viability is the ability of the fetus to live outside the uterus before coming to term, or 22 to 24 weeks since LMP. Preterm is 20 to 37 weeks of gestation. Postdate or postterm is a pregnancy that extends beyond 42 weeks or what is considered the limit of full term.

The nurse should be aware that an effective plan to achieve adequate pain relief without maternal risk is most effective if: a. The mother gives birth without any analgesic or anesthetic. b. The mother and family's priorities and preferences are incorporated into the plan. c. The primary health care provider decides the best pain relief for the mother and family. d. The nurse informs the family of all alternative methods of pain relief available in the hospital setting.

ANS: B The assessment of the woman, her fetus, and her labor is a joint effort of the nurse and the primary health care providers, who consult with the woman about their findings and recommendations. The needs of each woman are different, and many factors must be considered before a decision is made whether pharmacologic methods, nonpharmacologic methods, or a combination of the two will be used to manage labor pain.

A patient at 24 weeks of gestation contacts the nurse at her obstetric provider's office to complain that she has cravings for dirt and gravel. The nurse is aware that this condition is known as ________ and may indicate anemia. a. Ptyalism c. Pica b. Pyrosis d. Decreased peristalsis

ANS: C Pica (a desire to eat nonfood substances) is an indication of iron deficiency and should be evaluated. Ptyalism (excessive salivation), pyrosis (heartburn), and decreased peristalsis are normal findings of gastrointestinal change during pregnancy. Food cravings during pregnancy are normal.

1. A woman's obstetric history indicates that she is pregnant for the fourth time and all of her children from previous pregnancies are living. One was born at 39 weeks of gestation, twins were born at 34 weeks of gestation, and another child was born at 35 weeks of gestation. What is her gravidity and parity using the GTPAL system? a. 3-1-1-1-3 c. 3-0-3-0-3 b. 4-1-2-0-4 d. 4-2-1-0-3

ANS: B The correct calculation of this woman's gravidity and parity is 4-1-2-0-4. The numbers reflect the woman's gravidity and parity information. Using the GPTAL system, her information is calculated as: G: The first number reflects the total number of times the woman has been pregnant; she is pregnant for the fourth time. T: This number indicates the number of pregnancies carried to term, not the number of deliveries at term; only one of her pregnancies has resulted in a fetus at term. P: This is the number of pregnancies that resulted in a preterm birth; the woman has had two pregnancies in which she delivered preterm. A: This number signifies whether the woman has had any abortions or miscarriages before the period of viability; she has not. L: This number signifies the number of children born that currently are living; the woman has four children.

26. The nurse should have knowledge of the purpose of the pinch test. It is used to: a. Check the sensitivity of the nipples. b. Determine whether the nipple is everted or inverted. c. Calculate the adipose buildup in the abdomen. d. See whether the fetus has become inactive.

ANS: B The pinch test is used to determine whether the nipple is everted or inverted. Nipples must be everted to allow breastfeeding.

28. The maternity nurse understands that vascular volume increases 40% to 60% during pregnancy to: a. Compensate for decreased renal plasma flow. b. Provide adequate perfusion of the placenta. c. Eliminate metabolic wastes of the mother. d. Prevent maternal and fetal dehydration.

ANS: B The primary function of increased vascular volume is to transport oxygen and nutrients to the fetus via the placenta. Renal plasma flow increases during pregnancy. Assisting with pulling metabolic wastes from the fetus for maternal excretion is one purpose of the increased vascular volume.

17. To reassure and educate pregnant clients about changes in the uterus, nurses should be aware that: a. Lightening occurs near the end of the second trimester as the uterus rises into a different position. b. The woman's increased urinary frequency in the first trimester is the result of exaggerated uterine antireflexion caused by softening. c. Braxton Hicks contractions become more painful in the third trimester, particularly if the woman tries to exercise. d. The uterine souffle is the movement of the fetus.

ANS: B The softening of the lower uterine segment is called Hegar's sign. Lightening occurs in the last 2 weeks of pregnancy, when the fetus descends. Braxton Hicks contractions become more defined in the final trimester but are not painful. Walking or exercise usually causes them to stop. The uterine souffle is the sound made by blood in the uterine arteries; it can be heard with a fetal stethoscope.

To assist the woman after delivery of the infant, the nurse knows that the blood patch is used after spinal anesthesia to relieve: a. Hypotension. b. Headache. c. Neonatal respiratory depression. d. Loss of movement.

ANS: B The subarachnoid block may cause a postspinal headache resulting from loss of cerebrospinal fluid from the puncture in the dura. When blood is injected into the epidural space in the area of the dural puncture, it forms a seal over the hole to stop leaking of cerebrospinal fluid. Hypotension is prevented by increasing fluid volume before the procedure. Neonatal respiratory depression is not an expected outcome with spinal anesthesia. Loss of movement is an expected outcome of spinal anesthesia.

If an opioid antagonist is administered to a laboring woman, she should be told that: a. Her pain will decrease. b. Her pain will return. c. She will feel less anxious. d. She will no longer feel the urge to push.

ANS: B The woman should be told that the pain that was relieved by the opioid analgesic will return with administration of the opioid antagonist. Opioid antagonists, such as Narcan, promptly reverse the central nervous system (CNS) depressant effects of opioids. In addition, the antagonist counters the effect of the stress-induced levels of endorphins. An opioid antagonist is especially valuable if labor is more rapid than expected and birth is anticipated when the opioid is at its peak effect.

30. Which statement about multifetal pregnancy is inaccurate? a. The expectant mother often develops anemia because the fetuses have a greater demand for iron. b. Twin pregnancies come to term with the same frequency as single pregnancies. c. The mother should be counseled to increase her nutritional intake and gain more weight. d. Backache and varicose veins often are more pronounced.

ANS: B Twin pregnancies often end in prematurity. Serious efforts should be made to bring the pregnancy to term. A woman with a multifetal pregnancy often develops anemia, suffers more or worse backache, and needs to gain more weight. Counseling is needed to help her adjust to these conditions.

29. Physiologic anemia often occurs during pregnancy as a result of: a. Inadequate intake of iron. b. Dilution of hemoglobin concentration. c. The fetus establishing iron stores. d. Decreased production of erythrocytes.

ANS: B When blood volume expansion is more pronounced and occurs earlier than the increase in red blood cells, the woman has physiologic anemia, which is the result of dilution of hemoglobin concentration rather than inadequate hemoglobin. Inadequate intake of iron may lead to true anemia. There is an increased production of erythrocytes during pregnancy.

Maternal hypotension is a potential side effect of regional anesthesia and analgesia. What nursing interventions could you use to raise the client's blood pressure (Select all that apply)? a. Place the woman in a supine position. b. Place the woman in a lateral position. c. Increase intravenous (IV) fluids. d. Administer oxygen. e. Perform a vaginal examination.

ANS: B, C, D Nursing interventions for maternal hypotension arising from analgesia or anesthesia include turning the woman to a lateral position, increasing IV fluids, administering oxygen via face mask, elevating the woman's legs, notifying the physician, administering an IV vasopressor, and monitoring the maternal and fetal status at least every 5 minutes until these are stable. Placing the client in a supine position would cause venous compression, thereby limiting blood flow to and oxygenation of the placenta and fetus. A sterile vaginal examination has no bearing on maternal blood pressure.

Foodborne illnesses can cause adverse effects for both mother and fetus. The nurse is in an ideal position to evaluate the client's knowledge regarding steps to prevent a foodborne illness. The nurse asks the client to "teach back" the fours simple steps of food preparation. What are they? (Select all that apply.) a.Purchase b.Clean c.Separate d.Cook e.Chill

ANS: B, C, D, E According to the U.S. Food and Drug Administration (2013), the "four simple steps" are: • Clean: Frequently cleanse hands, food preparation surfaces, and utensils. • Separate: Avoid contact among raw meat, fish, or poultry and other foods that will not be cooked before consumption. • Cook: Cook foods to the proper temperature. • Chill: Properly store foods, and promptly refrigerate. DIF: Cognitive Level: Apply REF: p. 361 TOP: Nursing Process: Assessment MSC: Client Needs: Health Promotion and Maintenance

A pregnant woman's diet consists almost entirely of whole grain breads and cereals, fruits, and vegetables. Which dietary requirement is the nurse most concerned about? a.Calcium b.Protein c.Vitamin B12 d.Folic acid

ANS: C A pregnant woman's diet is consistent with that followed by a strict vegetarian (vegan). Vegans consume only plant products. Because vitamin B12 is found in foods of animal origin, this diet is deficient in vitamin B12. Depending on the woman's food choices, a pregnant woman's diet may be adequate in calcium. Protein needs can be sufficiently met by a vegetarian diet. The nurse should be more concerned with the woman's intake of vitamin B12 attributable to her dietary restrictions. Folic acid needs can be met by enriched bread products.

Pregnant adolescents are at greater risk for decreased BMI and "fad" dieting with which condition? a.Obesity b.Gestational diabetes c.Low-birth-weight babies d.High-birth-weight babies

ANS: C Adolescents tend to have lower BMIs. In addition, the fetus and the still-growing mother appear to compete for nutrients. These factors, along with inadequate weight gain, lend themselves to a higher incidence of low-birth-weight babies. Obesity is associated with a higher-than-normal BMI. Unless the teenager has type 1 diabetes, an adolescent with a low BMI is less likely to develop gestational diabetes. High-birth-weight or large-for-gestational age (LGA) babies are most often associated with gestational diabetes.

With regard to weight gain during pregnancy, the nurse should be aware of which important information? a.In pregnancy, the woman's height is not a factor in determining her target weight. b.Obese women may have their health concerns, but their risk of giving birth to a child with major congenital defects is the same as with women of normal weight. c.Women with inadequate weight gain have an increased risk of delivering a preterm infant with intrauterine growth restriction (IUGR). d.Greater than expected weight gain during pregnancy is almost always attributable to old-fashioned overeating.

ANS: C IUGR is associated with women with inadequate weight gain. The primary factor in making a weight gain recommendation is the appropriateness of the prepregnancy weight for the woman's height. Obese women are twice as likely as women of normal weight to give birth to a child with major congenital defects. Overeating is only one of several likely causes.

Which minerals and vitamins are usually recommended as a supplement in a pregnant client's diet? a.Fat-soluble vitamins A and D b.Water-soluble vitamins C and B6 c.Iron and folate d.Calcium and zinc

ANS: C Iron should generally be supplemented, and folic acid supplements are often needed because folate is so important in pregnancy. Fat-soluble vitamins should be supplemented as a medical prescription, as vitamin D might be for lactose-intolerant women. Water-soluble vitamin C is sometimes naturally consumed in excess; vitamin B6 is prescribed only if the woman has a very poor diet; and zinc is sometimes supplemented. Most women get enough calcium.

The labor and delivery nurse is preparing a client who is severely obese (bariatric) for an elective cesarean birth. Which piece of specialized equipment will not likely be needed when providing care for this pregnant woman? a.Extra-long surgical instruments b.Wide surgical table c.Temporal thermometer d.Increased diameter blood pressure cuff

ANS: C Obstetricians today are seeing an increasing number of morbidly obese pregnant women weighing 400, 500, and 600 pounds. To manage their conditions and to meet their logistical needs, a new medical subspecialty,bariatric obstetrics, has arisen. Extra-wide blood pressure cuffs, scales that can accommodate up to 880 pounds, and extra-wide surgical tables designed to hold the weight of these women are used. Special techniques for ultrasound examination and longer surgical instruments for cesarean birth are also required. A temporal thermometer can be used for a pregnant client of any size.

While obtaining a diet history, the nurse might be told that the expectant mother has cravings for ice chips, cornstarch, and baking soda. Which nutritional problem does this behavior indicate? a.Preeclampsia b.Pyrosis c.Pica d.Purging

ANS: C The consumption of foods low in nutritional value or of nonfood substances (e.g., dirt, laundry starch) is called pica. Preeclampsia is a vasospastic disease process encountered after 20 weeks of gestation. Characteristics of preeclampsia include increasing hypertension, proteinuria, and hemoconcentration. Pyrosis is a burning sensation in the epigastric region, otherwise known as heartburn. Purging refers to self-induced vomiting after consuming large quantities of food.

If a client's normal prepregnancy diet contains 45 g of protein daily, how many more grams of protein should she consume per day during pregnancy? a.5 b.10 c.25 d.30

ANS: C The recommended intake of protein for the pregnant woman is 70 g. Therefore, additional protein intakes of 5, 10, or 15 g would be inadequate to meet protein needs during pregnancy. A protein intake of 30 g is more than would be necessary and would add extra calories.

A 27-year-old pregnant woman had a preconceptual body mass index (BMI) of 19. What is this client's total recommended weight gain during pregnancy? a.20 kg (44 lb) b.16 kg (35 lb) c.12.5 kg (27.5 lb) d.10 kg (22 lb)

ANS: C This woman has a normal BMI and should gain 11.5 to 16 kg during her pregnancy. A weight gain of 20 kg (44 lb) is unhealthy for most women; a weight gain of 16 kg (35 lb) is at the high end of the range of weight this woman should gain in her pregnancy; and a weight gain of 10 kg (22 lb) is appropriate for an obese woman. This woman has a normal BMI, which indicates that her weight is average.

A laboring woman received an opioid agonist (meperidine) intravenously 90 minutes before she gave birth. Which medication should be available to reduce the postnatal effects of Demerol on the neonate? a. Fentanyl (Sublimaze) b. Promethazine (Phenergan) c. Naloxone (Narcan) d. Nalbuphine (Nubain)

ANS: C An opioid antagonist can be given to the newborn as one part of the treatment for neonatal narcosis, which is a state of central nervous system (CNS) depression in the newborn produced by an opioid. Opioid antagonists such as naloxone (Narcan) can promptly reverse the CNS depressant effects, especially respiratory depression. Fentanyl, promethazine, and nalbuphine do not act as opioid antagonists to reduce the postnatal effects of Demerol on the neonate. Although meperidine (Demerol) is a low-cost medication and readily available, the use of Demerol in labor has been controversial because of its effects on the neonate.

22. With regard to the initial visit with a client who is beginning prenatal care, nurses should be aware that: a. The first interview is a relaxed, get-acquainted affair in which nurses gather some general impressions. b. If nurses observe handicapping conditions, they should be sensitive and not inquire about them because the client will do that in her own time. c. Nurses should be alert to the appearance of potential parenting problems, such as depression or lack of family support. d. Because of legal complications, nurses should not ask about illegal drug use; that is left to physicians

ANS: C Besides these potential problems, nurses need to be alert to the woman's attitude toward health care. The initial interview needs to be planned, purposeful, and focused on specific content. A lot of ground must be covered. Nurses must be sensitive to special problems, but they do need to inquire because discovering individual needs is important. People with chronic or handicapping conditions forget to mention them because they have adapted to them. Getting information on drug use is important and can be done confidentially. Actual testing for drug use requires the client's consent.

23. Some pregnant clients may complain of changes in their voice and impaired hearing. The nurse can tell these clients that these are common reactions to: a. A decreased estrogen level. b. Displacement of the diaphragm, resulting in thoracic breathing. c. Congestion and swelling, which occur because the upper respiratory tract has become more vascular. d. Increased blood volume.

ANS: C Estrogen levels increase, causing the upper respiratory tract to become more vascular producing swelling and congestion in the nose and ears leading to voice changes and impaired hearing. The diaphragm is displaced, and the volume of blood is increased. However, the main concern is increased estrogen levels.

32. In response to requests by the U.S. Public Health Service for new models of prenatal care, an innovative new approach to prenatal care known as centering pregnancy was developed. Which statement would accurately apply to the centering model of care? a. Group sessions begin with the first prenatal visit. b. At each visit, blood pressure, weight, and urine dipsticks are obtained by the nurse. c. Eight to 12 women are placed in gestational-age cohort groups. d. Outcomes are similar to those of traditional prenatal care.

ANS: C Gestational age cohorts comprise the groups, with approximately 8 to 12 women in each group. This group remains intact throughout the pregnancy. Individual follow-up visits are scheduled as needed. Group sessions begin at 12 to 16 weeks of gestation and end with an early postpartum visit. Before group sessions the client has an individual assessment, physical examination, and history. At the beginning of each group meeting, clients measure their own blood pressure, weight, and urine dips and enter these in their record. Fetal heart rate assessment and fundal height are obtained by the nurse. Results evaluating this approach have been very promising. In a study of adolescent clients, there was a decrease in low-birth-weight infants and an increase in breastfeeding rates.

Nurses with an understanding of cultural differences regarding likely reactions to pain may be better able to help clients. Nurses should know that _____ women may be stoic until late in labor, when they may become vocal and request pain relief. a. Chinese b. Arab or Middle Eastern c. Hispanic d. African-American

ANS: C Hispanic women may be stoic early and more vocal and ready for medications later. Chinese women may not show reactions to pain. Medical interventions must be offered more than once. Arab or Middle Eastern women may be vocal in response to labor pain from the start. They may prefer pain medications. African-American women may express pain openly; use of medications for pain is more likely to vary with the individual.

20. To reassure and educate pregnant clients about changes in their breasts, nurses should be aware that: a. The visibility of blood vessels that form an intertwining blue network indicates full function of Montgomery's tubercles and possibly infection of the tubercles. b. The mammary glands do not develop until 2 weeks before labor. c. Lactation is inhibited until the estrogen level declines after birth. d. Colostrum is the yellowish oily substance used to lubricate the nipples for breastfeeding

ANS: C Lactation is inhibited until after birth. The visible blue network of blood vessels is a normal outgrowth of a richer blood supply. The mammary glands are functionally complete by midpregnancy. Colostrum is a creamy, white-to-yellow premilk fluid that can be expressed from the nipples before birth.

19. With regard to a woman's reordering of personal relationships during pregnancy, the maternity nurse should understand that: a. Because of the special motherhood bond, a woman's relationship with her mother is even more important than with the father of the child. b. Nurses need not get involved in any sexual issues the couple has during pregnancy, particularly if they have trouble communicating them to each other. c. Women usually express two major relationship needs during pregnancy: feeling loved and valued and having the child accepted by the father. d. The woman's sexual desire is likely to be highest in the first trimester because of the excitement and because intercourse is physically easier.

ANS: C Love and support help a woman feel better about her pregnancy. The most important person to the pregnant woman is usually the father. Nurses can facilitate communication between partners about sexual matters if, as is common, they are nervous about expressing their worries and feelings. The second trimester is the time when a woman's sense of well-being, along with certain physical changes, increases her desire for sex. Desire is decreased in the first and third trimesters.

A woman in labor has just received an epidural block. The most important nursing intervention is to: a. Limit parenteral fluids. b. Monitor the fetus for possible tachycardia. c. Monitor the maternal blood pressure for possible hypotension. d. Monitor the maternal pulse for possible bradycardia.

ANS: C The most important nursing intervention for a woman who has received an epidural block is to monitor the maternal blood pressure frequently for signs of hypotension. Intravenous fluids are increased for a woman receiving an epidural, to prevent hypotension. The nurse observes for signs of fetal bradycardia. The nurse monitors for signs of maternal tachycardia secondary to hypotension.

11. The nurse should be aware that the partner's main role in pregnancy is to: a. Provide financial support. b. Protect the pregnant woman from "old wives' tales." c. Support and nurture the pregnant woman. d. Make sure the pregnant woman keeps prenatal appointments.

ANS: C The partner's main role in pregnancy is to nurture the pregnant woman and respond to her feelings of vulnerability. In older societies, the man enacted the ritual couvade. Changing cultural and professional attitudes have encouraged fathers' participation in the birth experience over the past 30 years.

A woman has requested an epidural for her pain. She is 5 cm dilated and 100% effaced. The baby is in a vertex position and is engaged. The nurse increases the woman's intravenous fluid for a preprocedural bolus. She reviews her laboratory values and notes that the woman's hemoglobin is 12 g/dL, hematocrit is 38%, platelets are 67,000, and white blood cells (WBCs) are 12,000/mm3. Which factor would contraindicate an epidural for the woman? a. She is too far dilated. b. She is anemic. c. She has thrombocytopenia. d. She is septic.

ANS: C The platelet count indicates a coagulopathy, specifically, thrombocytopenia (low platelets), which is a contraindication to epidural analgesia/anesthesia. Typically epidural analgesia/anesthesia is used in the laboring woman when a regular labor pattern has been achieved, as evidenced by progressive cervical change. The laboratory values show that the woman's hemoglobin and hematocrit are in the normal range and show a slight increase in the WBC count that is not uncommon in laboring women.

27. To provide the patient with accurate information about dental care during pregnancy, maternity nurses should be aware that: a. Dental care can be dropped from the priority list because the woman has enough to worry about and is getting a lot of calcium anyway. b. Dental surgery, in particular, is contraindicated because of the psychologic stress it engenders. c. If dental treatment is necessary, the woman will be most comfortable with it in the second trimester. d. Dental care interferes with the expectant mother's need to practice conscious relaxation.

ANS: C The second trimester is best for dental treatment because that is when the woman will be able to sit most comfortably in the dental chair. Dental care such as brushing with fluoride toothpaste is especially important during pregnancy because nausea during pregnancy may lead to poor oral hygiene. Emergency dental surgery is permissible, but the mother must clearly understand the risks and benefits. Conscious relaxation is useful, and it may even help the woman get through any dental appointments; it is not a reason to avoid them

24. With regard to follow-up visits for women receiving prenatal care, nurses should be aware that: a. The interview portions become more intensive as the visits become more frequent over the course of the pregnancy. b. Monthly visits are scheduled for the first trimester, every 2 weeks for the second trimester, and weekly for the third trimester. c. During the abdominal examination, the nurse should be alert for supine hypotension. d. For pregnant women, a systolic blood pressure (BP) of 130 and a diastolic BP of 80 is sufficient to be considered hypertensive.

ANS: C The woman lies on her back during the abdominal examination, possibly compressing the vena cava and aorta, which can cause a decrease in blood pressure and a feeling of faintness. The interview portion of follow-up examinations is less extensive than in the initial prenatal visits, during which so much new information must be gathered. Monthly visits are routinely scheduled for the first and second trimesters; visits increase to every 2 weeks at week 28 and to once a week at week 36. For pregnant women hypertension is defined as a systolic BP of 140 or greater and a diastolic BP of 90 or greater.

A woman in labor is breathing into a mouthpiece just before the start of her regular contractions. As she inhales, a valve opens, and gas is released. She continues to inhale the gas slowly and deeply until the contraction starts to subside. When the inhalation stops, the valve closes. This procedure is: a. Not used much anymore. b. Likely to be used in the second stage of labor but not in the first stage. c. An application of nitrous oxide. d. A prelude to cesarean birth.

ANS: C This is an application of nitrous oxide, which could be used in either the first or second stage of labor (or both) as part of the preparation for a vaginal birth. Nitrous oxide is self-administered and found to be very helpful.

4. A pregnant woman at 10 weeks of gestation jogs three or four times per week. She is concerned about the effect of exercise on the fetus. The nurse should inform her: a. "You don't need to modify your exercising any time during your pregnancy." b. "Stop exercising because it will harm the fetus." c. "You may find that you need to modify your exercise to walking later in your pregnancy, around the seventh month." d. "Jogging is too hard on your joints; switch to walking now."

ANS: C Typically running should be replaced with walking around the seventh month of pregnancy. The nurse should inform the woman that she may need to reduce her exercise level as the pregnancy progresses. Physical activity promotes a feeling of well-being in pregnant women. It improves circulation, promotes relaxation and rest, and counteracts boredom. Simple measures should be initiated to prevent injuries, such as warm-up and stretching exercises to prepare the joints for more strenuous exercise.

33. During pregnancy, many changes occur as a direct result of the presence of the fetus. Which of these adaptations meet this criteria? a. Leukorrhea b. Development of the operculum c. Quickening d. Ballottement e. Lightening

ANS: C, D, E Leukorrhea is a white or slightly gray vaginal discharge that develops in response to cervical stimulation by estrogen and progesterone. Quickening is the first recognition of fetal movements or "feeling life." Quickening is often described as a flutter and is felt earlier in multiparous women than in primiparas. Lightening occurs when the fetus begins to descend into the pelvis. This occurs 2 weeks before labor in the nullipara and at the start of labor in the multipara. Mucus fills the cervical canal creating a plug otherwise known as the operculum. The operculum acts as a barrier against bacterial invasion during the pregnancy. Passive movement of the unengaged fetus is referred to as ballottement.

A woman has come to the clinic for preconception counseling because she wants to start trying to get pregnant. Which guidance should she expect to receive? a."Discontinue all contraception now." b."Lose weight so that you can gain more during pregnancy." c."You may take any medications you have been regularly taking." d."Make sure you include adequate folic acid in your diet."

ANS: D A healthy diet before conception is the best way to ensure that adequate nutrients are available for the developing fetus. A woman's folate or folic acid intake is of particular concern in the periconception period. Neural tube defects are more common in infants of women with a poor folic acid intake. Depending on the type of contraception that she has been using, discontinuing all contraception at this time may not be appropriate. Advising this client to lose weight now so that she can gain more during pregnancy is also not appropriate advice. Depending on the type of medications the woman is taking, continuing to take them regularly may not be appropriate.

Which pregnant woman should strictly follow weight gain recommendations during pregnancy? a.Pregnant with twins b.In early adolescence c.Shorter than 62 inches or 157 cm d.Was 20 pounds overweight before pregnancy

ANS: D A weight gain of 5 to 9 kg will provide sufficient nutrients for the fetus. Overweight and obese women should be advised to lose weight before conception to achieve the best pregnancy outcomes. A higher weight gain in twin gestations may help prevent low birth weights. Adolescents need to gain weight toward the higher acceptable range, which provides for their own growth, as well as for fetal growth. In the past, women of short stature were advised to restrict their weight gain; however, evidence to support these guidelines has not been found.

The major source of nutrients in the diet of a pregnant woman should be composed of what? a.Simple sugars b.Fats c.Fiber d.Complex carbohydrates

ANS: D Complex carbohydrates supply the pregnant woman with vitamins, minerals, and fiber. The most common simple carbohydrate is table sugar, which is a source of energy but does not provide any nutrients. Fats provide 9 kcal in each gram, in contrast to carbohydrates and proteins, which provide only 4 kcal in each gram. Fiber is primarily supplied by complex carbohydrates.

A woman in the 34th week of pregnancy reports that she is very uncomfortable because of heartburn. Which recommendation would be appropriate for this client? a.Substitute other calcium sources for milk in her diet. b.Lie down after each meal. c.Reduce the amount of fiber she consumes. d.Eat five small meals daily.

ANS: D Eating small, frequent meals may help with heartburn, nausea, and vomiting. Substituting other calcium sources for milk, lying down after eating, and reducing fiber intake are inappropriate dietary suggestions for all pregnant women and do not alleviate heartburn.

Which vitamins or minerals may lead to congenital malformations of the fetus if taken in excess by the mother? a.Zinc b.Vitamin D c.Folic acid d.Vitamin A

ANS: D If taken in excess, vitamin A causes a number of problems. An analog of vitamin A appears in prescribed acne medications, which must not be taken during pregnancy. Zinc, vitamin D, and folic acid are all vital to good maternity and fetal health and are highly unlikely to be consumed in excess.

A client states that she does not drink milk. Which foods should the nurse encourage this woman to consume in greater amounts to increase her calcium intake? a.Fresh apricots b.Canned clams c.Spaghetti with meat sauce d.Canned sardines

ANS: D Sardines are rich in calcium. Fresh apricots, canned clams, and spaghetti with meat sauce are not high in calcium.

A pregnant woman's diet may not meet her increased need for folates. Which food is a rich source of this nutrient? a.Chicken b.Cheese c.Potatoes d.Green leafy vegetables

ANS: D Sources of folates include green leafy vegetables, whole grains, fruits, liver, dried peas, and beans. Chicken and cheese are excellent sources of protein but are poor sources for folates. Potatoes contain carbohydrates and vitamins and minerals but are poor sources for folates.

Nutrition is an alterable and important preventive measure for a variety of potential problems such as low birth weight and prematurity. While completing the physical assessment of the pregnant client, the nurse is able to evaluate the client's nutritional status by observing a number of physical signs. Which physical sign indicates to the nurse that the client has unmet nutritional needs? a.Normal heart rate, rhythm, and blood pressure b.Bright, clear, and shiny eyes c.Alert and responsive with good endurance d.Edema, tender calves, and tingling

ANS: D The physiologic changes of pregnancy may complicate the interpretation of physical findings. Lower extremity edema often occurs when caloric and protein deficiencies are present; however, edema in the lower extremities may also be a common physical finding during the third trimester. Completing a thorough health history and physical assessment and requesting further laboratory testing, if indicated, are essential for the nurse. The malnourished pregnant client may display rapid heart rate, abnormal rhythm, enlarged heart, and elevated blood pressure. A client receiving adequate nutrition will have bright, shiny eyes with no sores and moist, pink membranes. Pale or red membranes, dryness, infection, dull appearance of the cornea, or blue sclerae are signs of poor nutrition. A client who is alert and responsive with good endurance is well nourished. A listless, cachectic, easily fatigued, and tired presentation would be an indication of a poor nutritional status.

Which action is the first priority for the nurse who is assessing the influence of culture on a client's diet? a.Evaluate the client's weight gain during pregnancy. b.Assess the socioeconomic status of the client. c.Discuss the four food groups with the client. d.Identify the food preferences and methods of food preparation common to the client's culture.

ANS: D Understanding the client's food preferences and how she prepares food will assist the nurse in determining whether the client's culture is adversely affecting her nutritional intake. An evaluation of a client's weight gain during pregnancy should be included for all clients, not only for clients from different cultural backgrounds. The socioeconomic status of the client may alter the nutritional intake but not the cultural influence. Teaching the food groups to the client should come after assessing her food preferences.

31. The phenomenon of someone other than the mother-to-be experiencing pregnancy-like symptoms such as nausea and weight gain applies to the: a. Mother of the pregnant woman. c. Sister of the pregnant woman. b. Couple's teenage daughter. d. Expectant father.

ANS: D An expectant father's experiencing pregnancy-like symptoms is called the couvade syndrome.

The obstetric nurse is preparing the patient for an emergency cesarean birth, with no time to administer spinal anesthesia. The nurse is aware and prepared for the greatest risk of administering general anesthesia to the patient. This risk is: a. Respiratory depression. b. Uterine relaxation. c. Inadequate muscle relaxation. d. Aspiration of stomach contents.

ANS: D Aspiration of acidic gastric contents with possible airway obstruction is a potentially fatal complication of general anesthesia. Respirations can be altered during general anesthesia, and the anesthesiologist will take precautions to maintain proper oxygenation. Uterine relaxation can occur with some anesthesia; however, this can be monitored and prevented. Inadequate muscle relaxation can be improved with medication.

33. While you are assessing the vital signs of a pregnant woman in her third trimester, the patient complains of feeling faint, dizzy, and agitated. Which nursing intervention is appropriate? a. Have the patient stand up and retake her blood pressure. b. Have the patient sit down and hold her arm in a dependent position. c. Have the patient lie supine for 5 minutes and recheck her blood pressure on both arms. d. Have the patient turn to her left side and recheck her blood pressure in 5 minutes.

ANS: D Blood pressure is affected by maternal position during pregnancy. The supine position may cause occlusion of the vena cava and descending aorta. Turning the pregnant woman to a lateral recumbent position alleviates pressure on the blood vessels and quickly corrects supine hypotension. Pressures are significantly higher when the patient is standing. This option causes an increase in systolic and diastolic pressures. The arm should be supported at the same level of the heart. The supine position may cause occlusion of the vena cava and descending aorta, creating hypotension.

22. To reassure and educate their pregnant clients about changes in their blood pressure, maternity nurses should be aware that: a. A blood pressure cuff that is too small produces a reading that is too low; a cuff that is too large produces a reading that is too high. b. Shifting the client's position and changing from arm to arm for different measurements produces the most accurate composite blood pressure reading at each visit. c. The systolic blood pressure increases slightly as pregnancy advances; the diastolic pressure remains constant. d. Compression of the iliac veins and inferior vena cava by the uterus contributes to hemorrhoids in the later stage of term pregnancy.

ANS: D Compression of the iliac veins and inferior vena cava also leads to varicose veins in the legs and vulva. The tightness of a cuff that is too small produces a reading that is too high; similarly the looseness of a cuff that is too large results in a reading that is too low. Because maternal positioning affects readings, blood pressure measurements should be obtained in the same arm and with the woman in the same position. The systolic blood pressure generally remains constant but may decline slightly as pregnancy advances. The diastolic blood pressure first decreases and then gradually increases.

An 18-year-old pregnant woman, gravida 1, is admitted to the labor and birth unit with moderate contractions every 5 minutes that last 40 seconds. The woman states, "My contractions are so strong that I don't know what to do with myself." The nurse should: a. Assess for fetal well-being. b. Encourage the woman to lie on her side. c. Disturb the woman as little as possible. d. Recognize that pain is personalized for each individual.

ANS: D Each woman's pain during childbirth is unique and is influenced by a variety of physiologic, psychosocial, and environmental factors. A critical issue for the nurse is how support can make a difference in the pain of the woman during labor and birth. Assessing for fetal well-being includes no information that would indicate fetal distress or a logical reason to be overly concerned about the well-being of the fetus. The left lateral position is used to alleviate fetal distress, not maternal stress. The nurse has an obligation to provide physical, emotional, and psychosocial care and support to the laboring woman. This client clearly needs support.

In the current practice of childbirth preparation, emphasis is placed on: a. The Dick-Read (natural) childbirth method. b. The Lamaze (psychoprophylactic) method. c. The Bradley (husband-coached) method. d. Having expectant parents attend childbirth preparation in any or no specific method.

ANS: D Encouraging expectant parents to attend childbirth preparation class is most important because preparation increases a woman's confidence and thus her ability to cope with labor and birth. Although still popular, the "method" format of classes is being replaced with other offerings such as Hypnobirthing and Birthing from Within.

Which statement correctly describes the effects of various pain factors? a. Higher prostaglandin levels arising from dysmenorrhea can blunt the pain of childbirth. b. Upright positions in labor increase the pain factor because they cause greater fatigue. c. Women who move around trying different positions are experiencing more pain. d. Levels of pain-mitigating b-endorphins are higher during a spontaneous, natural childbirth.

ANS: D Higher endorphin levels help women tolerate pain and reduce anxiety and irritability. Higher prostaglandin levels correspond to more severe labor pains. Upright positions in labor usually result in improved comfort and less pain. Moving freely to find more comfortable positions is important for reducing pain and muscle tension.

16. Human chorionic gonadotropin (hCG) is an important biochemical marker for pregnancy and the basis for many tests. A maternity nurse should be aware that: a. hCG can be detected 2.5 weeks after conception. b. The hCG level increases gradually and uniformly throughout pregnancy. c. Much lower than normal increases in the level of hCG may indicate a postdate pregnancy. d. A higher than normal level of hCG may indicate an ectopic pregnancy or Down syndrome.

ANS: D Higher levels also could be a sign of multiple gestation. hCG can be detected 7 to 8 days after conception. The hCG level fluctuates during pregnancy: peaking, declining, stabilizing, and increasing again. Abnormally slow increases may indicate impending miscarriage.

18. To reassure and educate pregnant clients about changes in the cervix, vagina, and position of the fetus, nurses should be aware that: a. Because of a number of changes in the cervix, abnormal Papanicolaou (Pap) tests are much easier to evaluate. b. Quickening is a technique of palpating the fetus to engage it in passive movement. c. The deepening color of the vaginal mucosa and cervix (Chadwick's sign) usually appears in the second trimester or later as the vagina prepares to stretch during labor. d. Increased vascularity of the vagina increases sensitivity and may lead to a high degree of arousal, especially in the second trimester.

ANS: D Increased sensitivity and an increased interest in sex sometimes go together. This frequently occurs during the second trimester. Cervical changes make evaluation of abnormal Pap tests more difficult. Quickening is the first recognition of fetal movements by the mother. Ballottement is a technique used to palpate the fetus. Chadwick's sign appears from the sixth to eighth weeks.

Which method of pain management is safest for a gravida 3 para 2 admitted at 8 cm cervical dilation? a. Epidural anesthesia b. Narcotics c. Spinal block d. Breathing and relaxation techniques

ANS: D Nonpharmacologic methods of pain management may be the best option for a woman in advanced labor. It is unlikely that enough time remains to administer epidural or spinal anesthesia. A narcotic given at this time may reach its peak about the time of birth and result in respiratory depression in the newborn.

Which statement about a condition of pregnancy is accurate? a. Insufficient salivation (ptyalism) is caused by increases in estrogen. b. Acid indigestion (pyrosis) begins early but declines throughout pregnancy. c. Hyperthyroidism often develops (temporarily) because hormone production increases. d. Nausea and vomiting rarely have harmful effects on the fetus and may be beneficial.

ANS: D Normal nausea and vomiting rarely produce harmful effects, and nausea and vomiting periods may be less likely to result in miscarriage or preterm labor. Ptyalism is excessive salivation, which may be caused by a decrease in unconscious swallowing or stimulation of the salivary glands. Pyrosis begins in the first trimester and intensifies through the third trimester. Increased hormone production does not lead to hyperthyroidism in pregnant women.

28. When discussing work and travel during pregnancy with a pregnant patient, nurses should instruct them that: a. Women should sit for as long as possible and cross their legs at the knees from time to time for exercise. b. Women should avoid seat belts and shoulder restraints in the car because they press on the fetus. c. Metal detectors at airport security checkpoints can harm the fetus if the woman passes through them a number of times. d. While working or traveling in a car or on a plane, women should arrange to walk around at least every hour or so

ANS: D Periodic walking helps prevent thrombophlebitis. Pregnant women should avoid sitting or standing for long periods and crossing the legs at the knees. Pregnant women must wear lap belts and shoulder restraints. The most common injury to the fetus comes from injury to the mother. Metal detectors at airport security checkpoints do not harm fetuses.

11. The nurse caring for the pregnant client must understand that the hormone essential for maintaining pregnancy is: a. Estrogen. b. Human chorionic gonadotropin (hCG). c. Oxytocin. d. Progesterone.

ANS: D Progesterone is essential for maintaining pregnancy; it does so by relaxing smooth muscles. This reduces uterine activity and prevents miscarriage. Estrogen plays a vital role in pregnancy, but it is not the primary hormone for maintaining pregnancy. hCG levels increase at implantation but decline after 60 to 70 days. Oxytocin stimulates uterine contractions.

3. Which symptom is considered a first-trimester warning sign and should be reported immediately by the pregnant woman to her health care provider? a. Nausea with occasional vomiting c. Urinary frequency b. Fatigue d. Vaginal bleeding

ANS: D Signs and symptoms that must be reported include severe vomiting, fever and chills, burning on urination, diarrhea, abdominal cramping, and vaginal bleeding. These symptoms may be signs of potential complications of the pregnancy. Nausea with occasional vomiting, fatigue, and urinary frequency are normal first-trimester complaints. Although they may be worrisome or annoying to the mother, they usually are not indications of pregnancy problems.

7. Numerous changes in the integumentary system occur during pregnancy. Which change persists after birth? a. Epulis c. Telangiectasia b. Chloasma d. Striae gravidarum

ANS: D Striae gravidarum, or stretch marks, reflect separation within the underlying connective tissue of the skin. They usually fade after birth, although they never disappear completely. An epulis is a red, raised nodule on the gums that bleeds easily. Chloasma, or mask of pregnancy, is a blotchy, brown hyperpigmentation of the skin over the cheeks, nose, and forehead, especially in dark-complexioned pregnant women. Chloasma usually fades after the birth. Telangiectasia, or vascular spiders, are tiny, star-shaped or branchlike, slightly raised, pulsating end-arterioles usually found on the neck, thorax, face, and arms. They occur as a result of elevated levels of circulating estrogen. These usually disappear after birth.

15. In her work with pregnant women of various cultures, a nurse practitioner has observed various practices that seemed strange or unusual. She has learned that cultural rituals and practices during pregnancy seem to have one purpose in common. Which statement best describes that purpose? a. To promote family unity b. To ward off the "evil eye" c. To appease the gods of fertility d. To protect the mother and fetus during pregnancy

ANS: D The purpose of all cultural practices is to protect the mother and fetus during pregnancy. Although many cultures consider pregnancy normal, certain practices are expected of women of all cultures to ensure a good outcome. Cultural prescriptions tell women what to do, and cultural proscriptions establish taboos. The purposes of these practices are to prevent maternal illness resulting from a pregnancy-induced imbalanced state and to protect the vulnerable fetus.

7. A woman who is 14 weeks pregnant tells the nurse that she always had a glass of wine with dinner before she became pregnant. She has abstained during her first trimester and would like to know if it is safe for her to have a drink with dinner now. The nurse would tell her: "Since you're in your second trimester, there's no problem with having one drink with dinner." b. "One drink every night is too much. One drink three times a week should be fine." c. "Since you're in your second trimester, you can drink as much as you like." d. "Because no one knows how much or how little alcohol it takes to cause fetal problems, the best course is to abstain throughout your pregnancy."

ANS: D The statement "Because no one knows how much or how little alcohol it takes to cause fetal problems, the best course is to abstain throughout your pregnancy" is accurate. A safe level of alcohol consumption during pregnancy has not yet been established. Although the consumption of occasional alcoholic beverages may not be harmful to the mother or her developing fetus, complete abstinence is strongly advised.

10. A woman is 3 months pregnant. At her prenatal visit, she tells the nurse that she doesn't know what is happening; one minute she's happy that she is pregnant, and the next minute she cries for no reason. Which response by the nurse is most appropriate? a. "Don't worry about it; you'll feel better in a month or so." b. "Have you talked to your husband about how you feel?" c. "Perhaps you really don't want to be pregnant." d. "Hormonal changes during pregnancy commonly result in mood swings."

ANS: D The statement "Hormonal changes during pregnancy commonly result in mood swings" is accurate and the most appropriate response by the nurse. The statement "Don't worry about it; you'll feel better in a month or so" dismisses the client's concerns and is not the most appropriate response. Although women should be encouraged to share their feelings, "Have you talked to your husband about how you feel" is not the most appropriate response and does not provide the client with a rationale for the psychosocial dynamics of her pregnancy. "Perhaps you really don't want to be pregnant" is completely inappropriate and deleterious to the psychologic well-being of the woman. Hormonal and metabolic adaptations often cause mood swings in pregnancy. The woman's responses are normal. She should be reassured about her feelings.

20. What represents a typical progression through the phases of a woman's establishing a relationship with the fetus? a. Accepts the fetus as distinct from herself—accepts the biologic fact of pregnancy—has a feeling of caring and responsibility b. Fantasizes about the child's gender and personality—views the child as part of herself—becomes introspective c. Views the child as part of herself—has feelings of well-being—accepts the biologic fact of pregnancy d. "I am pregnant."—"I am going to have a baby."—"I am going to be a mother.

ANS: D The woman first centers on herself as pregnant, then on the baby as an entity separate from herself, and then on her responsibilities as a mother. The expressions, "I am pregnant," "I am going to have a baby," and "I am going to be a mother" sum up the progression through the three phases.

A woman in the active phase of the first stage of labor is using a shallow pattern of breathing, which is about twice the normal adult breathing rate. She starts to complain about feeling lightheaded and dizzy and states that her fingers are tingling. The nurse should: a. Notify the woman's physician. b. Tell the woman to slow the pace of her breathing. c. Administer oxygen via a mask or nasal cannula. d. Help her breathe into a paper bag

ANS: D This woman is experiencing the side effects of hyperventilation, which include the symptoms of lightheadedness, dizziness, tingling of the fingers, or circumoral numbness. Having the woman breathe into a paper bag held tightly around her mouth and nose may eliminate respiratory alkalosis. This enables her to rebreathe carbon dioxide and replace the bicarbonate ion.

Maternity nurses often have to answer questions about the many, sometimes unusual ways people have tried to make the birthing experience more comfortable. For instance, nurses should be aware that: a. Music supplied by the support person has to be discouraged because it could disturb others or upset the hospital routine. b. Women in labor can benefit from sitting in a bathtub, but they must limit immersion to no longer than 15 minutes at a time. c. Effleurage is permissible, but counterpressure is almost always counterproductive. d. Electrodes attached to either side of the spine to provide high-intensity electrical impulses facilitate the release of endorphins.

ANS: D Transcutaneous electrical nerve stimulation does help. Music may be very helpful for reducing tension and certainly can be accommodated by the hospital. Women can stay in a bath as long as they want, although repeated baths with breaks may be more effective than a long soak. Counterpressure can help the woman cope with lower back pain.

9. For what reason would breastfeeding be contraindicated? a. Hepatitis B b. Everted nipples c. History of breast cancer 3 years ago d. Human immunodeficiency virus (HIV) positive

ANS: D Women who are HIV positive are discouraged from breastfeeding. Although hepatitis B antigen has not been shown to be transmitted through breast milk, as an added precaution infants born to HBsAg-positive women should receive the hepatitis B vaccine and immune globulin immediately after birth. Everted nipples are functional for breastfeeding. Newly diagnosed breast cancer would be a contraindication to breastfeeding

A patient whose cervix is dilated to 5 cm is considered to be in which phase of labor?

Active phase

Nurses can help their clients by keeping them informed about the distinctive stages of labor. What description of the phases of the first stage of labor is accurate?

Active: Moderate, regular contractions; 4- to 7-cm dilation; duration of 3 to 6 hours

Concerning the third stage of labor, nurses should be aware that:

An expectant or active approach to managing this stage of labor reduces the risk of complications.

A laboring woman is lying in the supine position. The most appropriate nursing action at this time is to:

Ask her to turn to one side

When a nulliparous woman telephones the hospital to report that she is in labor, the nurse initially should:

Ask the woman to describe why she believes she is in labor.

A multiparous woman has been in labor for 8 hours. Her membranes have just ruptured. The nurse's initial response would be to:

Assess the fetal heart rate and pattern.

When assessing a woman in labor, the nurse is aware that the relationship of the fetal body parts to one another is called fetal:

Attitude

In planning for an expected cesarean birth for a woman who has given birth by cesarean previously and who has a fetus in the transverse presentation, which information would the nurse include? a. "Because this is a repeat procedure, you are at the lowest risk for complications." b. "Even though this is your second cesarean birth, you may wish to review the preoperative and postoperative procedures." c. "Because this is your second cesarean birth, you will recover faster." d. "You will not need preoperative teaching because this is your second cesarean birth."

B "Even though this is your second cesarean birth, you may wish to review the preoperative and postoperative procedures" is the most appropriate statement. It is not accurate to state that the woman is at the lowest risk for complications. Both maternal and fetal risks are associated with every cesarean section. "Because this is your second cesarean birth, you will recover faster" is not an accurate statement. Physiologic and psychologic recovery from a cesarean section is multifactorial and individual to each client each time. Preoperative teaching should always be performed, regardless of whether the client has already had this procedure.

A woman who is 39 weeks pregnant expresses fear about her impending labor and how she will manage. The nurse's best response is: a. "Don't worry about it. You'll do fine." b. "It's normal to be anxious about labor. Let's discuss what makes you afraid." c. "Labor is scary to think about, but the actual experience isn't." d. "You can have an epidural. You won't feel anything."

B "It's normal to be anxious about labor. Let's discuss what makes you afraid" allows the woman to share her concerns with the nurse and is a therapeutic communication tool. "Don't worry about it. You'll do fine" negates the woman's fears and is not therapeutic. "Labor is scary to think about, but the actual experience isn't" negates the woman's fears and offers a false sense of security. It is not true that every woman may have an epidural. A number of criteria must be met for use of an epidural. Furthermore, many women still experience the feeling of pressure with an epidural.

A new client and her partner arrive on the labor, delivery, recovery, and postpartum unit for the birth of their first child. You apply the electronic fetal monitor (EFM) to the woman. Her partner asks you to explain what is printing on the graph, referring to the EFM strip. He wants to know what the baby's heart rate should be. Your best response is: a. "Don't worry about that machine; that's my job." b. "The top line graphs the baby's heart rate. Generally the heart rate is between 110 and 160. The heart rate will fluctuate in response to what is happening during labor." c. "The top line graphs the baby's heart rate, and the bottom line lets me know how strong the contractions are." d. "Your doctor will explain all of that later."

B "The top line graphs the baby's heart rate. Generally the heart rate is between 110 and 160. The heart rate will fluctuate in response to what is happening during labor" educates the partner about fetal monitoring and provides support and information to alleviate his fears. "Don't worry about that machine; that's my job" discredits the partner's feelings and does not provide the teaching he is requesting. "The top line graphs the baby's heart rate, and the bottom line lets me know how strong the contractions are" provides inaccurate information and does not address the partner's concerns about the fetal heart rate. The EFM graphs the frequency and duration of the contractions, not the intensity. Nurses should take every opportunity to provide client and family teaching, especially when information is requested.

A maternal indication for the use of vacuum extraction is: a. A wide pelvic outlet. c. A history of rapid deliveries. b. Maternal exhaustion. d. Failure to progress past 0 station.

B A mother who is exhausted may be unable to assist with the expulsion of the fetus. The patient with a wide pelvic outlet will likely not require vacuum extraction. With a rapid delivery, vacuum extraction is not necessary. A station of 0 is too high for a vacuum extraction.

Concerning the third stage of labor, nurses should be aware that: a. The placenta eventually detaches itself from a flaccid uterus. b. An expectant or active approach to managing this stage of labor reduces the risk of complications. c. It is important that the dark, roughened maternal surface of the placenta appear before the shiny fetal surface. d. The major risk for women during the third stage is a rapid heart rate.

B Active management facilitates placental separation and expulsion, thus reducing the risk of complications. The placenta cannot detach itself from a flaccid (relaxed) uterus. Which surface of the placenta comes out first is not clinically important. The major risk for women during the third stage of labor is postpartum hemorrhage.

A woman who is gravida 3 para 2 enters the intrapartum unit. The most important nursing assessments are: a. Contraction pattern, amount of discomfort, and pregnancy history. b. Fetal heart rate, maternal vital signs, and the woman's nearness to birth. c. Identification of ruptured membranes, the woman's gravida and para, and her support person. d. Last food intake, when labor began, and cultural practices the couple desires.

B All options describe relevant intrapartum nursing assessments; however, this focused assessment has priority. If the maternal and fetal conditions are normal and birth is not imminent, other assessments can be performed in an unhurried manner. This includes: gravida, para, support person, pregnancy history, pain assessment, last food intake, and cultural practices.

What is an expected characteristic of amniotic fluid? a. Deep yellow color b. Pale, straw color with small white particles c. Acidic result on a Nitrazine test d. Absence of ferning

B Amniotic fluid normally is a pale, straw-colored fluid that may contain white flecks of vernix. Yellow-stained fluid may indicate fetal hypoxia up to 36 hours before rupture of membranes, fetal hemolytic disease, or intrauterine infection. Amniotic fluid produces an alkaline result on a Nitrazine test. The presence of ferning is a positive indication of amniotic fluid.

Under which circumstance would it be unnecessary for the nurse to perform a vaginal examination? a. An admission to the hospital at the start of labor b. When accelerations of the fetal heart rate (FHR) are noted c. On maternal perception of perineal pressure or the urge to bear down d. When membranes rupture

B An accelerated FHR is a positive sign; however, variable decelerations merit a vaginal examination. Vaginal examinations should be performed when the woman is admitted, when she perceives perineal pressure or the urge to bear down, when her membranes rupture, when a significant change in her uterine activity has occurred, or when variable decelerations of the FHR are noted.

While caring for the patient who requires an induction of labor, the nurse should be cognizant that: a. Ripening the cervix usually results in a decreased success rate for induction. b. Labor sometimes can be induced with balloon catheters or laminaria tents. c. Oxytocin is less expensive than prostaglandins and more effective but creates greater health risks. d. Amniotomy can be used to make the cervix more favorable for labor.

B Balloon catheters or laminaria tents are mechanical means of ripening the cervix. Ripening the cervix, making it softer and thinner, increases the success rate of induced labor. Prostaglandin E1 is less expensive and more effective than oxytocin but carries a greater risk. Amniotomy is the artificial rupture of membranes, which is used to induce labor only when the cervix is already ripe.

Nurses should know some basic definitions concerning preterm birth, preterm labor, and low birth weight. For instance: a. The terms preterm birth and low birth weight can be used interchangeably. b. Preterm labor is defined as cervical changes and uterine contractions occurring between 20 and 37 weeks of pregnancy. c. Low birth weight is anything below 3.7 pounds. d. In the United States early in this century, preterm birth accounted for 18% to 20% of all births.

B Before 20 weeks, it is not viable (miscarriage); after 37 weeks, it can be considered term. Although these terms are used interchangeably, they have different meanings: preterm birth describes the length of gestation (37 weeks) regardless of weight; low birth weight describes weight only (2500 g or less) at the time of birth, whenever it occurs. Low birth weight is anything less than 2500 g, or about 5.5 pounds. In 2003 the preterm birth rate in the United States was 12.3%, but it is increasing in frequency.

Which statement is the best rationale for assessing maternal vital signs between contractions? a. During a contraction, assessing fetal heart rates is the priority. b. Maternal circulating blood volume increases temporarily during contractions. c. Maternal blood flow to the heart is reduced during contractions. d. Vital signs taken during contractions are not accurate.

B During uterine contractions, blood flow to the placenta temporarily stops, causing a relative increase in the mother's blood volume, which in turn temporarily increases blood pressure and slows pulse. It is important to monitor fetal response to contractions; however, this question is concerned with the maternal vital signs. Maternal blood flow is increased during a contraction. Vital signs are altered by contractions but are considered accurate for that period of time.

The slight overlapping of cranial bones or shaping of the fetal head during labor is called: a. Lightening. c. Ferguson reflex. b. Molding. d. Valsalva maneuver.

B Fetal head formation is called molding. Molding also permits adaptation to various diameters of the maternal pelvis. Lightening is the mother's sensation of decreased abdominal distention, which usually occurs the week before labor. The Ferguson reflex is the contraction urge of the uterus after stimulation of the cervix. The Valsalva maneuver describes conscious pushing during the second stage of labor.

The primary difference between the labor of a nullipara and that of a multipara is the: a. Amount of cervical dilation. c. Level of pain experienced. b. Total duration of labor. d. Sequence of labor mechanisms.

B In a first-time pregnancy, descent is usually slow but steady; in subsequent pregnancies, descent is more rapid, resulting in a shorter duration of labor. Cervical dilation is the same for all labors. Level of pain is individual to the woman, not to the number of labors she has experienced. The sequence of labor mechanisms is the same with all labors.

What correctly matches the type of deceleration with its likely cause? a. Early deceleration—umbilical cord compression b. Late deceleration—uteroplacental inefficiency c. Variable deceleration—head compression d. Prolonged deceleration—cause unknown

B Late deceleration is caused by uteroplacental inefficiency. Early deceleration is caused by head compression. Variable deceleration is caused by umbilical cord compression. Prolonged deceleration has a variety of either benign or critical causes.

The primary difference between the labor of a nullipara and that of a multipara is the: a. Amount of cervical dilation. b. Total duration of labor. c. Level of pain experienced. d. Sequence of labor mechanisms.

B Multiparas usually labor more quickly than nulliparas, thus making the total duration of their labor shorter. Cervical dilation is the same for all labors. The level of pain is individual to the woman, not to the number of labors she has experienced. The sequence of labor mechanisms remains the same with all labors.

The nurse expects to administer an oxytocic (e.g., Pitocin, Methergine) to a woman after expulsion of her placenta to: a. Relieve pain. b. Stimulate uterine contraction. c. Prevent infection. d. Facilitate rest and relaxation.

B Oxytocics stimulate uterine contractions, which reduce blood loss after the third stage of labor. Oxytocics are not used to treat pain or prevent infection. They cause the uterus to contract, which reduces blood loss. Oxytocics do not facilitate rest and relaxation.

With regard to dysfunctional labor, nurses should be aware that: a. Women who are underweight are more at risk. b. Women experiencing precipitous labor are about the only "dysfunctionals" not to be exhausted. c. Hypertonic uterine dysfunction is more common than hypotonic dysfunction. d. Abnormal labor patterns are most common in older women.

B Precipitous labor lasts less than 3 hours. Short women more than 30 pounds overweight are more at risk for dysfunctional labor. Hypotonic uterine dysfunction, in which the contractions become weaker, is more common. Abnormal labor patterns are more common in women less than 20 years of age.

What three measures should the nurse implement to provide intrauterine resuscitation? Select the response that best indicates the priority of actions that should be taken. a. Call the provider, reposition the mother, and perform a vaginal examination. b. Reposition the mother, increase intravenous (IV) fluid, and provide oxygen via face mask. c. Administer oxygen to the mother, increase IV fluid, and notify the care provider. d. Perform a vaginal examination, reposition the mother, and provide oxygen via face mask.

B Repositioning the mother, increasing intravenous (IV) fluid, and providing oxygen via face mask are correct nursing actions for intrauterine resuscitation. The nurse should initiate intrauterine resuscitation in an ABC manner, similar to basic life support. The first priority is to open the maternal and fetal vascular systems by repositioning the mother for improved perfusion. The second priority is to increase blood volume by increasing the IV fluid. The third priority is to optimize oxygenation of the circulatory volume by providing oxygen via face mask. If these interventions do not resolve the fetal heart rate issue quickly, the primary provider should be notified immediately.

After an emergency birth, the nurse encourages the woman to breastfeed her newborn. The primary purpose of this activity is to: a. Facilitate maternal-newborn interaction. b. Stimulate the uterus to contract. c. Prevent neonatal hypoglycemia. d. Initiate the lactation cycle.

B Stimulation of the nipples through breastfeeding or manual stimulation causes the release of oxytocin and prevents maternal hemorrhage. Breastfeeding facilitates maternal-newborn interaction, but it is not the primary reason a woman is encouraged to breastfeed after an emergency birth. The primary intervention for preventing neonatal hypoglycemia is thermoregulation. Cold stress can result in hypoglycemia. The woman is encouraged to breastfeed after an emergency birth to stimulate the release of oxytocin, which prevents hemorrhage. Breastfeeding is encouraged to initiate the lactation cycle, but it is not the primary reason for this activity after an emergency birth.

The nurse providing care for a woman with preterm labor who is receiving terbutaline would include which intervention to identify side effects of the drug? a. Assessing deep tendon reflexes (DTRs) b. Assessing for chest discomfort and palpitations c. Assessing for bradycardia d. Assessing for hypoglycemia

B Terbutaline is a 2-adrenergic agonist that affects the cardiopulmonary and metabolic systems of the mother. Signs of cardiopulmonary decompensation would include chest pain and palpitations. Assessing DTRs would not address these concerns. 2-Adrenergic agonist drugs cause tachycardia, not bradycardia. The metabolic effect leads to hyperglycemia, not hypoglycemia.

Nurses can help their clients by keeping them informed about the distinctive stages of labor. What description of the phases of the first stage of labor is accurate? a. Latent: Mild, regular contractions; no dilation; bloody show; duration of 2 to 4 hours b. Active: Moderate, regular contractions; 4- to 7-cm dilation; duration of 3 to 6 hours c. Lull: No contractions; dilation stable; duration of 20 to 60 minutes d. Transition: Very strong but irregular contractions; 8- to 10-cm dilation; duration of 1 to 2 hours

B The active phase is characterized by moderate, regular contractions; 4- to 7-cm dilation; and a duration of 3 to 6 hours. The latent phase is characterized by mild-to-moderate, irregular contractions; dilation up to 3 cm; brownish-to-pale pink mucus, and a duration of 6 to 8 hours. No official "lull" phase exists in the first stage. The transition phase is characterized by strong to very strong, regular contractions; 8- to 10-cm dilation; and a duration of 20 to 40 minutes.

During labor a fetus with an average heart rate of 135 beats/min over a 10-minute period would be considered to have: a. Bradycardia. c. Tachycardia. b. A normal baseline heart rate. d. Hypoxia.

B The baseline heart rate is measured over 10 minutes; a normal range is 110 to 160 beats/min. Bradycardia is a fetal heart rate (FHR) below 110 beats/min for 10 minutes or longer. Tachycardia is an FHR over 160 beats/min for 10 minutes or longer. Hypoxia is an inadequate supply of oxygen; no indication of this condition exists with a baseline heart rate in the normal range.

A pregnant woman at 29 weeks of gestation has been diagnosed with preterm labor. Her labor is being controlled with tocolytic medications. She asks when she would be able to go home. Which response by the nurse is most accurate? a. "After the baby is born." b. "When we can stabilize your preterm labor and arrange home health visits." c. "Whenever the doctor says that it is okay." d. "It depends on what kind of insurance coverage you have."

B The client's preterm labor is being controlled with tocolytics. Once she is stable, home care may be a viable option for this type of client. Care of a woman with preterm labor is multifactorial; the goal is to prevent delivery. In many cases this may be achieved at home. Care of the preterm client is multidisciplinary and multifactorial. Managed care may dictate earlier hospital discharges or a shift from hospital to home care. Insurance coverage may be one factor in the care of clients, but ultimately client safety remains the most important factor.

To teach patients about the process of labor adequately, the nurse knows that which event is the best indicator of true labor? a. Bloody show c. Fetal descent into the pelvic inlet b. Cervical dilation and effacement d. Uterine contractions every 7 minutes

B The conclusive distinction between true and false labor is that contractions of true labor cause progressive change in the cervix. Bloody show can occur before true labor. Fetal descent can occur before true labor. False labor may have contractions that occur this frequently; however, this is usually inconsistent.

The nurse has received report regarding her patient in labor. The woman's last vaginal examination was recorded as 3 cm, 30%, and ?2-2. The nurse's interpretation of this assessment is that: a. The cervix is effaced 3 cm, it is dilated 30%, and the presenting part is 2 cm above the ischial spines. b. The cervix is 3 cm dilated, it is effaced 30%, and the presenting part is 2 cm above the ischial spines. c. The cervix is effaced 3 cm, it is dilated 30%, and the presenting part is 2 cm below the ischial spines. d. The cervix is dilated 3 cm, it is effaced 30%, and the presenting part is 2 cm below the ischial spines.

B The correct description of the vaginal examination for this woman in labor is the cervix is 3 cm dilated, it is effaced 30%, and the presenting part is 2 cm above the ischial spines. The sterile vaginal examination is recorded as centimeters of cervical dilation, percentage of cervical dilation, and the relationship of the presenting part to the ischial spines (either above or below).

Through vaginal examination the nurse determines that a woman is 4 cm dilated, and the external fetal monitor shows uterine contractions every 3.5 to 4 minutes. The nurse would report this as: a. First stage, latent phase. b. First stage, active phase. c. First stage, transition phase. d. Second stage, latent phase.

B The first stage, active phase of maternal progress indicates that the woman is in the active phase of the first stage of labor. During the latent phase of the first stage of labor, the expected maternal progress would be 0 to 3 cm dilation with contractions every 5 to 30 minutes. During the transition phase of the first stage of labor, the expected maternal progress is 8 to 10 cm dilation with contractions every 2 to 3 minutes. During the latent phase of the second stage of labor, the woman is completely dilated and experiences a restful period of "laboring down."

For women who have a history of sexual abuse, a number of traumatic memories may be triggered during labor. The woman may fight the labor process and react with pain or anger. Alternately, she may become a passive player and emotionally absent herself from the process. The nurse is in a unique position of being able to assist the client to associate the sensations of labor with the process of childbirth and not the past abuse. The nurse can implement a number of care measures to help the client view the childbirth experience in a positive manner. Which intervention would be key for the nurse to use while providing care? a. Telling the client to relax and that it won't hurt much b. Limiting the number of procedures that invade her body c. Reassuring the client that as the nurse you know what is best d. Allowing unlimited care providers to be with the client

B The number of invasive procedures such as vaginal examinations, internal monitoring, and intravenous therapy should be limited as much as possible. The nurse should always avoid words and phrases that may result in the client's recalling the phrases of her abuser (e.g., "Relax, this won't hurt" or "Just open your legs.") The woman's sense of control should be maintained at all times. The nurse should explain procedures at the client's pace and wait for permission to proceed. Protecting the client's environment by providing privacy and limiting the number of staff who observe the client will help to make her feel safe.

Nurses alert to signs of the onset of the second stage of labor can be certain that this stage has begun when: a. The woman has a sudden episode of vomiting. b. The nurse is unable to feel the cervix during a vaginal examination. c. Bloody show increases. d. The woman involuntarily bears down.

B The only certain objective sign that the second stage has begun is the inability to feel the cervix because it is fully dilated and effaced. Vomiting, an increase in bloody show, and involuntary bearing down are only suggestions of second-stage labor.

As relates to fetal positioning during labor, nurses should be aware that: a. Position is a measure of the degree of descent of the presenting part of the fetus through the birth canal. b. Birth is imminent when the presenting part is at +4 to +5 cm below the spine. c. The largest transverse diameter of the presenting part is the suboccipitobregmatic diameter. d. Engagement is the term used to describe the beginning of labor.

B The station of the presenting part should be noted at the beginning of labor so that the rate of descent can be determined. Position is the relation of the presenting part of the fetus to the four quadrants of the mother's pelvis; station is the measure of degree of descent. The largest diameter usually is the biparietal diameter. The suboccipitobregmatic diameter is the smallest, although one of the most critical. Engagement often occurs in the weeks just before labor in nulliparas and before or during labor in multiparas.

Why is continuous electronic fetal monitoring usually used when oxytocin is administered? a. The mother may become hypotensive. b. Uteroplacental exchange may be compromised. c. Maternal fluid volume deficit may occur. d. Fetal chemoreceptors are stimulated.

B The uterus may contract more firmly, and the resting tone may be increased with oxytocin use. This response reduces entrance of freshly oxygenated maternal blood into the intervillous spaces, thus depleting fetal oxygen reserves. Hypotension is not a common side effect of oxytocin. All laboring women are at risk for fluid volume deficit; oxytocin administration does not increase the risk. Oxytocin affects the uterine muscles.

The nurse is caring for a client whose labor is being augmented with oxytocin. He or she recognizes that the oxytocin should be discontinued immediately if there is evidence of: a. Uterine contractions occurring every 8 to 10 minutes. b. A fetal heart rate (FHR) of 180 with absence of variability. c. The client's needing to void. d. Rupture of the client's amniotic membranes.

B This FHR is nonreassuring. The oxytocin should be discontinued immediately, and the physician should be notified. The oxytocin should be discontinued if uterine hyperstimulation occurs. Uterine contractions that are occurring every 8 to 10 minutes do not qualify as hyperstimulation. The client's needing to void is not an indication to discontinue the oxytocin induction immediately or to call the physician. Unless a change occurs in the FHR pattern that is nonreassuring or the client experiences uterine hyperstimulation, the oxytocin does not need to be discontinued. The physician should be notified that the client's membranes have ruptured.

The nurse providing care for the laboring woman realizes that variable fetal heart rate (FHR) decelerations are caused by: a. Altered fetal cerebral blood flow. c. Uteroplacental insufficiency. b. Umbilical cord compression. d. Fetal hypoxemia.

B Variable decelerations can occur any time during the uterine contracting phase and are caused by compression of the umbilical cord. Altered fetal cerebral blood flow would result in early decelerations in the FHR. Uteroplacental insufficiency would result in late decelerations in the FHR. Fetal hypoxemia would result in tachycardia initially and then bradycardia if hypoxia continues.

Complications and risks associated with cesarean births include (Select all that apply): a. Placental abruption. b. Wound dehiscence. c. Hemorrhage. d. Urinary tract infections. e. Fetal injuries.

B, C, D, E Placental abruption and placenta previa are both indications for cesarean birth and are not complications thereof. Wound dehiscence, hemorrhage, urinary tract infection, and fetal injuries are all possible complications and risks associated with delivery by cesarean section.

A tiered system of categorizing FHR has been recommended by regulatory agencies. Nurses, midwives, and physicians who care for women in labor must have a working knowledge of fetal monitoring standards and understand the significance of each category. These categories include (Select all that apply): a. Reassuring. b. Category I. c. Category II. d. Nonreassuring. e. Category III.

B, C, E The three tiered system of FHR tracings include Category I, II, and III. Category I is a normal tracing requiring no action. Category II FHR tracings are indeterminate. This category includes tracings that do not meet Category I or III criteria. Category III tracings are abnormal and require immediate intervention.

The nurse recognizes that uterine hyperstimulation with oxytocin requires emergency interventions. What clinical cues would alert the nurse that the woman is experiencing uterine hyperstimulation (Select all that apply)? a. Uterine contractions lasting <90 seconds and occurring >2 minutes in frequency b. Uterine contractions lasting >90 seconds and occurring <2 minutes in frequency c. Uterine tone <20 mm Hg d. Uterine tone >20 mm Hg e. Increased uterine activity accompanied by a nonreassuring fetal heart rate (FHR) and pattern

B, D, E Uterine contractions that occur less than 2 minutes apart and last more than 90 seconds, a uterine tone of over 20 mm Hg, and a nonreassuring FHR and pattern are all indications of uterine hyperstimulation with oxytocin administration. Uterine contractions that occur more than 2 minutes apart and last less than 90 seconds are the expected goal of oxytocin induction. A uterine tone of less than 20 mm Hg is normal.

A pregnant woman in her first trimester with a history of epilepsy is transported to the hospital via ambulance after suffering a seizure in a restaurant. The nurse expects which health care provider orders to be included in the plan of care? (Select all that apply.) A. valproate (Depakote). B. Serum lab levels of medications. C. Abdominal ultrasounds. D. Prenatal vitamins with vitamin D. E. carbamazepine (Tegretol).

B. Serum lab levels of medications. C. Abdominal ultrasounds. D. Prenatal vitamins with vitamin D. Carbamazepine (Tegretol) and valproate (Depakote) should be avoided if possible during pregnancy, especially during the first trimester, because their use is associated with NTDs in the fetus. Checking lab levels of medications, performing abdominal ultrasounds to assess fetal growth, and taking prenatal vitamins with vitamin D are all expected interventions for a pregnant woman diagnosed with epilepsy.

From 4% to 8% of pregnant women have asthma, making it one of the most common preexisting conditions of pregnancy. Severity of symptoms usually peaks: A. in the first trimester. B. between 24 to 36 weeks of gestation. C. during the last 4 weeks of pregnancy. D. immediately postpartum

B. between 24 to 36 weeks of gestation. Women often have few symptoms of asthma during the first trimester. The severity of symptoms peaks between 24 and 36 weeks of gestation. Asthma appears to be associated with intrauterine growth restriction and preterm birth. During the last 4 weeks of pregnancy symptoms often subside. The period between 24 and 36 weeks of pregnancy is associated with the greatest severity of symptoms. Issues have often resolved by the time the woman delivers.

A pregnant woman with cardiac disease is informed about signs of cardiac decompensation. She should be told that the earliest sign of decompensation is most often: A. orthopnea. B. decreasing energy levels. C. moist frequent cough and frothy sputum. D. crackles (rales) at the bases of the lungs on auscultation.

B. decreasing energy levels. Orthopnea is a finding that appears later when a failing heart reduces renal perfusion and fluid accumulates in the pulmonary interstitial space, leading to pulmonary edema. Decreasing energy level (fatigue) is an early finding of heart failure. Care must be taken to recognize it as a warning rather than a typical change of the third trimester. Cardiac decompensation is most likely to occur early in the third trimester, during childbirth, and during the first 48 hours following birth. A moist, frequent cough appears later when a failing heart reduces renal perfusion and fluid accumulates in the pulmonary interstitial space, leading to pulmonary edema. Crackles and rales appear later when a failing heart reduces renal perfusion and fluid accumulates in the pulmonary interstitial space, leading to pulmonary edema.

Diabetes in pregnancy puts the fetus at risk in several ways. Nurses should be aware that: A. with good control of maternal glucose levels, sudden and unexplained stillbirth is no longer a major concern. B. the most important cause of perinatal loss in diabetic pregnancy is congenital malformations. C. infants of mothers with diabetes have the same risks for respiratory distress syndrome because of the careful monitoring. D. at birth, the neonate of a diabetic mother is no longer at any greater risk.

B. the most important cause of perinatal loss in diabetic pregnancy is congenital malformations. Even with good control, sudden and unexplained stillbirth remains a major concern. Congenital malformations account for 30% to 50% of perinatal deaths. Infants of diabetic mothers are at increased risk for respiratory distress syndrome. The transition to extrauterine life often is marked by hypoglycemia and other metabolic abnormalities.

Which nursing assessment indicates that a woman who is in second-stage labor is almost ready to give birth? a. The fetal head is felt at 0 station during vaginal examination. b. Bloody mucus discharge increases. c. The vulva bulges and encircles the fetal head. d. The membranes rupture during a contraction.

C A bulging vulva that encircles the fetal head describes crowning, which occurs shortly before birth. Birth of the head occurs when the station is +4. A 0 station indicates engagement. Bloody show occurs throughout the labor process and is not an indication of an imminent birth. Rupture of membranes can occur at any time during the labor process and does not indicate an imminent birth.

The priority nursing care associated with an oxytocin (Pitocin) infusion is: a. Measuring urinary output. b. Increasing infusion rate every 30 minutes. c. Monitoring uterine response. d. Evaluating cervical dilation.

C Because of the risk of hyperstimulation, which could result in decreased placental perfusion and uterine rupture, the nurse's priority intervention is monitoring uterine response. Monitoring urinary output is also important; however, it is not the top priority during the administration of Pitocin. The infusion rate may be increased after proper assessment that it is an appropriate interval to do so. Monitoring labor progression is the standard of care for all labor patients.

As relates to the use of tocolytic therapy to suppress uterine activity, nurses should be aware that: a. The drugs can be given efficaciously up to the designated beginning of term at 37 weeks. b. There are no important maternal (as opposed to fetal) contraindications. c. Its most important function is to afford the opportunity to administer antenatal glucocorticoids. d. If the client develops pulmonary edema while receiving tocolytics, intravenous (IV) fluids should be given.

C Buying time for antenatal glucocorticoids to accelerate fetal lung development may be the best reason to use tocolytics. Once the pregnancy has reached 34 weeks, the risks of tocolytic therapy outweigh the benefits. There are important maternal contraindications to tocolytic therapy. Tocolytic-induced edema can be caused by IV fluids.

A nulliparous woman who has just begun the second stage of her labor would most likely: a. Experience a strong urge to bear down. b. Show perineal bulging. c. Feel tired yet relieved that the worst is over. d. Show an increase in bright red bloody show.

C Common maternal behaviors during the latent phase of the second stage of labor include feeling a sense of accomplishment and optimism because "the worst is over." During the latent phase of the second stage of labor, the urge to bear down often is absent or only slight during the acme of contractions. Perineal bulging occurs during the transition phase of the second stage of labor, not at the beginning of the second stage. An increase in bright red bloody show occurs during the descent phase of the second stage of labor.

Which nursing assessment indicates that a woman who is in second-stage labor is almost ready to give birth? a. The fetal head is felt at 0 station during vaginal examination. b. Bloody mucus discharge increases. c. The vulva bulges and encircles the fetal head. d. The membranes rupture during a contraction.

C During the active pushing (descent) phase, the woman has strong urges to bear down as the presenting part of the fetus descends and presses on the stretch receptors of the pelvic floor. The vulva stretches and begins to bulge encircling the fetal head. Birth of the head occurs when the station is +4. A 0 station indicates engagement. Bloody show occurs throughout the labor process and is not an indication of an imminent birth. Rupture of membranes can occur at any time during the labor process and does not indicate an imminent birth.

The nurse knows that the second stage of labor, the descent phase, has begun when: a. The amniotic membranes rupture. b. The cervix cannot be felt during a vaginal examination. c. The woman experiences a strong urge to bear down. d. The presenting part is below the ischial spines.

C During the descent phase of the second stage of labor, the woman may experience an increase in the urge to bear down. Rupture of membranes has no significance in determining the stage of labor. The second stage of labor begins with full cervical dilation. Many women may have an urge to bear down when the presenting part is below the level of the ischial spines. This can occur during the first stage of labor, as early as 5-cm dilation.

Fetal bradycardia is most common during: a. Intraamniotic infection. b. Fetal anemia. c. Prolonged umbilical cord compression. d. Tocolytic treatment using terbutaline.

C Fetal bradycardia can be considered a later sign of fetal hypoxia and is known to occur before fetal death. Bradycardia can result from placental transfer of drugs, prolonged compression of the umbilical cord, maternal hypothermia, and maternal hypotension. Intraamniotic infection, fetal anemia, and tocolytic treatment using terbutaline would most likely result in fetal tachycardia.

While providing care to a patient in active labor, the nurse should instruct the woman that: a. The supine position commonly used in the United States increases blood flow. b. The "all fours" position, on her hands and knees, is hard on her back. c. Frequent changes in position will help relieve her fatigue and increase her comfort. d. In a sitting or squatting position, her abdominal muscles will have to work harder.

C Frequent position changes relieve fatigue, increase comfort, and improve circulation. Blood flow can be compromised in the supine position; any upright position benefits cardiac output. The "all fours" position is used to relieve backache in certain situations. In a sitting or squatting position, the abdominal muscles work in greater harmony with uterine contractions.

The nurse thoroughly dries the infant immediately after birth primarily to: a. Stimulate crying and lung expansion. b. Remove maternal blood from the skin surface. c. Reduce heat loss from evaporation. d. Increase blood supply to the hands and feet.

C Infants are wet with amniotic fluid and blood at birth, and this accelerates evaporative heat loss. The primary purpose of drying the infant is to prevent heat loss. Rubbing the infant does stimulate crying; however, it is not the main reason for drying the infant. This process does not remove all the maternal blood.

Prepidil (prostaglandin gel) has been ordered for a pregnant woman at 43 weeks of gestation. The nurse recognizes that this medication will be administered to: a. Enhance uteroplacental perfusion in an aging placenta. b. Increase amniotic fluid volume. c. Ripen the cervix in preparation for labor induction. d. Stimulate the amniotic membranes to rupture.

C It is accurate to state that Prepidil will be administered to ripen the cervix in preparation for labor induction. It is not administered to enhance uteroplacental perfusion in an aging placenta, increase amniotic fluid volume, or stimulate the amniotic membranes to rupture.

The nurse caring for the woman in labor should understand that maternal hypotension can result in: a. Early decelerations. c. Uteroplacental insufficiency. b. Fetal dysrhythmias. d. Spontaneous rupture of membranes.

C Low maternal blood pressure reduces placental blood flow during uterine contractions and results in fetal hypoxemia. Maternal hypotension is not associated with early decelerations, fetal dysrhythmias, or spontaneous rupture of membranes.

The nurse caring for the woman in labor should understand that increased variability of the fetal heart rate may be caused by: a. Narcotics. c. Methamphetamines. b. Barbiturates. d. Tranquilizers.

C Narcotics, barbiturates, and tranquilizers may be causes of decreased variability; methamphetamines may cause increased variability.

Because the risk for childbirth complications may be revealed, nurses should know that the point of maximal intensity (PMI) of the fetal heart tone (FHT) is: a. Usually directly over the fetal abdomen. b. In a vertex position heard above the mother's umbilicus. c. Heard lower and closer to the midline of the mother's abdomen as the fetus descends and rotates internally. d. In a breech position heard below the mother's umbilicus.

C Nurses should be prepared for the shift. The PMI of the FHT usually is directly over the fetal back. In a vertex position it is heard below the mother's umbilicus. In a breech position it is heard above the mother's umbilicus.

At 1 minute after birth, the nurse assesses the newborn to assign an Apgar score. The apical heart rate is 110 bpm, and the infant is crying vigorously with the limbs flexed. The infant's trunk is pink, but the hands and feet are blue. What is the correct Apgar score for this infant? a. 7 b. 8 c. 9 d. 10

C The Apgar score is 9 because 1 point is deducted from the total score of 10 for the infant's blue hands and feet. The baby received 2 points for each of the categories except color. Because the infant's hands and feet were blue, this category is given a grade of 1.

Surgical, medical, or mechanical methods may be used for labor induction. Which technique is considered a mechanical method of induction? a. Amniotomy c. Transcervical catheter b. Intravenous Pitocin d. Vaginal insertion of prostaglandins

C Placement of a balloon-tipped Foley catheter into the cervix is a mechanical method of induction. Other methods to expand and gradually dilate the cervix include hydroscopic dilators such as laminaria tents (made from desiccated seaweed), or Lamicel (contains magnesium sulfate). Amniotomy is a surgical method of augmentation and induction. Intravenous Pitocin and insertion of prostaglandins are medical methods of induction.

In planning for home care of a woman with preterm labor, which concern must the nurse address? a. Nursing assessments will be different from those done in the hospital setting. b. Restricted activity and medications will be necessary to prevent recurrence of preterm labor. c. Prolonged bed rest may cause negative physiologic effects. d. Home health care providers will be necessary.

C Prolonged bed rest may cause adverse effects such as weight loss, loss of appetite, muscle wasting, weakness, bone demineralization, decreased cardiac output, risk for thrombophlebitis, alteration in bowel functions, sleep disturbance, and prolonged postpartum recovery. Nursing assessments will differ somewhat from those performed in the acute care setting, but this is not the concern that needs to be addressed. Restricted activity and medication may prevent preterm labor, but not in all women. In addition, the plan of care is individualized to meet the needs of each woman. Many women will receive home health nurse visits, but care is individualized for each woman.

The nurse caring for a woman in labor understands that prolonged decelerations: a. Are a continuing pattern of benign decelerations that do not require intervention. b. Constitute a baseline change when they last longer than 5 minutes. c. Usually are isolated events that end spontaneously. d. Require the usual fetal monitoring by the nurse.

C Prolonged decelerations usually are isolated events that end spontaneously. However, in certain combinations with late and/or variable decelerations, they are a danger sign that requires the nurse to notify the physician or midwife immediately. A deceleration that lasts longer than 10 minutes constitutes a baseline change.

The nurse recognizes that a woman is in true labor when she states: a. "I passed some thick, pink mucus when I urinated this morning." b. "My bag of waters just broke." c. "The contractions in my uterus are getting stronger and closer together." d. "My baby dropped, and I have to urinate more frequently now."

C Regular, strong contractions with the presence of cervical change indicate that the woman is experiencing true labor. Loss of the mucous plug (operculum) often occurs during the first stage of labor or before the onset of labor, but it is not the indicator of true labor. Spontaneous rupture of membranes often occurs during the first stage of labor, but it is not the indicator of true labor. The presenting part of the fetus typically becomes engaged in the pelvis at the onset of labor, but this is not the indicator of true labor.

According to standard professional thinking, nurses should auscultate the fetal heart rate (FHR): a. Every 15 minutes in the active phase of the first stage of labor in the absence of risk factors. b. Every 20 minutes in the second stage, regardless of whether risk factors are present. c. Before and after ambulation and rupture of membranes. d. More often in a woman's first pregnancy.

C The FHR should be auscultated before and after administration of medications and induction of anesthesia. In the active phase of the first stage of labor, the FHR should be auscultated every 30 minutes if no risk factors are involved; with risk factors it should be auscultated every 15 minutes. In the second stage of labor the FHR should be auscultated every 15 minutes if no risk factors are involved; with risk factors it should be auscultated every 5 minutes. The fetus of a first-time mother is automatically at greater risk.

Which description of the phases of the second stage of labor is accurate? a. Latent phase: Feeling sleepy, fetal station 2+ to 4+, duration 30 to 45 minutes b. Active phase: Overwhelmingly strong contractions, Ferguson reflux activated, duration 5 to 15 minutes c. Descent phase: Significant increase in contractions, Ferguson reflux activated, average duration varied d. Transitional phase: Woman "laboring down," fetal station 0, duration 15 minutes

C The descent phase begins with a significant increase in contractions; the Ferguson reflex is activated, and the duration varies, depending on a number of factors. The latent phase is the lull, or "laboring down," period at the beginning of the second stage. It lasts 10 to 30 minutes on average. The second stage of labor has no active phase. The transition phase is the final phase in the second stage of labor; contractions are strong and painful.

To provide safe care for the woman, the nurse understands that which condition is a contraindication for an amniotomy? a. Dilation less than 3 cm c. -2 station b. Cephalic presentation d. Right occiput posterior position

C The dilation of the cervix must be great enough to determine labor. The presenting part of the fetus should be engaged and well applied to the cervix before the procedure in order to prevent cord prolapse. Amniotomy is deferred if the presenting part is higher in the pelvis. ROP indicates a cephalic presentation, which is appropriate for an amniotomy.

When assessing the fetus using Leopold maneuvers, the nurse feels a round, firm, movable fetal part in the fundal portion of the uterus and a long, smooth surface in the mother's right side close to midline. What is the likely position of the fetus? a. ROA c. RSA b. LSP d. LOA

C The fetus is positioned anteriorly in the right side of the maternal pelvis with the sacrum as the presenting part. RSA is the correct three-letter abbreviation to indicate this fetal position. The first letter indicates the presenting part in either the right or left side of the maternal pelvis. The second letter indicates the anatomic presenting part of the fetus. The third letter stands for the location of the presenting part in relation to the anterior, posterior, or transverse portion of the maternal pelvis. Palpation of a round, firm fetal part in the fundal portion of the uterus would be the fetal head, indicating that the fetus is in a breech position with the sacrum as the presenting part in the maternal pelvis. Palpation of the fetal spine along the mother's right side denotes the location of the presenting part in the mother's pelvis. The ability to palpate the fetal spine indicates that the fetus is anteriorly positioned in the maternal pelvis.

The least common cause of long, difficult, or abnormal labor (dystocia) is: a. Midplane contracture of the pelvis. b. Compromised bearing-down efforts as a result of pain medication. c. Disproportion of the pelvis. d. Low-lying placenta.

C The least common cause of dystocia is disproportion of the pelvis.

The nurse providing care to a woman in labor should understand that cesarean birth: a. Is declining in frequency in the twenty-first century in the United States. b. Is more likely to be performed for poor women in public hospitals who do not receive the nurse counseling as do wealthier clients. c. Is performed primarily for the benefit of the fetus. d. Can be either elected or refused by women as their absolute legal right.

C The most common indications for cesarean birth are danger to the fetus related to labor and birth complications. Cesarean births are increasing in the United States in this century. Wealthier women who have health insurance and who give birth in a private hospital are more likely to experience cesarean birth. A woman's right to elect cesarean surgery is in dispute, as is her right to refuse it if in doing so she endangers the fetus. Legal issues are not absolutely clear.

With regard to factors that affect how the fetus moves through the birth canal, nurses should be aware that: a. The fetal attitude describes the angle at which the fetus exits the uterus. b. Of the two primary fetal lies, the horizontal lie is that in which the long axis of the fetus is parallel to the long axis of the mother. c. The normal attitude of the fetus is called general flexion. d. The transverse lie is preferred for vaginal birth.

C The normal attitude of the fetus is general flexion. The fetal attitude is the relation of fetal body parts to one another. The horizontal lie is perpendicular to the mother; in the longitudinal (or vertical) lie the long axes of the fetus and the mother are parallel. Vaginal birth cannot occur if the fetus stays in a transverse lie.

The uterine contractions of a woman early in the active phase of labor are assessed by an internal uterine pressure catheter (IUPC). The nurse notes that the intrauterine pressure at the peak of the contraction ranges from 65 to 70 mm Hg and the resting tone range is 6 to 10 mm Hg. The uterine contractions occur every 3 to 4 minutes and last an average of 55 to 60 seconds. On the basis of this information, the nurse should: a. Notify the woman's primary health care provider immediately. b. Prepare to administer an oxytocic to stimulate uterine activity. c. Document the findings because they reflect the expected contraction pattern for the active phase of labor. d. Prepare the woman for the onset of the second stage of labor.

C The nurse is responsible for monitoring the uterine contractions to ascertain whether they are powerful and frequent enough to accomplish the work of expelling the fetus and the placenta. In addition, the nurse would document these findings in the client's medical record. This labor pattern indicates that the client is in the active phase of the first stage of labor. Nothing indicates a need to notify the primary care provider at this time. Oxytocin augmentation is not needed for this labor pattern; this contraction pattern indicates adequate active labor. Her contractions eventually will become stronger, last longer, and come closer together during the transition phase of the first stage of labor. The transition phase precedes the second stage of labor, or delivery of the fetus.

Which action is correct when palpation is used to assess the characteristics and pattern of uterine contractions? a. Place the hand on the abdomen below the umbilicus and palpate uterine tone with the fingertips. b. Determine the frequency by timing from the end of one contraction to the end of the next contraction. c. Evaluate the intensity by pressing the fingertips into the uterine fundus. d. Assess uterine contractions every 30 minutes throughout the first stage of labor.

C The nurse or primary care provider may assess uterine activity by palpating the fundal section of the uterus using the fingertips. Many women may experience labor pain in the lower segment of the uterus that may be unrelated to the firmness of the contraction detectable in the uterine fundus. The frequency of uterine contractions is determined by palpating from the beginning of one contraction to the beginning of the next contraction. Assessment of uterine activity is performed in intervals based on the stage of labor. As labor progresses this assessment is performed more frequently.

With regard to the turns and other adjustments of the fetus during the birth process, known as the mechanism of labor, nurses should be aware that: a. The seven critical movements must progress in a more or less orderly sequence. b. Asynclitism sometimes is achieved by means of the Leopold maneuver. c. The effects of the forces determining descent are modified by the shape of the woman's pelvis and the size of the fetal head. d. At birth the baby is said to achieve "restitution" (i.e., a return to the C-shape of the womb).

C The size of the maternal pelvis and the ability of the fetal head to mold also affect the process. The seven identifiable movements of the mechanism of labor occur in combinations simultaneously, not in precise sequences. Asynclitism is the deflection of the baby's head; the Leopold maneuver is a means of judging descent by palpating the mother's abdomen. Restitution is the rotation of the baby's head after the infant is born.

What is an advantage of external electronic fetal monitoring? a. The ultrasound transducer can accurately measure short-term variability and beat-to-beat changes in the fetal heart rate. b. The tocotransducer can measure and record the frequency, regularity, intensity, and approximate duration of uterine contractions (UCs). c. The tocotransducer is especially valuable for measuring uterine activity during the first stage of labor. d. Once correctly applied by the nurse, the transducer need not be repositioned even when the woman changes positions.

C The tocotransducer is especially valuable for measuring uterine activity during the first stage of labor, particularly when the membranes are intact. Short-term changes cannot be measured with this technology. The tocotransducer cannot measure and record the intensity of UCs. The transducer must be repositioned when the woman or fetus changes position.

The maternity nurse understands that as the uterus contracts during labor, maternal-fetal exchange of oxygen and waste products: a. Continues except when placental functions are reduced. b. Increases as blood pressure decreases. c. Diminishes as the spiral arteries are compressed. d. Is not significantly affected.

C Uterine contractions during labor tend to decrease circulation through the spiral electrodes and subsequent perfusion through the intervillous space. The maternal blood supply to the placenta gradually stops with contractions. The exchange of oxygen and waste products decreases. The exchange of oxygen and waste products is affected by contractions.

The nurse providing care for the laboring woman should understand that late fetal heart rate (FHR) decelerations are the result of: a. Altered cerebral blood flow. c. Uteroplacental insufficiency. b. Umbilical cord compression. d. Meconium fluid.

C Uteroplacental insufficiency would result in late decelerations in the FHR. Altered fetal cerebral blood flow would result in early decelerations in the FHR. Umbilical cord compression would result in variable decelerations in the FHR. Meconium-stained fluid may or may not produce changes in the fetal heart rate, depending on the gestational age of the fetus and whether other causative factors associated with fetal distress are present.

A woman is having her first child. She has been in labor for 15 hours. Two hours ago her vaginal examination revealed the cervix to be dilated to 5 cm and 100% effaced, and the presenting part was at station 0. Five minutes ago her vaginal examination indicated that there had been no change. What abnormal labor pattern is associated with this description? a. Prolonged latent phase c. Arrest of active phase b. Protracted active phase d. Protracted descent

C With an arrest of the active phase, the progress of labor has stopped. This client has not had any anticipated cervical change, thus indicating an arrest of labor. In the nulliparous woman a prolonged latent phase typically would last more than 20 hours. A protracted active phase, the first or second stage of labor, would be prolonged (slow dilation). With protracted descent, the fetus would fail to descend at an anticipated rate during the deceleration phase and second stage of labor.

A primigravida at 40 weeks of gestation is having uterine contractions every 1.5 to 2 minutes and says that they are very painful. Her cervix is dilated 2 cm and has not changed in 3 hours. The woman is crying and wants an epidural. What is the likely status of this woman's labor? a. She is exhibiting hypotonic uterine dysfunction. b. She is experiencing a normal latent stage. c. She is exhibiting hypertonic uterine dysfunction. d. She is experiencing pelvic dystocia.

C Women who experience hypertonic uterine dysfunction, or primary dysfunctional labor, often are anxious first-time mothers who are having painful and frequent contractions that are ineffective at causing cervical dilation or effacement to progress. With hypotonic uterine dysfunction, the woman initially makes normal progress into the active stage of labor; then the contractions become weak and inefficient or stop altogether. The contraction pattern seen in this woman signifies hypertonic uterine activity. Typically uterine activity in this phase occurs at 4- to 5-minute intervals lasting 30 to 45 seconds. Pelvic dystocia can occur whenever contractures of the pelvic diameters reduce the capacity of the bony pelvis, including the inlet, midpelvis, outlet, or any combination of these planes.

The baseline fetal heart rate (FHR) is the average rate during a 10-minute segment. Changes in FHR are categorized as periodic or episodic. These patterns include both accelerations and decelerations. The labor nurse is evaluating the patient's most recent 10-minute segment on the monitor strip and notes a late deceleration. This is likely to be caused by which physiologic alteration (Select all that apply)? a. Spontaneous fetal movement b. Compression of the fetal head c. Placental abruption d. Cord around the baby's neck e. Maternal supine hypotension

C, E Late decelerations are almost always caused by uteroplacental insufficiency. Insufficiency is caused by uterine tachysystole, maternal hypotension, epidural or spinal anesthesia, IUGR, intraamniotic infection, or placental abruption. Spontaneous fetal movement, vaginal examination, fetal scalp stimulation, fetal reaction to external sounds, uterine contractions, fundal pressure and abdominal palpation are all likely to cause accelerations of the FHR. Early decelerations are most often the result of fetal head compression and may be caused by uterine contractions, fundal pressure, vaginal examination, and placement of an internal electrode. A variable deceleration is likely caused by umbilical cord compression. This may happen when the umbilical cord is around the baby's neck, arm, leg, or other body part or when there is a short cord, a knot in the cord, or a prolapsed cord.

Maternal and neonatal risks associated with gestational diabetes mellitus are: A. maternal premature rupture of membranes and neonatal sepsis. B. maternal hyperemesis and neonatal low birth weight. C. Maternal preeclampsia and fetal macrosomia. D. maternal placenta previa and fetal prematurity.

C. Maternal preeclampsia and fetal macrosomnia Premature rupture of membranes and neonatal sepsis are not risks associated with gestational diabetes. Hyperemesis is not seen with gestational diabetes, nor is there an association with low birth weight of the infant. Women with gestational diabetes have twice the risk of developing hypertensive disorders such as preeclampsia, and the baby usually has macrosomia. Placental previa and subsequent prematurity of the neonate are not risks associated with gestational diabetes.

During pregnancy, alcohol withdrawal may be treated using: A. disulfiram (Antabuse). B. corticosteroids. C. benzodiazepines. E. aminophylline.

C. benzodiazepines. Disulfiram is contraindicated in pregnancy because it is teratogenic. Corticosteroids are not used to treat alcohol withdrawal. Symptoms that occur during alcohol withdrawal can be managed with short-acting barbiturates or benzodiazepines. Aminophylline is not used to treat alcohol withdrawal.

Which presentation is described accurately in terms of both presenting part and frequency of occurrence?

Cephalic: occiput; at least 95%

To teach patients about the process of labor adequately, the nurse knows that which event is the best indicator of true labor?

Cervical dilation and effacement

In order to care for obstetric patients adequately, the nurse understands that labor contractions facilitate cervical dilation by: a. Contracting the lower uterine segment. b. Enlarging the internal size of the uterus. c. Promoting blood flow to the cervix. d. Pulling the cervix over the fetus and amniotic sac.

D Effective uterine contractions pull the cervix upward at the same time that the fetus and amniotic sac are pushed downward. The contractions are stronger at the fundus. The internal size becomes smaller with the contractions; this helps to push the fetus down. Blood flow decreases to the uterus during a contraction.

Which basic type of pelvis includes the correct description and percentage of occurrence in women? a. Gynecoid: classic female; heart shaped; 75% b. Android: resembling the male; wider oval; 15% c. Anthropoid: resembling the ape; narrower; 10% d. Platypelloid: flattened, wide, shallow; 3%

D A platypelloid pelvis is flattened, wide, and shallow; about 3% of women have this shape. The gynecoid shape is the classical female shape, slightly ovoid and rounded; about 50% of women have this shape. An android, or malelike, pelvis is heart shaped; about 23% of women have this shape. An anthropoid, or apelike, pelvis is oval and wider; about 24% of women have this shape.

The most common cause of decreased variability in the fetal heart rate (FHR) that lasts 30 minutes or less is: a. Altered cerebral blood flow. c. Umbilical cord compression. b. Fetal hypoxemia. d. Fetal sleep cycles.

D A temporary decrease in variability can occur when the fetus is in a sleep state. These sleep states do not usually last longer than 30 minutes. Altered fetal cerebral blood flow would result in early decelerations in the FHR. Fetal hypoxemia would be evidenced by tachycardia initially and then bradycardia. A persistent decrease or loss of FHR variability may be seen. Umbilical cord compression would result in variable decelerations in the FHR.

When a nulliparous woman telephones the hospital to report that she is in labor, the nurse initially should: a. Tell the woman to stay home until her membranes rupture. b. Emphasize that food and fluid intake should stop. c. Arrange for the woman to come to the hospital for labor evaluation. d. Ask the woman to describe why she believes she is in labor.

D Assessment begins at the first contact with the woman, whether by telephone or in person. By asking the woman to describe her signs and symptoms, the nurse can begin the assessment and gather data. The amniotic membranes may or may not spontaneously rupture during labor. The client may be instructed to stay home until the uterine contractions become strong and regular. The nurse may want to discuss the appropriate oral intake for early labor such as light foods or clear liquids, depending on the preference of the client or her primary health care provider. Before instructing the woman to come to the hospital, the nurse should initiate the assessment during the telephone interview.

A woman at 26 weeks of gestation is being assessed to determine whether she is experiencing preterm labor. What finding indicates that preterm labor is occurring? a. Estriol is not found in maternal saliva. b. Irregular, mild uterine contractions are occurring every 12 to 15 minutes. c. Fetal fibronectin is present in vaginal secretions. d. The cervix is effacing and dilated to 2 cm.

D Cervical changes such as shortened endocervical length, effacement, and dilation are predictors of imminent preterm labor. Changes in the cervix accompanied by regular contractions indicate labor at any gestation. Estriol is a form of estrogen produced by the fetus that is present in plasma at 9 weeks of gestation. Levels of salivary estriol have been shown to increase before preterm birth. Irregular, mild contractions that do not cause cervical change are not considered a threat. The presence of fetal fibronectin in vaginal secretions between 24 and 36 weeks of gestation could predict preterm labor, but it has only a 20% to 40% positive predictive value. Of more importance are other physiologic clues of preterm labor such as cervical changes.

The factors that affect the process of labor and birth, known commonly as the five Ps, include all except: a. Passenger. b. Powers. c. Passageway. d. Pressure.

D The five Ps are passenger (fetus and placenta), passageway (birth canal), powers (contractions), position of the mother, and psychologic response.

When assessing the relative advantages and disadvantages of internal and external electronic fetal monitoring, nurses comprehend that both: a. Can be used when membranes are intact. b. Measure the frequency, duration, and intensity of uterine contractions. c. May need to rely on the woman to indicate when uterine activity (UA) is occurring. d. Can be used during the antepartum and intrapartum periods.

D External monitoring can be used in both periods; internal monitoring can be used only in the intrapartum period. For internal monitoring the membranes must have ruptured, and the cervix must be sufficiently dilated. Internal monitoring measures the intensity of contractions; external monitoring cannot do this. With external monitoring, the woman may need to alert the nurse that UA is occurring; internal monitoring does not require this.

Increasing the infusion rate of nonadditive intravenous fluids can increase fetal oxygenation primarily by: a. Maintaining normal maternal temperature. b. Preventing normal maternal hypoglycemia. c. Increasing the oxygen-carrying capacity of the maternal blood. d. Expanding maternal blood volume.

D Filling the mother's vascular system makes more blood available to perfuse the placenta and may correct hypotension. Increasing fluid volume may alter the maternal temperature only if she is dehydrated. Most intravenous fluids for laboring women are isotonic and do not provide extra glucose. Oxygen-carrying capacity is increased by adding more red blood cells.

With regard to the care management of preterm labor, nurses should be aware that: a. Because all women must be considered at risk for preterm labor and prediction is so hit-and-miss, teaching pregnant women the symptoms probably causes more harm through false alarms. b. Braxton Hicks contractions often signal the onset of preterm labor. c. Because preterm labor is likely to be the start of an extended labor, a woman with symptoms can wait several hours before contacting the primary caregiver. d. The diagnosis of preterm labor is based on gestational age, uterine activity, and progressive cervical change.

D Gestational age of 20 to 37 weeks, uterine contractions, and a cervix that is 80% effaced or dilated 2 cm indicates preterm labor. It is essential that nurses teach women how to detect the early symptoms of preterm labor. Braxton Hicks contractions resemble preterm labor contractions, but they are not true labor. Waiting too long to see a health care provider could result in not administering essential medications. Preterm labor is not necessarily long-term labor.

Which fetal heart rate (FHR) finding would concern the nurse during labor? a. Accelerations with fetal movement c. An average FHR of 126 beats/min b. Early decelerations d. Late decelerations

D Late decelerations are caused by uteroplacental insufficiency and are associated with fetal hypoxemia. They are considered ominous if persistent and uncorrected. Accelerations in the FHR are an indication of fetal well-being. Early decelerations in the FHR are associated with head compression as the fetus descends into the maternal pelvic outlet; they generally are not a concern during normal labor.

Nurses should be aware that the induction of labor: a. Can be achieved by external and internal version techniques. b. Is also known as a trial of labor (TOL). c. Is almost always done for medical reasons. d. Is rated for viability by a Bishop score.

D Induction of labor is likely to be more successful with a Bishop score of 9 or higher for first-time mothers and 5 or higher for veterans. Version is turning of the fetus to a better position by a physician for an easier or safer birth. A trial of labor is the observance of a woman and her fetus for several hours of active labor to assess the safety of vaginal birth. Two thirds of cases of induced labor are elective and are not done for medical reasons.

The standard of care for obstetrics dictates that an internal version may be used to manipulate the: a. Fetus from a breech to a cephalic presentation before labor begins. b. Fetus from a transverse lie to a longitudinal lie before cesarean birth. c. Second twin from an oblique lie to a transverse lie before labor begins. d. Second twin from a transverse lie to a breech presentation during vaginal birth.

D Internal version is used only during vaginal birth to manipulate the second twin into a presentation that allows it to be born vaginally. For internal version to occur, the cervix needs to be completely dilated.

For the labor nurse, care of the expectant mother begins with any or all of these situations, with the exception of: a. The onset of progressive, regular contractions. b. The bloody, or pink, show. c. The spontaneous rupture of membranes. d. Formulation of the woman's plan of care for labor.

D Labor care begins when progressive, regular contractions begin; the blood-tinged mucoid vaginal discharge appears; or fluid is discharged from the vagina. The woman and nurse can formulate their plan of care before labor or during treatment.

If a woman complains of back labor pain, the nurse could best suggest that she: a. Lie on her back for a while with her knees bent. b. Do less walking around. c. Take some deep, cleansing breaths. d. Lean over a birth ball with her knees on the floor.

D The hands-and-knees position, with or without the aid of a birth ball, should help with the back pain. The supine position should be discouraged. Walking generally is encouraged.

When using intermittent auscultation (IA) for fetal heart rate, nurses should be aware that: a. They can be expected to cover only two or three clients when IA is the primary method of fetal assessment. b. The best course is to use the descriptive terms associated with electronic fetal monitoring (EFM) when documenting results. c. If the heartbeat cannot be found immediately, a shift must be made to EFM. d. Ultrasound can be used to find the fetal heartbeat and reassure the mother if initial difficulty was a factor.

D Locating fetal heartbeats often takes time. Mothers can be reassured verbally and by the ultrasound pictures if ultrasound is used to help locate the heartbeat. When used as the primary method of fetal assessment, auscultation requires a nurse-to-client ratio of one to one. Documentation should use only terms that can be numerically defined; the usual visual descriptions of EFM are inappropriate.

As a perinatal nurse you realize that a fetal heart rate that is tachycardic, is bradycardic, or has late decelerations or loss of variability is nonreassuring and is associated with: a. Hypotension. c. Maternal drug use. b. Cord compression. d. Hypoxemia.

D Nonreassuring heart rate patterns are associated with fetal hypoxemia. Fetal bradycardia may be associated with maternal hypotension. Fetal variable decelerations are associated with cord compression. Maternal drug use is associated with fetal tachycardia.

The priority nursing intervention after an amniotomy should be to: a. Assess the color of the amniotic fluid. b. Change the patient's gown. c. Estimate the amount of amniotic fluid. d. Assess the fetal heart rate.

D The fetal heart rate must be assessed immediately after the rupture of the membranes to determine whether cord prolapse or compression has occurred. Secondary to FHR assessment, amniotic fluid amount, color, odor, and consistency is assessed. Dry clothing is important for patient comfort; however, it is not the top priority.

Which assessment is least likely to be associated with a breech presentation? a. Meconium-stained amniotic fluid b. Fetal heart tones heard at or above the maternal umbilicus c. Preterm labor and birth d. Post-term gestation

D Post-term gestation is not likely to be seen with a breech presentation. The presence of meconium in a breech presentation may result from pressure on the fetal wall as it traverses the birth canal. Fetal heart tones heard at the level of the umbilical level of the mother are a typical finding in a breech presentation because the fetal back would be located in the upper abdominal area. Breech presentations often occur in preterm births.

Which collection of risk factors most likely would result in damaging lacerations (including episiotomies)? a. A dark-skinned woman who has had more than one pregnancy, who is going through prolonged second-stage labor, and who is attended by a midwife b. A reddish-haired mother of two who is going through a breech birth c. A dark-skinned, first-time mother who is going through a long labor d. A first-time mother with reddish hair whose rapid labor was overseen by an obstetrician

D Reddish-haired women have tissue that is less distensible than that of darker-skinned women and therefore may have less efficient healing. First time mothers are also more at risk, especially with breech births, long second-stage labors, or rapid labors in which there is insufficient time for the perineum to stretch. The rate of episiotomies is higher when obstetricians rather than midwives attend births.

For a woman at 42 weeks of gestation, which finding would require further assessment by the nurse? a. Fetal heart rate of 116 beats/min b. Cervix dilated 2 cm and 50% effaced c. Score of 8 on the biophysical profile d. One fetal movement noted in 1 hour of assessment by the mother

D Self-care in a post-term pregnancy should include performing daily fetal kick counts three times per day. The mother should feel four fetal movements per hour. If fewer than four movements have been felt by the mother, she should count for 1 more hour. Fewer than four movements in that hour warrants evaluation. Normal findings in a 42-week gestation include fetal heart rate of 116 beats/min, cervix dilated 20 cm and 50% effaced, and a score of 8 on the biophysical profile.

While evaluating an external monitor tracing of a woman in active labor whose labor is being induced, the nurse notes that the fetal heart rate (FHR) begins to decelerate at the onset of several contractions and returns to baseline before each contraction ends. The nurse should: a. Change the woman's position. b. Discontinue the oxytocin infusion. c. Insert an internal monitor. d. Document the finding in the client's record.

D The FHR indicates early decelerations, which are not an ominous sign and do not require any intervention. The nurse should simply document these findings.

In order to evaluate the condition of the patient accurately during labor, the nurse should be aware that: a. The woman's blood pressure will increase during contractions and fall back to prelabor normal between contractions. b. Use of the Valsalva maneuver is encouraged during the second stage of labor to relieve fetal hypoxia. c. Having the woman point her toes will reduce leg cramps. d. The endogenous endorphins released during labor will raise the woman's pain threshold and produce sedation.

D The endogenous endorphins released during labor will raise the woman's pain threshold and produce sedation. In addition, physiologic anesthesia of the perineal tissues, caused by the pressure of the presenting part, decreases the mother's perception of pain. Blood pressure increases during contractions but remains somewhat elevated between them. Use of the Valsalva maneuver is discouraged during second stage labor because of a number of unhealthy outcomes, including fetal hypoxia. Pointing the toes can cause leg cramps, as can the process of labor itself.

A new mother asks the nurse when the "soft spot" on her son's head will go away. The nurse's answer is based on the knowledge that the anterior fontanel closes after birth by _____ months. a. 2 b. 12 c. 8 d. 18

D The larger of the two fontanels, the anterior fontanel, closes by 18 months after birth.

The nurse who performs vaginal examinations to assess a woman's progress in labor should: a. Perform an examination at least once every hour during the active phase of labor. b. Perform the examination with the woman in the supine position. c. Wear two clean gloves for each examination. d. Discuss the findings with the woman and her partner.

D The nurse should discuss the findings of the vaginal examination with the woman and her partner and report them to the primary care provider. A vaginal examination should be performed only when indicated by the status of the woman and her fetus. The woman should be positioned to avoid supine hypotension. The examiner should wear a sterile glove while performing a vaginal examination for a laboring woman.

The nurse practicing in a labor setting knows that the woman most at risk for uterine rupture is: a. A gravida 3 who has had two low-segment transverse cesarean births. b. A gravida 2 who had a low-segment vertical incision for delivery of a 10-pound infant. c. A gravida 5 who had two vaginal births and two cesarean births. d. A gravida 4 who has had all cesarean births.

D The risk of uterine rupture increases for the patient who has had multiple prior births with no vaginal births. As the number of prior uterine incisions increases, so does the risk for uterine rupture. Low-segment transverse cesarean scars do not predispose the patient to uterine rupture.

A nurse may be called on to stimulate the fetal scalp: a. As part of fetal scalp blood sampling. b. In response to tocolysis. c. In preparation for fetal oxygen saturation monitoring. d. To elicit an acceleration in the fetal heart rate (FHR).

D The scalp can be stimulated using digital pressure during a vaginal examination. Fetal scalp blood sampling involves swabbing the scalp with disinfectant before a sample is collected. The nurse would stimulate the fetal scalp to elicit an acceleration of the FHR. Tocolysis is relaxation of the uterus. Fetal oxygen saturation monitoring involves the insertion of a sensor.

In evaluating the effectiveness of magnesium sulfate for the treatment of preterm labor, what finding would alert the nurse to possible side effects? a. Urine output of 160 mL in 4 hours b. Deep tendon reflexes 2+ and no clonus c. Respiratory rate of 16 breaths/min d. Serum magnesium level of 10 mg/dL

D The therapeutic range for magnesium sulfate management is 5 to 8 mg/dL. A serum magnesium level of 10 mg/dL could lead to signs and symptoms of magnesium toxicity, including oliguria and respiratory distress. Urine output of 160 mL in 4 hours, deep tendon reflexes 2+ with no clonus, and respiratory rate of 16 breaths/min are normal findings.

A multiparous woman has been in labor for 8 hours. Her membranes have just ruptured. The nurse's initial response would be to: a. Prepare the woman for imminent birth. b. Notify the woman's primary health care provider. c. Document the characteristics of the fluid. d. Assess the fetal heart rate and pattern.

D The umbilical cord may prolapse when the membranes rupture. The fetal heart rate and pattern should be monitored closely for several minutes immediately after ROM to ascertain fetal well-being, and the findings should be documented. Rupture of membranes (ROM) may increase the intensity and frequency of the uterine contractions, but it does not indicate that birth is imminent. The nurse may notify the primary care provider after ROM occurs and fetal well-being and the response to ROM have been assessed. The nurse's priority is to assess fetal well-being. The nurse should document the characteristics of the amniotic fluid, but the initial response is to assess fetal well-being and the response to ROM.

A primigravida at 39 weeks of gestation is observed for 2 hours in the intrapartum unit. The fetal heart rate has been normal. Contractions are 5 to 9 minutes apart, 20 to 30 seconds in duration, and of mild intensity. Cervical dilation is 1 to 2 cm and uneffaced (unchanged from admission). Membranes are intact. The nurse should expect the woman to be: a. Admitted and prepared for a cesarean birth. b. Admitted for extended observation. c. Discharged home with a sedative. d. Discharged home to await the onset of true labor.

D This situation describes a woman with normal assessments who is probably in false labor and will likely not deliver rapidly once true labor begins. There is no indication that further assessments or observations are indicated; therefore, the patient will be discharged along with instructions to return when contractions increase in intensity and frequency. Neither a cesarean birth nor a sedative is required at this time.

A primigravida at 39 weeks of gestation is observed for 2 hours in the intrapartum unit. The fetal heart rate has been normal. Contractions are 5 to 9 minutes apart, 20 to 30 seconds in duration, and of mild intensity. Cervical dilation is 1 to 2 cm and uneffaced (unchanged from admission). Membranes are intact. The nurse should expect the woman to be: a. Admitted and prepared for a cesarean birth. b. Admitted for extended observation. c. Discharged home with a sedative. d. Discharged home to await the onset of true labor.

D This situation describes a woman with normal assessments who is probably in false labor and will probably not deliver rapidly once true labor begins. These are all indications of false labor without fetal distress. There is no indication that further assessment or cesarean birth is indicated. The patient will likely be discharged; however, there is no indication that a sedative is needed.

You are evaluating the fetal monitor tracing of your client, who is in active labor. Suddenly you see the fetal heart rate (FHR) drop from its baseline of 125 down to 80. You reposition the mother, provide oxygen, increase intravenous (IV) fluid, and perform a vaginal examination. The cervix has not changed. Five minutes have passed, and the fetal heart rate remains in the 80s. What additional nursing measures should you take? a. Scream for help. b. Insert a Foley catheter. c. Start Pitocin. d. Notify the care provider immediately.

D To relieve an FHR deceleration, the nurse can reposition the mother, increase IV fluid, and provide oxygen. If oxytocin is infusing, it should be discontinued. If the FHR does not resolve, the primary care provider should be notified immediately. Inserting a Foley catheter is an inappropriate nursing action. If the FHR were to continue in a nonreassuring pattern, a cesarean section could be warranted, which would require a Foley catheter. However, the physician must make that determination. Pitocin may place additional stress on the fetus.

In documenting labor experiences, nurses should know that a uterine contraction is described according to all these characteristics except: a. Frequency (how often contractions occur). b. Intensity (the strength of the contraction at its peak). c. Resting tone (the tension in the uterine muscle). d. Appearance (shape and height).

D Uterine contractions are described in terms of frequency, intensity, duration, and resting tone.

During a physical assessment of an at-risk client, the nurse notes generalized edema, crackles at the base of the lungs, and some pulse irregularity. These are most likely signs of: A. euglycemia. B. rheumatic fever. C. pneumonia. D. cardiac decompensation.

D. cardiac decompensation. Euglycemia is a condition of normal glucose levels. These symptoms indicate cardiac decompensation. Rheumatic fever can cause heart problems, but it does not present with these symptoms, which indicate cardiac decompensation. Pneumonia is an inflammation of the lungs and would not likely generate these symptoms, which indicate cardiac decompensation. Symptoms of cardiac decompensation may appear abruptly or gradually.

In planning for the care of a 30-year-old woman with pregestational diabetes, the nurse recognizes that the most important factor affecting pregnancy outcome is the: A. mother's age. B. number of years since diabetes was diagnosed. C. amount of insulin required prenatally. D. degree of glycemic control during pregnancy.

D. degree of glycemic control during pregnancy. Although advanced maternal age may pose some health risks, for the woman with pregestational diabetes the most important factor remains the degree of glycemic control during pregnancy. The number of years since diagnosis is not as relevant to outcomes as the degree of glycemic control. The key to reducing risk in the pregestational diabetic woman is not the amount of insulin required but rather the level of glycemic control. Women with excellent glucose control and no blood vessel disease should have good pregnancy outcomes.

A pregnant woman at 28 weeks of gestation has been diagnosed with gestational diabetes. The nurse caring for this client understands that: A. oral hypoglycemic agents can be used if the woman is reluctant to give herself insulin. B. dietary modifications and insulin are both required for adequate treatment. C. glucose levels are monitored by testing urine 4r times a day and at bedtime. D. dietary management involves distributing nutrient requirements over three meals and two or three snacks.

D. dietary management involves distributing nutrient requirements over three meals and two or three snacks. Oral hypoglycemic agents can be harmful to the fetus and less effective than insulin in achieving tight glucose control. In some women gestational diabetes can be controlled with dietary modifications alone. Blood, not urine, glucose levels are monitored several times a day. Urine is tested for ketone content; results should be negative. Small frequent meals over a 24-hour period help decrease the risk for hypoglycemia and ketoacidosis.

A primigravida at 39 weeks of gestation is observed for 2 hours in the intrapartum unit. The fetal heart rate has been normal. Contractions are 5 to 9 minutes apart, 20 to 30 seconds in duration, and of mild intensity. Cervical dilation is 1 to 2 cm and uneffaced (unchanged from admission). Membranes are intact. The nurse should expect the woman to be:

Discharged home to await the onset of true labor.

The nurse who performs vaginal examinations to assess a woman's progress in labor should:

Discuss the findings with the woman and her partner.

Which description of the four stages of labor is correct for both definition and duration?

First stage: onset of regular uterine contractions to full dilation; less than 1 hour to 20 hours

If a woman complains of back labor pain, the nurse could best suggest that she:

Lean over a birth ball with her knees on the floor.

A nulliparous woman who has just begun the second stage of her labor would most likely:

Feel tired yet relieved that the worst is over.

A woman who is gravida 3 para 2 enters the intrapartum unit. The most important nursing assessments are:

Fetal heart rate, maternal vital signs, and the woman's nearness to birth.

To care for a laboring woman adequately, the nurse understands that the __________ stage of labor varies the most in length.

First [The first stage of labor is considered to last from the onset of regular uterine contractions to full dilation of the cervix. The first stage is much longer than the second and third stages combined.]

Through vaginal examination the nurse determines that a woman is 4 cm dilated, and the external fetal monitor shows uterine contractions every 3.5 to 4 minutes. The nurse would report this as:

First stage, active phase.

For the labor nurse, care of the expectant mother begins with any or all of these situations, with the exception of:

Formulation of the woman's plan of care for labor.

While providing care to a patient in active labor, the nurse should instruct the woman that:

Frequent changes in position will help relieve her fatigue and increase her comfort.

Leopold maneuvers would be an inappropriate method of assessment to determine:

Gender of the fetus.

Because the risk for childbirth complications may be revealed, nurses should know that the point of maximal intensity (PMI) of the fetal heart tone (FHT) is:

Heard lower and closer to the midline of the mother's abdomen as the fetus descends and rotates internally.

What is an essential part of nursing care for the laboring woman?

Helping the woman manage the pain

The nurse would expect which maternal cardiovascular finding during labor?

Increased cardiac output

The most critical nursing action in caring for the newborn immediately after birth is:

Keeping the newborn's airway clear. [The care given immediately after the birth focuses on assessing and stabilizing the newborn.]

For women who have a history of sexual abuse, a number of traumatic memories may be triggered during labor. The woman may fight the labor process and react with pain or anger. Alternately, she may become a passive player and emotionally absent herself from the process. The nurse is in a unique position of being able to assist the client to associate the sensations of labor with the process of childbirth and not the past abuse. The nurse can implement a number of care measures to help the client view the childbirth experience in a positive manner. Which intervention would be key for the nurse to use while providing care?

Limiting the number of procedures that invade her body

In order to care for obstetric patients adequately, the nurse understands that labor contractions facilitate cervical dilation by:

Pulling the cervix over the fetus and amniotic sac.

When assessing the fetus using Leopold maneuvers, the nurse feels a round, firm, movable fetal part in the fundal portion of the uterus and a long, smooth surface in the mother's right side close to midline. What is the likely position of the fetus?

RSA

The nurse thoroughly dries the infant immediately after birth primarily to:

Reduce heat loss from evaporation.

Women who have participated in childbirth education classes often bring a "birth bag" or "Lamaze bag" with them to the hospital. These items often assist in reducing stress and providing comfort measures. The nurse caring for women in labor should be aware of common items that a client may bring, including:

Rolling pin. Tennis balls. Pillow. Stuffed animal or photo. ?????????

Steps of Vaginal Exam

Step 1. Use sterile gloves and soluble gel for lubrication. Step 2. Position the woman to prevent supine hypotension. Step 3. Cleanse the perineum and vulva if necessary. Step 4. After obtaining permission, gently insert the index and middle fingers into the vagina. Step 5. Determine dilation, presenting part, status of membranes, and characteristics of amniotic fluid. Step 6. Explain findings to the patient. Step 7. Document findings and report to the provider.

After an emergency birth, the nurse encourages the woman to breastfeed her newborn. The primary purpose of this activity is to:

Stimulate the uterus to contract [Stimulation of the nipples through breastfeeding or manual stimulation causes the release of oxytocin and prevents maternal hemorrhage.]

Which factors influence cervical dilation?

Strong uterine contractions The force of the presenting fetal part against the cervix The pressure applied by the amniotic sac Scarring of the cervix

A means of controlling the birth of the fetal head with a vertex presentation is:

The Ritgen maneuver. [The Ritgen maneuver extends the head during the actual birth and protects the perineum. Gentle, steady pressure against the fundus of the uterus facilitates vaginal birth.]

The primary difference between the labor of a nullipara and that of a multipara is the:

Total duration of labor.

The nurse has received report regarding her patient in labor. The woman's last vaginal examination was recorded as 3 cm, 30%, and -2. The nurse's interpretation of this assessment is that:

The cervix is 3 cm dilated, it is effaced 30%, and the presenting part is 2 cm above the ischial spines.

35. A pregnant patient would like to know a good food source of calcium other than dairy products. Your best answer is: a. Legumes b. Yellow vegetables c. Lean meat d. Whole grains

a. Legumes

Nurses alert to signs of the onset of the second stage of labor can be certain that this stage has begun when:

The nurse is unable to feel the cervix during a vaginal examination. [The only certain objective sign that the second stage has begun is the inability to feel the cervix because it is fully dilated and effaced.]

With regard to the turns and other adjustments of the fetus during the birth process, known as the mechanism of labor, nurses should be aware that:

The effects of the forces determining descent are modified by the shape of the woman's pelvis and the size of the fetal head.

In order to evaluate the condition of the patient accurately during labor, the nurse should be aware that:

The endogenous endorphins released during labor will raise the woman's pain threshold and produce sedation.

It is paramount for the obstetric nurse to understand the regulatory procedures and criteria for admitting a woman to the hospital labor unit. Which guideline is an important legal requirement of maternity care?

The patient's weight gain is calculated to determine whether she is at greater risk for cephalopelvic disproportion (CPD) and cesarean birth.

When assessing a multiparous woman who has just given birth to an 8 lb boy, nurse notes that the woman's fundus is firm and has become globular in shape. A gush of dark red blood comes from her vagina. nurse concludes that:

The placenta has separated.

Which nursing assessment indicates that a woman who is in second-stage labor is almost ready to give birth?

The vulva bulges and encircles the fetal head.

The nurse knows that the second stage of labor, the descent phase, has begun when:

The woman experiences a strong urge to bear down.

Under which circumstance would it be unnecessary for the nurse to perform a vaginal examination?

When accelerations of the fetal heart rate (FHR) are noted [An accelerated FHR is a positive sign; however, variable decelerations merit a vaginal examination]

Within the first hour after birth, the nurse would expect to find the woman's fundus: a) At the level of the umbilicus b) One fingerbreadth below the umbilicus c) Between the umbilicus and symphysis pubis d) 2 cm above the umbilicus

a) At the level of the umbilicus Rationale: After birth, the fundus is located midline between the umbilicus and symphysis pubis but then slowly rises to the level of the umbilicus during the first hour after birth. Then the uterus contracts, approximately 1 cm (or fingerbreadth) each day after birth.

With regard to breathing during labor, maternity nurses should be aware that: a) Breathing techniques in the first stage of labor is designed to increase the size of the abdominal cavity to reduce friction b) By the time labor has begun, it is too late for instruction in breathing and relaxation c) Controlled breathing techniques are most difficult near the end of the second stage of labor d) The patterned-paced breathing technique can help prevent hyperventilation

a) Breathing techniques in the first stage of labor is designed to increase the size of the abdominal cavity to reduce friction

A fetus is in the LST position. The nurse interprets this as indicating which of the following as the presenting part? a) Buttocks b) Fetal head c) Shoulder d) Chin

a) Buttocks Rationale: The letter "S" indicates the sacrum or buttocks as the presenting part. The fetal head would be noted by the letter "O," indicating occiput. The fetal chin would be noted by the letter "M," indicating mentum. The fetal shoulder would be noted by the letter "A," indicating the acromion process.

A petite, 5-foot tall, 95-pound woman who is 28 years old is about to deliver her first child and would like to have a vaginal delivery. She has two sisters, both of whom have given birth vaginally. She has gained 25 pounds during a normal, uneventful pregnancy. What type of pelvis would a nurse expect this woman to have upon assessment of the patient? a) Cannot be determined b) Android c) Platypelloid d) Gynecoid

a) Cannot be determined Rationale: Pelvis shape cannot be determined by the information included in the statement. Early in the pregnancy, particularly if a woman has never delivered a baby vaginally, the practitioner may take pelvic measurements to estimate the size of the true pelvis. This helps to determine if the size is adequate for vaginal delivery. However, these measurements do not consistently predict which women will have difficulty delivering vaginally, so most practitioners allow the woman to labor and attempt a vaginal birth.

A client in labor is agitated and nervous about the birth of her child. The nurse explains to the client that fear and anxiety cause the release of certain compounds which can prolong labor. Which of the following is the nurse referring to in the explanation? a) Catecholamines b) Relaxin c) Prostaglandins d) Oxytocin

a) Catecholamines Rationale: Fear and anxiety cause the release of catecholamines, such as norepinephrine and epinephrine which stimulate the adrenergic receptors of the myometrium. This in turn interferes with effective uterine contractions and results in prolonged labor. Estrogen promotes the release of prostaglandins and oxytocin. Relaxin is a hormone that is involved in producing backache by acting on the pelvic joints. Prostaglandins, oxytocin and relaxin are not produced due to fear or anxiety in clients during labor.

When performing a vaginal examinations on laboring women, the nurse should be guided by what principle? a) Cleanse the vulva and perineum before and after the examination as needed b) Wear a clean glove lubricated with tap water and reduce discomfort c) Perform the examination every hour during the active phase of the first stage of labor d) Perform an examination immediately if active bleeding is present

a) Cleanse the vulva and perineum before and after the examination as needed

A woman is experiencing back labor and complains of intense pain in her lower back. An effective relief measure is to use: a) Counterpressure against the sacrum b) Pant-blow (breaths and puffs) breathing techniques c) Effleurage d) Biofeedback

a) Counterpressure against the sacrum

The nurse notes that the fetal head is at the vaginal opening and does not regress between contractions. The nurse interprets this finding as which of the following? a) Crowning b) Engagement c) Descent d) Restitution

a) Crowning Rationale: Crowning occurs when the top of the fetal head appears at the vaginal orifice and no longer regresses between contractions. Engagement occurs when the greatest transverse diameter of the head passes through the pelvic inlet. Descent is the downward movement of the fetal head until it is within the pelvic inlet. Restitution or external rotation occurs after the head is born and free of resistance. It untwists, causing the occiput to move about 45 degrees back to its original left or right position.

When managing the care of a woman in the second stage of labor, the nurse uses various measures to enhance the progress of fetal descent. These measures include: a) Encouraging the woman to try various upright positions, including squatting and standing b) Telling the woman to start pushing as soon as her cervix is fully dilated c) Continuing on epidural anesthetic so that pain is reduced and the woman can relax d) Coaching the woman to use sustained 10 to 15 second, closed-glottis bearing-down efforts with each contraction

a) Encouraging the woman to try various upright positions, including squatting and standing

The initial descent of the fetus into the pelvis to zero station is which one of the cardinal movements of labor? a) Engagement b) Felxion c) Extension d) Expulsion

a) Engagement Rationale: The movement of the fetus into the pelvis from the upper uterus is engagement. This is the first cardinal movement of the fetus in preparation for the spontaneous vaginal delivery. Flexion occurs as the fetus encounters resistance from the soft tissues and muscles of the pelvic floor. Extension is the state in which the fetal head is well flexed with the chin on the chest as the fetus travels through the birth canal. Expulsion occurs after delivery of the anterior and posterior shoulders.

Fetal circulation can be affected by many factors during labor. Accurate assessment of the laboring woman and fetus requires knowledge of these expected adaptations. Which factor will not affect fetal circulation during labor? a) Fetal position b) Uterine contractions c) Blood pressure d) Umbilical cord blood flow

a) Fetal position

A woman in active labor receives an opioid agonist analgesic. Which medication relieves severe, persistent, or recurrent pain, creates a sense of well-being, overcomes inhibitory factors, and may even relax the cervix but should be used cautiously in women with cardiac disease? a) Meperidine (Demerol) b) Promethazine (phenergan) c) Butorphanol tartrate (Stadol) d) Nalbuphine (Nubain)

a) Meperidine (Demerol)

With regard to primary and secondary powers, the maternity nurse should understand that: a) Primary powers are responsible for effacement and dilation of the cervix b) Effacement generally is well ahead of dilation in women giving birth for the first time; they are more together in subsequent pregnancies c) Scarring of the cervix caused by a previous infection or surgery may make the delivery a bit more painful, but it should not slow or inhibit dilation d) Pushing in the second stage of labor is more effective if the woman can breathe deeply and control some of her involuntary needs to push, as the nurse directs

a) Primary powers are responsible for effacement and dilation of the cervix

When teaching a group of nursing students about uterine contractions, which of the following would the instructor include as a typical feature? a) Retracting of the lower segment b) Contracting stronger in the lower segment c) Thinning out of the upper segment d) Pulling down of the cervical portion

a) Retracting of the lower segment Rationale: The nurse should identify retraction of the lower segment of the uterus as a feature of typical uterine contractions. As labor progresses the uterine contractions become stronger. The upper segment of the uterus contracts more actively than the lower segment. The lower segment retracts, pulling up the cervix. The upper segment thickens with time and the lower segment thins.

When teaching a group of nursing students about uterine contractions, which of the following would the instructor include as a typical feature? a) Retracting of the lower segment b) Pulling down of the cervical portion c) Thinning out of the upper segment d) Contracting stronger in the lower segment

a) Retracting of the lower segment Rationale: The nurse should identify retraction of the lower segment of the uterus as a feature of typical uterine contractions. As labor progresses the uterine contractions become stronger. The upper segment of the uterus contracts more actively than the lower segment. The lower segment retracts, pulling up the cervix. The upper segment thickens with time and the lower segment thins.

Nurses should be aware of the difference experience can make in labor pain, such as: a) Sensory pain for nulliparous women often is greater than for multiparous women during early labor b) Affective pain for nulliparous women usually is less than for multiparous women throughout the first stage of labor c) Women with a history of substance abuse experience more pain during labor d) Multiparous women have more fatigue from labor and therefore experience more pain

a) Sensory pain for nulliparous women often is greater than for multiparous women during early labor

A nurse is teaching a group of nursing students about the role of progesterone in labor. Which of the following should the nurse explain as the function of progesterone? a) Suppresses the uterine irritability throughout pregnancy b) Promotes oxytocin production from the posterior pituitary c) Sensitizes the uterus to effects of oxytocin on the myometrium d) Stimulates smooth muscle contraction in the uterus

a) Suppresses the uterine irritability throughout pregnancy Rationale: The function of progesterone is to suppress uterine irritability throughout pregnancy. The function of estrogen is to promote oxytocin production and to sensitize the uterus to the effects of oxytocin. Prostaglandin, and not progesterone, stimulates the smooth muscle contractions in the uterus.

When assessing a multiparous woman who has just given birth to an 8 pound boy, the nurse notes that the woman's fundus is firm and has become globular. A gush of dark red blood comes from her vagina. The nurse concludes that: a) The placenta has separated b) A cervical tear occurred during the birth c) The woman is beginning to hemorrhage d) Clots have formed in the upper uterine segment

a) The placenta has separated

The woman has just delivered a healthy baby boy, but the placenta has not yet delivered. What stage of labor does this scenario represent? a) Third b) Second c) Fourth d) First

a) Third Rationale: Stage 3 begins with the birth of the baby and ends with delivery of the placenta.

A nurse is caring for a client in her fourth stage of labor. Which of the following assessments would indicate normal physiologic changes occurring during the fourth stage of labor? Select all that apply. a) Well-contracted uterus in the midline b) Mild uterine cramping and shivering c) Increase in the blood pressure d) Decreased intra-abdominal pressure e) Decrease in the pulse rate

a) Well-contracted uterus in the midline b) Mild uterine cramping and shivering d) Decreased intra-abdominal pressure Rationale: The normal physiologic changes for which a nurse should assess during the fourth stage of labor are a well-contracted uterus in the midline of the abdomen, mild cramping pain and generalized shivering, and decreased intra-abdominal pressure. Hemodynamic changes are due to normal blood loss during delivery, causing moderate tachycardia and a slight fall in the blood pressure during the fourth stage of labor. A fall in the pulse rate and increased blood pressure are not normal findings occurring during the fourth stage of labor.

A nurse is assisting a client who is in the first stage of labor. Which of the following principles should the nurse keep in mind to help make this client's labor and birth as natural as possible? a) Women should be able to move about freely throughout labor b) The support person's access to the client should be limited to prevent the client from becoming overwhelmed c) Routine intravenous fluid should be implemented d) A woman should be allowed to assume a supine position

a) Women should be able to move about freely throughout labor Rationale: Six major concepts that make labor and birth as natural as possible are as follows: 1) labor should begin on its own, not be artificially induced; 2) women should be able to move about freely throughout labor, not be confined to bed; 3) women should receive continuous support from a caring other during labor; 4) no interventions such as intravenous fluid should be used routinely; 5) women should be allowed to assume a nonsupine position such as upright and side-lying for birth; and 6) mother and baby should be housed together after the birth, with unlimited opportunity for breast-feeding.

14. After you complete your nutritional counseling for a pregnant woman, you ask her to repeat your instructions so you can assess her understanding of the instructions given. Which statement indicates that she understands the role of protein in her pregnancy? a. "Protein will help my baby grow." b. "Eating protein will prevent me from becoming anemic." c. "Eating protein will make my baby have strong teeth after he is born." d. "Eating protein will prevent me from being diabetic."

a. "Protein will help my baby grow."

19. Which nutritional recommendation about fluids is accurate? a. A woman's daily intake should be eight to ten glasses (2.3 L) of water, milk, or juice. b. Coffee should be limited to no more than two cups, but tea and cocoa can be consumed without worry. c. Of the artificial sweeteners, only aspartame has not been associated with any maternity health concerns. d. Water with fluoride is especially encouraged because it reduces the child's risk of tooth decay.

a. A woman's daily intake should be eight to ten glasses (2.3 L) of water, milk, or juice.

17. Which statement regarding acronyms in nutrition is accurate? a. Dietary reference intakes (DRIs) consist of recommended dietary allowances (RDAs), adequate intakes (AIs), and upper limits (ULs). b. RDAs are the same as ULs except with better data. c. AIs offer guidelines for avoiding excessive amounts of nutrients. d. They all refer to green leafy vegetables, whole grains, and fruit.

a. Dietary reference intakes (DRIs) consist of recommended dietary allowances (RDAs), adequate intakes (AIs), and upper limits (ULs).

3. Which nutrient's recommended dietary allowance (RDA) is higher during lactation than during pregnancy? a. Energy (kcal) b. Iron c. Vitamin A d. Folic acid

a. Energy (kcal)

37. Identify a goal of a patient with the following nursing diagnosis: Imbalanced Nutrition: Less Than Body Requirements related to diet choices inadequate to meet nutrient requirements of pregnancy. a. Gain a total of 30 lb. b. Take daily supplements consistently. c. Decrease intake of snack foods. d. Increase intake of complex carbohydrates.

a. Gain a total of 30 lb.

18. With regard to protein in the diet of pregnant women, nurses should be aware that: a. Many protein-rich foods are also good sources of calcium, iron, and B vitamins. b. Many women need to increase their protein intake during pregnancy. c. As with carbohydrates and fat, no specific recommendations exist for the amount of protein in the diet. d. High-protein supplements can be used without risk by women on macrobiotic diets.

a. Many protein-rich foods are also good sources of calcium, iron, and B vitamins.

As the United States and Canada continue to become more culturally diverse, it is increasingly important for the nursing staff to recognize a wide range of varying cultural beliefs and practices. Nurses need to develop respect for these culturally diverse practices and learn to incorporate these into a mutually agreed on plan of care. Although it is common practice in the United States for the father of the baby to be present at the birth, in many societies this is not the case. When implementing care, the nurse would anticipate that a woman from which country would have the father of the baby in attendance? a. Mexico b. China c. Iran d. India

a. Mexico A woman from Mexico may be stoic about discomfort until the second stage, at which time she will request pain relief. Fathers and female relatives are usually in attendance during the second stage of labor. The father of the baby is expected to provide encouragement, support, and reassurance that all will be well. Fathers are usually not present in China. The Iranian father will not be present. Female support persons and female care providers are preferred. For many, a male caregiver is unacceptable. The father is usually not present in India, but female relatives are usually present. Natural childbirth methods are preferred.

6. A pregnant woman reports that she is still playing tennis at 32 weeks of gestation. The nurse would be most concerned that during and after tennis matches this woman consumes: a. Several glasses of fluid. b. Extra protein sources such as peanut butter. c. Salty foods to replace lost sodium. d. Easily digested sources of carbohydrate.

a. Several glasses of fluid.

26. To help a woman reduce the severity of nausea caused by morning sickness, the nurse might suggest that she: a. Try a tart food or drink such as lemonade or salty foods such as potato chips. b. Drink plenty of fluids early in the day. c. Brush her teeth immediately after eating. d. Never snack before bedtime.

a. Try a tart food or drink such as lemonade or salty foods such as potato chips.

With regard to spinal and epidural (block) anesthesia, nurses should know that: a) This type of anesthesia is commonly used for cesarean births but it not suitable for vaginal births b) A high incidence of postbirth headache is seen with spinal blocks c) Epidural blocks allow the woman to move freely d) Spinal and epidural blocks are never used together

b) A high incidence of postbirth headache is seen with spinal blocks

Assessment reveals that a woman's cervix is approximately 1 cm in length. The nurse would document this as: a) 100% effaced b) 50% effaced c) 0% effaced d) 75% effaced

b) 50% effaced Rationale: A cervix 1 cm in length is described as 50% effaced. A cervix that measures approximately 2 cm in length is described as 0% effaced. A cervix 1/2 cm in length would be described as 75% effaced. A cervix 0 cm in length would be described as 100% effaced.

The first stage of labor is often a time of introspection. In light of this, which of the following would guide your planning of nursing care? a) A woman should be left entirely alone during this period. b) A woman may spend time thinking about what is happening to her. c) No nursing care is needed to be done during this time. d) A woman will rarely speak or laugh during this period.

b) A woman may spend time thinking about what is happening to her. Rationale: Women need a support person with them during all stages of labor.

Nurses can help their clients by keeping them informed about the distinctive stages of labor. What description of the phases of the first stage of labor is accurate? a) Latent: milk, regular contractions; no dilation; blood show; duration of 2 to 4 hours b) Active: moderate, regular contractions; 4 to 7cm dilation; duration of 3 to 6 hours c) Lull: no contractions; dilation stable; duration of 20 to 60 minutes d) Transition: very strong but irregular contractions; 8 to 10cm dilation; duration of 1 to 2 hours

b) Active: moderate, regular contractions; 4 to 7cm dilation; duration of 3 to 6 hours

There are four essential components of labor. The first is the passageway. It is composed of the bony pelvis and soft tissues. What is one component of the passageway? a) Perineum b) Cervix c) False pelvis d) Uterus

b) Cervix Rationale: The cervix and vagina are soft tissues that form the part of the passageway known as the birth canal.

The nurse is measuring a contraction from the beginning of the increment to the end of the decrement for the same contraction, documenting this as which of the following? a) Peak b) Duration c) Frequency d) Intensity

b) Duration Rationale: Duration refers to how long a contraction lasts and is measured from the beginning of the increment to the end of the decrement for the same contraction. Intensity refers to the strength of the contraction determined by manual palpation or measured by an internal intrauterine catheter. Frequency refers to how often contractions occur and is measured from the increment of one contraction to the increment of the next contraction. The peak or acme of a contraction is the highest intensity of a contraction.

With regard to systemic analgesics administered during labor, nurses should be aware that: a) Systemic analgesics cross the maternal blood-brain barrier as easily as they do the fetal blood-brain barrier b) Effects on the fetus and newborn can include decreased alertness and delayed sucking c) IM administration is preferred over IV administration d) IV patient-controlled analgesia (PCA) results in increased use of an analgesic

b) Effects on the fetus and newborn can include decreased alertness and delayed sucking

All statements about normal labor are true except: a) A single fetus presents by vertex b) It is completed within 8 hours c) A regular progression of contractions, effacement, dilation, and descent occurs d) No complications are involved

b) It is completed within 8 hours

A woman who is 39 weeks pregnant expresses fear about her impending labor and how she will manage. The nurse's best response is: a) Dont worry about it. You'll do fine b) It's normal to be anxious about labor. Let's discuss what makes you afraid c) Labor is scary to think about, but the actual experience isn't d) You may have an epidural. You won't feel anything

b) It's normal to be anxious about labor. Let's discuss what makes you afraid

The nurse assesses a client in labor and finds that the fetal long axis is longitudinal to the maternal long axis. The nurse documents this finding as which of the following? a) Attitude b) Lie c) Position d) Presentation

b) Lie Rationale: The nurse is assessing fetal lie, the relationship of the fetal long axis to the maternal long axis. When the fetal long axis is longitudinal to the maternal long axis, the lie is said to longitudinal. Presentation is the portion of the fetus that overlies the maternal pelvic inlet. Attitude is the relationship of the different fetal parts to one another. Position is the relationship of the fetal denominator to the different sides of the maternal pelvis.

A 32-year-old woman presents to the labor-and-delivery suite in active labor. She is multigravida, relaxed, and talking with her husband. When examined by the nurse, the fetus is found to be in a cephalic presentation. His occiput is facing toward the front and slightly to the right of the mother's pelvis, and he is exhibiting a flexed attitude. How does the nurse document the position of the fetus? a) LOA b) ROA c) LOP d) ROP

b) ROA Rationale: Document the fetal position in the clinical record using abbreviations (Box 8-1). The first letter describes the side of the maternal pelvis toward which the presenting part is facing ("R" for right and "L" for left). The second letter or abbreviation indicates the reference point ("O" for occiput, "Fr" for frontum, etc.). The last part of the designation specifies whether the presenting part is facing the anterior (A) or the posterior (P) portion of the pelvis, or whether it is in a transverse (T) position.

After change of shift report the nurse assumes care of a multiparous client in labor. The woman is complaining of pain that radiates to her abdominal wall, lower back, buttocks, and down her thighs. Before implementing a plan of care, the nurse should understand that this type of pain is: a) Visceral b) Referred c) Somatic d) Afterpain

b) Referred

A nurse is performing a vaginal examination of a woman in the early stages of labor. The woman has been at 2 cm dilated for the past 2 hours, but effacement has progressed steadily. Which of the following should the nurse do to best encourage the client regarding her progress? a) Don't mention anything to the client yet; wait for further dilatation to occur. b) Say, "you are still 2 cm dilated, but the cervix is thinning out nicely." c) Say, "you haven't dilated any further, but hang in there; it will happen eventually." d) Say, "there has been no further dilatation; effacement is progressing."

b) Say, "you are still 2 cm dilated, but the cervix is thinning out nicely." Rationale: Women are anxious to have frequent reports during labor, to reassure them everything is progressing well. If giving a progress report, remember most women are aware of the word dilatation but not effacement. Just saying, "no further dilatation", therefore, is a depressing report. "You're not dilated a lot more, but a lot of thinning is happening and that's just as important" is the same report given in a positive manner.

One of the theories about the onset of labor is the prostaglandin theory. While not being conclusively proven that the action of prostaglandins initiate labor, it is known that prostaglandins do play a role in labor. What is an action of prostaglandins? a) Stimulates uterine muscle to relax b) Softens cervix c) Initiates relaxation of perineum d) Initiates cervical dilation

b) Softens cervix Rationale: The prostaglandin theory is another theory of labor initiation. Prostaglandins influence labor in several ways, which include softening the cervix and stimulating the uterus to contract. However, evidence supporting the theory that prostaglandins are the agents that trigger labor to begin is inconclusive.

Concerning the third stage of labor, nurses should be aware that: a) The placenta eventually detaches itself from a flaccid uterus b) The duration of the third stage may be as short as 3 to 5 minutes c) It is important that the dark, roughened maternal surface of the placenta appear before the shiny fetal surface d) The major risk for women during the third stage is a rapid heart rate

b) The duration of the third stage may be as short as 3 to 5 minutes

Vaginal examinations should be performed by the nurse under all these circumstances except: a) An admission to the hospital at the start of labor b) When accelerations of the fetal heart rate (FHR) are noted c) On maternal perception of perineal pressure or the urge to bear down d) When membranes ruptured

b) When accelerations of the fetal heart rate (FHR) are noted

7. Which statement made by a lactating woman would lead the nurse to believe that the woman might have lactose intolerance? a. "I always have heartburn after I drink milk." b. "If I drink more than a cup of milk, I usually have abdominal cramps and bloating." c. "Drinking milk usually makes me break out in hives." d. "Sometimes I notice that I have bad breath after I drink a cup of milk."

b. "If I drink more than a cup of milk, I usually have abdominal cramps and bloating."

A patient whose cervix is dilated to 5 cm is considered to be in which phase of labor? a. Latent phase b. Active phase c. Second stage d. Third stage

b. Active phase The latent phase is from the beginning of true labor until 3 cm of cervical dilation. The active phase of labor is characterized by cervical dilation of 4 to 7 cm. The second stage of labor begins when the cervix is completely dilated until the birth of the baby. The third stage of labor is from the birth of the baby until the expulsion of the placenta. This patient is in the active phase of labor.

27. Three servings of milk, yogurt, or cheese plus two servings of meat, poultry, or fish adequately supply the recommended amount of protein for a pregnant woman. Many patients are concerned about the increased levels of mercury in fish and may be afraid to include this source of nutrients in their diet. Sound advice by the nurse to assist the client in determining which fish is safe to consume would include: a. Canned white tuna is a preferred choice. b. Avoid shark, swordfish, and mackerel. c. Fish caught in local waterways are the safest. d. Salmon and shrimp contain high levels of mercury.

b. Avoid shark, swordfish, and mackerel.

22. With regard to nutritional needs during lactation, a maternity nurse should be aware that: a. The mother's intake of vitamin C, zinc, and protein now can be lower than during pregnancy. b. Caffeine consumed by the mother accumulates in the infant, who may be unusually active and wakeful. c. Critical iron and folic acid levels must be maintained. d. Lactating women can go back to their prepregnant calorie intake.

b. Caffeine consumed by the mother accumulates in the infant, who may be unusually active and wakeful.

38. In teaching the pregnant adolescent about nutrition, the nurse should: a. Emphasize the need to eliminate common teen snack foods because they are too high in fat and sodium. b. Determine the weight gain needed to meet adolescent growth and add 35 lb. c. Suggest that she not eat at fast-food restaurants to avoid foods of poor nutritional value. d. Realize that most adolescents are unwilling to make dietary changes during pregnancy.

b. Determine the weight gain needed to meet adolescent growth and add 35 lb.

5. A pregnant woman experiencing nausea and vomiting should: a. Drink a glass of water with a fat-free carbohydrate before getting out of bed in the morning. b. Eat small, frequent meals (every 2 to 3 hours). c. Increase her intake of high-fat foods to keep the stomach full and coated. d. Limit fluid intake throughout the day.

b. Eat small, frequent meals (every 2 to 3 hours).

33. The most important reason for evaluating the pattern of weight gain in pregnancy is to: a. Prevent excessive adipose tissue deposits b. Identify potential nutritional problems or complications of pregnancy c. Assess the need to limit caloric intake in obese women d. Determine cultural influences on the woman's diet

b. Identify potential nutritional problems or complications of pregnancy

13. Women with an inadequate weight gain during pregnancy are at higher risk of giving birth to an infant with: a. Spina bifida. b. Intrauterine growth restriction. c. Diabetes mellitus. d. Down syndrome.

b. Intrauterine growth restriction

16. Maternal nutritional status is an especially significant factor of the many factors that influence the outcome of pregnancy because: a. It is very difficult to adjust because of people's ingrained eating habits. b. It is an important preventive measure for a variety of problems. c. Women love obsessing about their weight and diets. d. A woman's preconception weight becomes irrelevant.

b. It is an important preventive measure for a variety of problems.

Which of the following would alert the nurse that the client is in the transition phase of labor? a) Reduction of rectal pressure b) Decrease in the bloody show c) Beginning urge to bear down d) Enthusiasm in the client

c) Beginning urge to bear down Rationale: Starting of the urge to bear down is a feature associated with the transition phase of labor. The transition phase is the last phase of the first stage of labor. In this phase the process of cervical dilatation is completed. During this phase the client experiences an increase in rectal pressure, an increase in the bloody show and an urge to bear down. The contractions are stronger and hence the client feels irritable, restless and nauseous. The client feels enthusiastic during the latent phase and not the transition phase.

A woman is documented on the labor and delivery board to be 7cm dilated. Her family wants to know how long she will be in labor. The nurse should provide which information to the family? a) "She is doing well, in the second stage and it could be anytime now." b) "She is in the transition phase of labor and it will be with in 2 to 3 hours, might be sooner." c) "She is in active labor, she is progressing at this point and we will keep you posted." d) "She is still in early latent labor and has much too long to go to tell when she will deliver."

c) "She is in active labor, she is progressing at this point and we will keep you posted." Rationale: At 7cm dilated, she is considered in Active Phase of labor. There is no science that can predict the length of labor. She is progressing in labor and it is not best to give the family a specific time frame.

A fetus is assessed at 2 cm above the ischial spines. The nurse would document fetal station as: a) +2 b) 0 c) -2 d) +4

c) -2 Rationale: When the presenting part is above the ischial spines, it is noted as a negative station. Since the measurement is 2 cm, the station would be -2. A 0 station indicates that the fetal presenting part is at the level of the ischial spines. Positive stations indicate that the presenting part is below the level of the ischial spines.

The nurse should initially implement which intervention when a nulliparous woman telephones the hospital to report that she is in labor. a) Emphasize that food and fluid should stop or be light b) Tell the woman to stay home until her membranes rupture c) Ask the woman to describe why she believes that she is in labor d) Arrange for the woman to come to the hospital for labor evaluation

c) Ask the woman to describe why she believes that she is in labor Rationale: The nurse needs further information to assist in determining if the woman is in true or false labor. She will need to ask the patient questions to seek further assessment and triage information. Having her wait until membranes rupture may be dangerous, as she may give birth before reaching the hospital. She should continue fluid intake until it is determined whether or not she is in labor. She may be in false labor, and more information should be obtained before she is brought to the hospital.

The nurse knows that the second stage of labor, the descent phase, has begun when: a) The amniotic membranes rupture b) The cervix cannot be felt during a vaginal examination c) The woman experiences a strong urge to bear down d) The presenting part is below the ischial spines

c) The woman experiences a strong urge to bear down

Braxton Hicks contractions are termed "practice contractions" and occur throughout pregnancy. When the woman's body is getting ready to go into labor, it begins to show anticipatory signs of impending labor. Among these signs are Braxton Hicks contractions that are more frequent and stronger in intensity. What differentiates Braxton Hicks contractions from true labor? a) Braxton Hicks contractions get closer together with activity b) Braxton Hicks contractions cause "ripening" of the cervix. c) Braxton Hicks contractions usually decrease in intensity with walking d) Braxton Hicks contractions do not last long enough to be true labor

c) Braxton Hicks contractions usually decrease in intensity with walking Rationale: Braxton Hicks contractions occur more frequently and are more noticeable as pregnancy approaches term. These irregular, practice contractions usually decrease in intensity with walking and position changes.

There are four essential components of labor. The first is the passageway. It is composed of the bony pelvis and soft tissues. What is one component of the passageway? a) Uterus b) False pelvis c) Cervix d) Perineum

c) Cervix Rationale: The cervix and vagina are soft tissues that form the part of the passageway known as the birth canal.

Which description of the pases of the second stage of labor is accurate? a) Latent phase: feels sleepy, fetal station is 2+ to 4+, duration is 30 to 45 minutes b) Active phase: overwhelmingly strong contractions, Ferguson reflex activated, duration is 5 to 15 minutes c) Descent phase: significant increase in contractions, Ferguson reflex activated, average duration varies d) Transitional phase: woman "laboring down," fetal station is 0, duration is 15 minutes

c) Descent phase: significant increase in contractions, Ferguson reflex activated, average duration varies

Evidence-based care practices designed to support normal labor and birth recommend which practice during the immediate newborn period? a) The healthy newborn should be taken to the nursery for a complete assessment b) After drying, the infant should be given to the mother wrapped in a receiving blanker c) Encourage skin-to-skin contact of mother and baby d) The father or support persons should be encouraged to hold the infant while awaiting delivery of the placenta

c) Encourage skin-to-skin contact of mother and baby

With regard to what might be called the tactile approaches to comfort management, nurses should be aware that: a) Either hot or cold applications may provide relief, but they should never be used together in the same treatment b) Acupuncture can be performed by a skilled nurse with just a little training c) Hand and foot massage may be especially relaxing in advanced labor when a womans tolerance for touch is limited d) Therapeutic touch uses handheld electronic stimulators that produce sympathetic vibrations

c) Hand and foot massage may be especially relaxing in advanced labor when a womans tolerance for touch is limited

If a fetus is in an ROA position during labor, you would interpret this to mean the fetus is a) Presenting with the face as the presenting part b) In a common breech delivery position c) In a longitudinal lie facing the left posterior d) Facing the right anterior abdominal quadrant

c) In a longitudinal lie facing the left posterior Rationale: ROA (right occiput anterior) means the occiput of the fetal head points toward the mother's right anterior pelvis; the head is the presenting part.

The skull is the most important factor in relation to the labor and birth process. The fetal skull must be small enough to travel through the bony pelvis. What feature of the fetal skull helps to make this passage possible? a) Vertex presentation b) Caput succedaneum c) Molding d) Cephalohematoma

c) Molding Rationale: The cartilage between the bones allows the bones to overlap during labor, a process called molding that elongates the fetal skull thereby reducing the diameter of the head.

The four essential components of labor are known as the "four P's". Which of the four P's involves the pelvis? a) Psyche b) Powers c) Passageway d) Passenger

c) Passageway Rationale: The passageway is one of the 4 P's and involves the pelvis, both bony pelvis and the soft tissues, cervix, and vagina. The "passenger" refers to the fetus. The primary powers are the involuntary contractions of the uterus, whereas the secondary powers come from the maternal abdominal muscles. The psyche refers to the mother's mental state.

When educating a group of nursing students about the theories of onset of labor, the nurse identifies which of the following factors as the possible causes for onset of labor? Select all that apply. a) Increase in the production of progesterone b) Fall in the estrogen at 34-35 weeks of pregnancy c) Prostaglandin production in the myometrium d) Increase in the fetal cortisol levels e) Release of oxytocin by the pituitary

c) Prostaglandin production in the myometrium d) Increase in the fetal cortisol levels e) Release of oxytocin by the pituitary Rationale: The possible causes for the onset of labor include increase in the fetal cortisol levels, release of oxytocin by the posterior pituitary and the production of prostaglandins. Progesterone withdrawal, and not an increase, initiates labor. There is a rise in the estrogen levels at 34-35 weeks of pregnancy. Estrogen stimulates prostaglandin production and also promotes the release of oxytocin.

In preparing for the actual birth, which fetal presentation would a nurse be least likely to find? a) Transverse lie b) Breech c) Shoulder d) Oblique lie

c) Shoulder Rationale: Shoulder presentations are the least likely to occur in less than 0.3 percent of all births. Approximately 97 percent of fetuses are in a cephalic presentation at the end of pregnancy. A longitudinal lie, in which the long axis of the fetus is parallel to the long axis of the mother, is the most common. When the fetus is in a transverse lie, the long axis of the fetus is perpendicular to the long axis of the woman. An oblique lie is in between the two. (

Which position would the nurse suggest for second-stage labor if the pelvic outlet needs to be increased? a) Semirecumbent b) Sitting c) Squatting d) Side-lying

c) Squatting

A post delivery CBC has noted an elevated WBC count of 22,000/mm3. Which rationale is accurate regarding the elevated WBC count? a) Abnormal finding and she needs antibiotics b) Might be a false result, recommend re-testing c) This is a normal variation due to stress of labor d) Occurs in patients who have cesarean birth, from the trauma of surgery

c) This is a normal variation due to stress of labor Rationale: An elevation of WBC up to 30,000mm/3 can be normal variation for any woman after delivery. This is related to the stress on her body from labor and delivery. Antibiotics are not indicated as this is a normal response to intense stress. The increase in WBC is not related to cesarean birth. Retesting would be wasteful as it is known that this is a normal response to any stress.

A nurse teaches a pregnant woman about the characteristics of true labor contractions. The nurse evaluates her understanding of the instructions when the woman states: a) True labor contractions will subside when I walked around b) True labor contractions will cause discomfort over the top of my uterus c) True labor contractions will continue and get stronger even if I relax and take a shower d) True labor contractions will remain irregular but become stronger

c) True labor contractions will continue and get stronger even if I relax and take a shower

9. A 27-year-old pregnant woman had a preconceptual body mass index (BMI) of 18.0. The nurse knows that this woman's total recommended weight gain during pregnancy should be at least: a. 20 kg (44 lb). b. 16 kg (35 lb). c. 12.5 kg (27.5 lb). d. 10 kg (22 lb).

c. 12.5 kg (27.5 lb).

34. If a patient's normal prepregnancy diet contains 45 g of protein daily, how many more grams of protein should she consume per day during pregnancy? a. 5 b. 10 c. 25 d. 30

c. 25

2. Which meal would provide the most absorbable iron? a. Toasted cheese sandwich, celery sticks, tomato slices, and a grape drink b. Oatmeal, whole wheat toast, jelly, and low-fat milk c. Black bean soup, wheat crackers, orange sections, and prunes d. Red beans and rice, cornbread, mixed greens, and decaffeinated tea

c. Black bean soup, wheat crackers, orange sections, and prunes

32. When providing care to the prenatal patient, the nurse understands that pica is defined as: a. Intolerance of milk products b. Iron deficiency anemia c. Ingestion of nonfood substances d. Episodes of anorexia and vomiting

c. Ingestion of nonfood substances

20. Which minerals and vitamins usually are recommended to supplement a pregnant woman's diet? a. Fat-soluble vitamins A and D b. Water-soluble vitamins C and B6 c. Iron and folate d. Calcium and zinc

c. Iron and folate

15. Pregnant adolescents are at high risk for _____ because of lower body mass indices (BMIs) and "fad" dieting. a. Obesity b. Diabetes c. Low-birth-weight babies d. High-birth-weight babies

c. Low-birth-weight babies

23. While taking a diet history, the nurse might be told that the expectant mother has cravings for ice chips, cornstarch, and baking soda. This represents a nutritional problem known as: a. Preeclampsia. b. Pyrosis. c. Pica. d. Purging.

c. Pica.

25. The labor and delivery nurse is preparing a bariatric patient for an elective cesarean birth. Which piece of "specialized" equipment is unnecessary when providing care for this pregnant woman. a. Extra long surgical instruments b. Wide surgical table c. Temporal thermometer d. Increased diameter blood pressure cuff

c. Temporal thermometer

4. A pregnant woman's diet consists almost entirely of whole grain breads and cereals, fruits, and vegetables. The nurse would be most concerned about this woman's intake of: a. Calcium. b. Protein. c. Vitamin B12. d. Folic acid.

c. Vitamin B12.

what medications should be avoided in moms with asthma

carboprost and methylergonovine

what is the difference of cardiovascular disorder classes in pregnancy

class I * asymptomatic without limitations of physical activity class II * symptomatic with slight limitation of activity class III * symptomatic with marked limitation of activity class IV * symptomatic with inability to carry on any physical activity without discomfort

The patient is having a routine prenatal visit and asks the nurse what the childbirth education teacher meant when she used the term zero station. What is the best response by the nurse? a) "This is just a way of determining your progress in labor." b) This indicates that you start labor within the next 24 hours." c) "This means +1 and the baby is entering the true pelvis." d) "The presenting part is at the true pelvis and is engaged."

d) "The presenting part is at the true pelvis and is engaged." Rationale: Zero station is when the fetus is engaged in the pelvis, or has dropped. This is an encouraging sign for the patient. This sign is indicative that labor may be beginning, but there is no set time frame regarding when it will start. Labor has not started yet, and the fetus has not begun to move out of the uterus.

A fetus is assessed at 2 cm above the ischial spines. The nurse would document fetal station as: a) +4 b) +2 c) 0 d) -2

d) -2 Rationale: When the presenting part is above the ischial spines, it is noted as a negative station. Since the measurement is 2 cm, the station would be -2. A 0 station indicates that the fetal presenting part is at the level of the ischial spines. Positive stations indicate that the presenting part is below the level of the ischial spines.

A pregnant woman, multipara, has been in labor for several hours. She cries out that her contractions are getting harder and that she can't do this. The patient is really irritable, nauseated, annoyed, and fearful of being left alone. Considering the client's behavior, the nurse would expect the cervix to be dilated how many centimeters? a) 5-6 b) 0-2 c) 3-5 d) 8-10

d) 8-10 Rationale: The reaction of the patient is indicative of entering or being in the transition phase of labor, stage 1. The dilation would be 8-10 cm. Before that, when dilation is 0-7 cm, the patient has an easier time using positive coping skills.

Nurses can advise their clients that all are signs that precede labor except: a) A return of urinary frequency as a result of increased bladder pressure b) Persistent low backache from relaxed pelvic joints c) Stronger and more frequent uterine (Braxton Hicks) contractions d) A decline in energy, as the body stores up for labor

d) A decline in energy, as the body stores up for labor

Which of the following would be least effective in promoting a positive birth outcome for a woman in labor? a) Promoting the woman's feelings of control b) Providing clear information about procedures c) Encouraging the woman to use relaxation techniques d) Allowing the woman time to be alone

d) Allowing the woman time to be alone Rationale: Positive support, not being alone, promotes a positive birth experience. Being alone can increase anxiety and fear, decreasing the woman's ability to cope. Feelings of control promote self-confidence and self-esteem, which in turn help the woman to cope with the challenges of labor. Information about procedures reduces anxiety about the unknown and fosters cooperation and self-confidence in her abilities to deal with labor. Catecholamines are secreted in response to anxiety and fear and can inhibit uterine blood flow and placental perfusion. Relaxation techniques can help to reduce anxiety and fear, in turn decreasing the secretion of catecholamines and ultimately improving the woman's ability to cope with labor.

What term is used to describe the position of the fetal long axis in relation to the long axis of the mother? a) Fetal position b) Fetal presentation c) Fetal attitude d) Fetal lie

d) Fetal lie Rationale: Fetal lie describes the position of the long axis of the fetus in relation to the long axis of the pregnant woman.

The third stage of labor is considered to be expulsion of the placenta. This stage can last anywhere from five to 20 minutes. What is a sign that the placenta is separating from the wall of the uterus? a) Expulsion of blood clots b) Firm fundus c) Shortening of the umbilical cord d) Globular shape to the fundus

d) Globular shape to the fundus Rationale: Signs that indicate the placenta is separating from the uterine wall include a gush of blood, lengthening of the umbilical cord, and a globular shape to the fundus.

A new OB/GYN physician has just finished evaluating her one hundredth patient. In reviewing the documentation from all patients thus far, which types of pelvis would you assume the physician has seen the most and the least? a) Android and platypelloid, respectively b) Gynecoid and android, respectively c) Anthropoid and gynecoid, respectively d) Gynecoid and platypelloid, respectively

d) Gynecoid and platypelloid, respectively Rationale: The gynecoid is the typical female pelvis shape; platypelloid pelvis is the least common type of pelvis in women. Women with anthropoid pelvic shapes are able to deliver normally one third of the time, and are somewhat rare. An android pelvis is similar to a male pelvis, and is seen in 16% of nonwhite women.

A woman in the active phase of the first stage of labor is using a shallow pattern of breathing, which is about twice the normal adult breathing rate. She starts to complain about feeling lightheaded and dizzy and states that her fingers are tingling. The nurse should: a) Notify the woman's physician b) Tell the woman to slow the pace of her breathing c) Administer oxygen via mask or nasal canula d) Help her breathe into a paper bag

d) Help her breathe into a paper bag

If a woman complains of back labor pain, the nurse might best suggest that she: a) Lie on her back for a while with her knees bent b) Do less walking around c) Take some deep, cleansing breaths d) Lean over a birth ball with her knees on the floor

d) Lean over a birth ball with her knees on the floor

Assessment reveals that the fetus of a client in labor is in the vertex presentation. The nurse determines that the presenting part is which of the following? a) Brow b) Buttocks c) Shoulders d) Occiput

d) Occiput Rationale: With a vertex presentation, a type of cephalic presentation, the fetal presenting part is the occiput. The shoulders are the presenting part when the fetus is in a shoulder presentation. The brow or sinciput is the presenting part when a fetus is in a brow presentation. The buttocks are the presenting part when a fetus is in a breech presentation.

A client in labor has been admitted to the labor and birth suite. The nurse assessing her notes that the fetus is in a cephalic presentation. Which of the following should the nurse identify by the term presentation? a) Relation of the different fetal body parts to one another b) Relationship of the presenting part to the maternal pelvis c) Relation of the fetal presenting part to the maternal ischial spine d) Part of the fetal body entering the maternal pelvis first

d) Part of the fetal body entering the maternal pelvis first Rationale: The term presentation is the part of the fetal body that is entering the maternal pelvis first. Relationship of the presenting part to the sides of the maternal pelvis is called the position. Attitude is the term that describes the relation of the different fetal body parts to one another. Relation of the fetal presenting part to maternal ischial spine is termed the station.

When going through the transition phase of labor women often feel out of control. What do women in the transition phase of labor need? a) Their significant other beside them b) Intense nursing care c) Just to be left alone d) Positive reinforcement

d) Positive reinforcement Rationale: Any women, even ones who have had natural childbirth classes, have a difficult time maintaining positive coping strategies during this phase of labor. Many women describe feeling out of control during this phase of labor. A woman in transition needs support, encouragement, and positive reinforcement.

A 32-year-old woman presents to the labor-and-delivery suite in active labor. She is multigravida, relaxed, and talking with her husband. When examined by the nurse, the fetus is found to be in a cephalic presentation. His occiput is facing toward the front and slightly to the right of the mother's pelvis, and he is exhibiting a flexed attitude. How does the nurse document the position of the fetus? a) ROP b) LOP c) LOA d) ROA

d) ROA Rationale: Document the fetal position in the clinical record using abbreviations (Box 8-1). The first letter describes the side of the maternal pelvis toward which the presenting part is facing ("R" for right and "L" for left). The second letter or abbreviation indicates the reference point ("O" for occiput, "Fr" for frontum, etc.). The last part of the designation specifies whether the presenting part is facing the anterior (A) or the posterior (P) portion of the pelvis, or whether it is in a transverse (T) position.

In order to accurately assess the health of the mother accurately during labor, the nurse should be aware that: a) The woman's blood pressure increases during contractions and falls back to prelabor normal between contractions b) Use of the Valsalva maneuver is encouraged during the second stage of labor to relieve fetal hypoxia c) Having the woman point her toes reduces leg cramps d) The endogenous endorphins released during labor raise the woman's pain threshold and produce sedation

d) The endogenous endorphins released during labor raise the woman's pain threshold and produce sedation

During the second stage of labor, a woman is generally a) very aware of activities immediately around her. b) anxious to have people around her. c) no longer in need of a support person. d) turning inward to concentrate on body sensations.

d) turning inward to concentrate on body sensations. Rationale: Second-stage contractions are so unusual that most women are unable to think of things other than what is happening inside their body.

11. A woman has come to the clinic for preconception counseling because she wants to start trying to get pregnant in 3 months. She can expect the following advice: a. "Discontinue all contraception now." b. "Lose weight so that you can gain more during pregnancy." c. "You may take any medications you have been taking regularly." d. "Make sure that you include adequate folic acid in your diet."

d. "Make sure that you include adequate folic acid in your diet."

8. A pregnant woman's diet history indicates that she likes the following list of foods. The nurse would encourage this woman to consume more of which food to increase her calcium intake? a. Fresh apricots b. Canned clams c. Spaghetti with meat sauce d. Canned sardines

d. Canned sardines

30. The major source of nutrients in the diet of a pregnant woman should be composed of: a. Simple sugars b. Fats c. Fiber d. Complex carbohydrates

d. Complex carbohydrates

24. When counseling a client about getting enough iron in her diet, the maternity nurse should tell her that: a. Milk, coffee, and tea aid iron absorption if consumed at the same time as iron. b. Iron absorption is inhibited by a diet rich in vitamin C. c. Iron supplements are permissible for children in small doses. d. Constipation is common with iron supplements.

d. Constipation is common with iron supplements.

10. A woman in week 34 of pregnancy reports that she is very uncomfortable because of heartburn. The nurse would suggest that the woman: a. Substitute other calcium sources for milk in her diet. b. Lie down after each meal. c. Reduce the amount of fiber she consumes. d. Eat five small meals daily.

d. Eat five small meals daily.

28. Nutrition is one of the most significant factors influencing the outcome of a pregnancy. It is an alterable and important preventive measure for various potential problems, such as low birth weight and prematurity. While completing the physical assessment of the pregnant client, the nurse can evaluate the client's nutritional status by observing a number of physical signs. Which sign would indicate that the client has unmet nutritional needs? a. Normal heart rate, rhythm, and blood pressure b. Bright, clear, shiny eyes c. Alert, responsive, and good endurance d. Edema, tender calves, and tingling

d. Edema, tender calves, and tingling

31. A pregnant woman's diet may not meet her need for folates. A good source of this nutrient is: a. Chicken b. Cheese c. Potatoes d. Green leafy vegetables

d. Green leafy vegetables

36. To determine the cultural influence on a patient's diet, the nurse should first: a. Evaluate the patient's weight gain during pregnancy b. Assess the socioeconomic status of the patient c. Discuss the four food groups with the patient d. Identify the food preferences and methods of food preparation common to that culture

d. Identify the food preferences and methods of food preparation common to that culture

Why do insulin requirements decrease during the first 24 hours after delivering

placenta is the main source of insulin resistance

1. A 22-year-old woman pregnant with a single fetus has a preconception body mass index (BMI) of 24. When she was seen in the clinic at 14 weeks of gestation, she had gained 1.8 kg (4 lb) since conception. How would the nurse interpret this? a. This weight gain indicates possible gestational hypertension. b. This weight gain indicates that the woman's infant is at risk for intrauterine growth restriction (IUGR). c. This weight gain cannot be evaluated until the woman has been observed for several more weeks. d. The woman's weight gain is appropriate for this stage of pregnancy.

d. The woman's weight gain is appropriate for this stage of pregnancy.

21. Which vitamins or minerals can lead to congenital malformations of the fetus if taken in excess by the mother? a. Zinc b. Vitamin D c. Folic acid d. Vitamin A

d. Vitamin A

29. Which pregnant woman should restrict her weight gain during pregnancy? a. Woman pregnant with twins b. Woman in early adolescence c. Woman shorter than 62 inches or 157 cm d. Woman who was 20 pounds overweight before pregnancy

d. Woman who was 20 pounds overweight before pregnancy

what can trigger sickle cell crisis

dehydration, hypoxia, acidosis

How often should an A1C be drawn and what should the % be during pregnancy

every 3 months <6

if mom has diabetes and is delivering by c section, when should the section be scheduled and how should insulin be adjusted for delivery

early in the morning take full dose of insulin the night before and nothing the morning of

when do you take iron suppliments

empty stomach

What medications might a mom with gestational diabetes be prescribed

insulin glyburide (does not cross placenta) metformin (crosses placenta/safe)

What insulin needs are most common with a mom who has pregestational diabetes

insulin dependent almost always

medications for epilepsy do what to oral contraceptives

interfere

what is the most common type of anemia in pregnancy

iron deficiency anemia

what foods are good to eat with iron deficiency anemia

liver meat whole grain dark green leafy vegetables legumes dried fruits

What is the management goal of gestational diabetes

maintain normal glucose levels

what is used as opiod replacement therapy

methadone

What can high levels of PKU cause

microcephaly cognitive impairment congenital heart defects

subjective s/s of cardiac decompensation

o Increasing fatigue and/or difficulty breathing with usually activities o Feeling of smothering o Frequent cough o Palpitations-heart "racing" o Generalized edema; swelling of face, feet, legs, fingers

objective s/s of cardiac decompensation

o Irregular, weak, rapid pulse (>100 bpm) o Progressive, generalized edema o Crackles at base of lungs that does not clear with coughing o Orthopnea o Rapid respirations (>25) o Moist, frequent cough o Cyanosis of lips and nailbeds

What are the maternal risks/complications of diabetes during pregnancy

o Miscarriage o Macrosomia Baby gets too big Can cause shoulder dystocia o Hydramnios o Increased infections o Ketoacidosis Can lead to metabolic acidosis

what are the effects on pregnancy with a mom who has asthma

o Preterm birth, preeclampsia, small for gestational age, IUGR (intrauterine growth restriction), increased cesarean birth

What should nursing mothers who have PKU do

stop breastfeeding

What cardiovascular disorders should be treated with prophylactic antibiotics

unrepaired cardiac defects mitral valve prolapse


Ensembles d'études connexes

chapter 3 review (legal aspects)

View Set

International Business Transactions -- Chapter 2 International Business Treaties

View Set

Introduction to Business chp 1 review

View Set

Skin and Subcutaneous Tissue infections

View Set

Lecture 2: Sustainability: Definitions, Principles, and Examples

View Set